Download C. Opisthorchis felineus

Document related concepts

Social history of viruses wikipedia , lookup

Globalization and disease wikipedia , lookup

Virus quantification wikipedia , lookup

History of virology wikipedia , lookup

Chickenpox wikipedia , lookup

Schistosoma mansoni wikipedia , lookup

Infection wikipedia , lookup

Human cytomegalovirus wikipedia , lookup

Neonatal infection wikipedia , lookup

West Nile fever wikipedia , lookup

Henipavirus wikipedia , lookup

Virology wikipedia , lookup

Hospital-acquired infection wikipedia , lookup

Marburg virus disease wikipedia , lookup

Hepatitis C wikipedia , lookup

Canine parvovirus wikipedia , lookup

Plasmodium falciparum wikipedia , lookup

Infection control wikipedia , lookup

Hepatitis B wikipedia , lookup

Transcript
Page 1 of 137
MICRO/PARA
1.Immunocyte of lymphocytes found in the germinal center of the lymphoid follicles is:
A. T cell
C. NK cell
B. Null cell
D. B cell
(Wheater’s Functional histology, 4th Ed., page 210)
2. What phase of the cell cycle does the cell replicates DNA?
A.GI Phase
C. Interphase
B. S phase
D. M phase
(Wheater’s Functional Histology,4th Ed., page 33)
3. Bone resorption by osteoclastic activity is:
A. TSH
C. Calcitonin
B. PTH
D. ACTH
(Wheater’s Functional Histology, 4th Ed., page 181)
4. Respiratory epithelium is seen in the following structures, EXCEPT:
A. Trachea
C. Terminal bronchiole
B. Secondary bronchus
D. Larynx
(Wheater’s Functional Histology, 4th Ed., page 230)
5. Endometrium of uterus is lined by:
A. Simple squamous
C. Simple cuboidal
B. Simple ciliated columnar
D. Stratified columnar
(Bloom and Fawcett’s Concise Histology, 2nd Ed., page 289)
6. Which of the following organs contain reticular fibers?
A. Stomach
C. Pancreas
B. Appendix
D. Lymph node
(Bloom and Fawcett’s Concise Histology, 2nd Ed., page 72)
7. Elastic connective tissue is seen in:
A. Walls of blood vessels
C. Wharton’s jelly of umbilical cord
B. Lamina propria of intestines
D. Subcutaneous tissue of skin
(Wheater’s Functional Histology, 4th Ed., page 72)
8. Destruction of these cells will lead to loss of intrinsic factor in the gastric mucosa.
A. Oxyntic cells
C. G cells
B. Mucus cells
D. Zymogenic cells
(Wheater’s Functional Histology, 4th Ed., page 255)
9. Apocrine sweat glands are distributed at:
A. Face
C. Palm
B. Trunk
D. Axilla
(Wheater’s Functional Histology, 4th Ed., page 164)
10. Lymphoid organ important for removal of particulate matters in circulation:
A. Spleen
C. Lymph nodes
B. Palatine tonsils
D. Thymus
(Wheater’s Functional Histology, 4th Ed., page 216)
11. Which of the following conducting airways of the lungs is characterized by a predominantly thick muscularis layer?
A. Primary bronchus
C. Lobar bronchus
B. Bronchiole
D. Segmental bronchus
(Wheater’s Functional Histology, 4th Ed., page 229)
12. Foreign bodies in the CNS are engulfed by:
A. Microglia
C. Astrocytes
B. Ependymal cells
D. Granule cells
(Bloom and Fawcett’s Concise Histology, 2nd Ed., page 130)
13. Protective barrier of the skin against UV radiation is:
A. Langerhans cell
C. Melanocyte
B. Merkel cell
D. Keratinocyte
(Bloom and Fawcett’s Concise Histology, 2nd Ed., page 166)
14. Part of the uterus that relaxes during time of parturition:
A. Fundus
C. Body
B. Cervix
D. Isthmus
(Bloom and Fawcett’s Concise Histology, 2nd Ed., page 289)
15. Which of the following statements regarding stomach is correct?
A. Mucosa is thrown into valve of Kerkring
B. Anatomic sphincter is found at the pyloric portion
C. Cells produce urogastrone
D. Mucosa is disrupted by submucosa
(Bloom and Fawcett’s Concise Histology, 2nd Ed., page 190)
16. Destruction of these cells will lead to low glucose level;
A. Beta cells
C. Delta cell
B. Alpha cell
D. G cell
(Wheater’s Functional Histology, 4th Ed., page 325)
MICROBIOLOGY
Page 2 of 137
17. Histologic features of appendix include:
A. Presence of taenia coli
C. Few and short crypts of Lieberkuhn
B. Presence of intestinal villi
D. Absence of submucosa
(Wheater’s Functional Histology, 4th Ed., page 271)
18. Adenohypophysis produced the following hormones, EXCEPT:
A. ACTH
C. Prolactine
B. FSH
D. Oxytocin
(Wheater’s Functional Histology, 4th Ed., page 311, 313)
19. Mucus secreting columnar cells is characteristic of:
A. Cervix
C. Vagina
B. Uterus
D. Oviduct
(Wheater’s Functional Histology, 4th Ed., page 357
20. The cells that participate in the tubuloglomerular feedback mechanism:
A. Podocytes
C. Lacis cells
B. Endothelial cells
D. Macula densa
(Wheater’s Functional Histology, 4th Ed., page 303)
21. This condition results to reduction of the total surface area available for gas exchange in which many of the alveoli beyond the
terminal bronchioles coalesce:
A. Bronchitis
C. Emphysema
B. Tuberculosis
D. Pneumonia
(Bloom and Fawcett’s Concise Histology, 2nd Ed., page 234)
22. Benign prostatic hypertrophy will show an increase in the number of:
A. Glandular acini and fibromuscular stroma
C. Prostatic vasculature
B. Corpora arenacea
D. Fibrous capsule
(Wheater’s Functional Histology, 4th Ed., page 280)
23. Hirshsprung’s disease is characterized by complete absence of:
A. Parasympathetic Auerbach’s plexus
C. Muscularis externa
B. Sympathetic Auerbach’s plexus
D. Muscularis interna
(Langman’s Medical Embryology, 9th Ed., page 316)
24. A 50 year old female develops post surgical tetany after thyroidectomy. This is directly caused by:
A. Transection of recurrent laryngeal nerve
B. Complete removal of thyroid & parathyroid glands
C. Hyperthyroid state prior to surgery
D. Hypothyroid state prior to surgery
(Wheater’s Functional Histology, 4th Ed., page 261)
25. Marked leukocytosis with regenerative shifts to the left means:
A. Absolute lymphocytosis
B. Increase leukocytosis with neutropenia
C. Absolute neutrophilic with stabosis
D. Neutrophilia with predominantly hypersegmented forms
(Bloom and Fawcett’s Concise Histology, 2nd Ed., page 46)
26. In acute inflammation of the appendix which WBC is expected to rise significantly in the peripheral blood:
A. Eosinophil
C. Lymphocyte
B. Neutrophil
D. Basophil
(Bloom and Fawcett’s Concise Histology, 2nd Ed., page 48)
27. A diabetic insipidus patient manifests excessive thirst and polyuria because of deficiency of:
A. Renin
C. Aldosterone
B. Oxytocin
D. Vasopressin
(Wheater’s Functional Histology, 4th Ed., page 257)
28. Which of the following is associated with cryptorchidism?
A. Direct inguinal hernia
C. Varicocoele
B. Hydrocoele
D. Precocious puberty
(Langman’s Medical Embryology, 9th Ed., 359)
29. Vasectomy will prevent the passage of sperm from this site:
A. Ejaculatory duct
C. Epididymis
B. Duct of seminal vesicle
D. Prostatic urethra
(Wheater’s Functional Histology, 4th Ed., page 336)
30. Overproduction of this hormone results in cushing syndrome:
A. Cortisol
C. Androgen
B. Aldosterone
D. Epinephrine
(Bloom and Fawcett’s Concise Histology, 2nd Ed., page 264)
31. Which of the following controls visceral activity and the chief effector of the limbic system?
A. Fornix
C. Hypothalamus
B. Thalamus
D. Hippocampal formation
(Basic Clinical Neuroanatomy by Paul Young, 1st Ed., page 209)
32. The only cranial nerve that is crossed dorsally from the brainstem:
A. Optic
C. Trochlear
MICROBIOLOGY
Page 3 of 137
B. Oculomotor
D. Facial
(Basic Clinical Neuroanatomy by Paul Young, 1st Ed., page 49)
33. Rhomboid fossa is formed by the floor of:
A. Lateral ventricles
C. 3rd ventricle
B. 4th ventricle
D. Cerebral aqueduct
(Duane Haines Neuroanatomy, 6th Ed., page 34)
34. The denticulate ligaments are toothlike processes arising from:
A. Pia
C. Arachnoid
B. Dura
D. Ependyma
(Basic Clinical Neuroanatomy by Paul Young, 1st Ed., page 17)
35. The corticospinal tract crosses the midline at the level of:
A. Pons
C. Midbrain
B. Spinal cord
D. Medulla
(Basic Clinical Neuroanatomy by Paul Young, 1st Ed., page 61, 65)
36. The frontal lobe performs the following function:
A. Coordinate motor activity
C. Control of motor speech
B. Control reflex activity
D. Understanding speech
(Basic Clinical Neuroanatomy by Paul Young, 1st Ed., page 193)
37. Blockage of CSF drainage in the ventricles leads to:
A. Stroke
C. Myelocoele
B. Hydrocephalus
D. Encephalocoele
(Basic Clinical Neuroanatomy by Paul Young, 1st Ed., page 257)
38. The dorsal column of the spinal cord carries the pathway of the following sensibilities, EXCEPT:
A. Vibration
C. Stereognosis
B. Position sense
D. Crude touch
(Basic Clinical Neuroanatomy by Paul Young, 1st Ed., page 132)
39. A CSF pressure of 300mm H2O is significant of:
A. Normal pressure
C. Increased intracranial pressure
B. Low pressure
D. Hydrocephalus
(Basic Clinical Neuroanatomy by Paul Young, 1st Ed., page 257)
40. The spinal meninges is differentiated from cerebral meninges by absence of:
A. Arachnoid villi
C. Epidural space
B. Pial specializations
D. Subarachnoid cisterns
(Duane Haines Neuroanatomy by Paul young, 1st Ed., page 46)
41. Lesion of the facial nerve results to:
A. Inability to open the eye
C. impaired parotid gland secretion
B. Impaired tearing
D. Weakness of lower facial muscles
(Basic Clinical Neuroanatomy by Paul Young, 1st Ed., page 52)
42. A sudden onset of right sided weakness associated with headache and vomiting in a 40 year old hypertensive suggests:
A. Intracranial hemorrhage
C. Bacterial meningitis
B. Meningioma
D. Hydrocephalus
(Basic Clinical Neuroanatomy by Paul Young, 1st Ed., page 241)
43. Patient manifests past pointing intention tremor and he lists to the right on standing. The probable location of the lesion involved is:
A. Cerebral cortex
C. Cerebellum
B.Brainstem
D. Spinal cord
(Basic Clinical Neuroanatomy by Paul Young, 1st Ed., page 107-108)
44. Ptosis and mydriasis are manifestations of injury to:
A. Sympathetic nerve
C. Abducens nerve
B. Oculomotor nerve
D. Facial nerve
(Basic Clinical Neuroanatomy by Paul Young, 1st Ed., page 49)
45. In performing a light reflex on a patient, only the left eye constricts (both direct and consensual reflexes are positive on the left eye).
What nerve is injured in this case?
A. Right optic
C. Left optic
B. Right oculomotor
D. Left oculomotor
(Basic Clinical Neuroanatomy by Paul Young, 1st Ed., page 161)
46. Impaired hearing sense on the right may be due to a lesion in:
A. Trapezoid body
C. Medial geniculate body
B. Inferior colliculus
D. Cochlear nerve
(Basic Clinical Neuroanatomy by Paul Young, 1st Ed., page 170)
47. Left homonymous hemianopsia results from injury to:
A. Optic nerve
C. Optic tract
B. Optic chiasm
D. Photoreceptors
(Basic Clinical Neuroanatomy by Paul Young, 1st Ed., page 160)
48. Brown-Sequard syndrome at T10 segment results to:
A. Weakness of the contralateral leg
B. Impaired pain on ipsilateral leg
C. Impaired position sense on contralateral leg
MICROBIOLOGY
Page 4 of 137
D. Impaired vibration sense on ipsilateral leg
(Basic Clinical Neuroanatomy by Paul Young, 1st Ed., page 141, 273)
49. Lesion of the decending brainstem pathway characterized by flexed arms and hyperextended legs:
A. Flaccidity
C. Decerebrate rigidity
B. Decorticate rigidity
D. Spasticity
(Basic Clinical Neuroanatomy by Paul Young, 1st Ed., page 77)
50. A focal lesion that affects the posterior limb of internal capsule can result from cerebrovascular accident of:
A. Anterior cerebral artery
C. Middle cerebral artery
B. Anteior communicating artery
D. Posterior cerebral artery
(Basic Clinical Neuroanatomy by Paul Young, 1st Ed., page 247)
51. The obturator internus passes through the:
A. greater sciatic foramen
C. obturator foramen
B. lesser sciatic foramen
D. obturator canal
( Clinical Anatomy for Medical Students by Richard Snell, 6th ed., page 90)
52. In the female, which of the following is found in the deep perineal pouch?
A. greater vestibular gland
C. crus of clitoris
B. vestibular bulb
D. sphincter urethrae
(Clinical Anatomy for Medical Students by Richard Snell, 6th ed., 371)
53. The lateral wall of the pelvis is covered by:
A. Pubococcygeus
C. Obturator internus
B. Puborectalis
D. Piriformis
(Clinically Oriented Anatomy by keith Moore & Arthur Dalley, 4 th ed., page 343)
54. One of the complications of hysterectomy is accidental transection of the ureter. This may occur because the ureter:
A. forms the posterior boundary of the ovarian fossa
B. crosses the bifurcation of the common iliac artery
C. is crosses by the uterine artery
D. enters the bladder wall obliquely
( Clinically Oriented Anatomy by Keith Moore & Arthur Dalley, 4 th ed., page 351)
55. A 14 y.o. boy was riding his bicycle when he stood up on the pedals and then his left foot accidentally slipped from the pedal. His
perineum hit the bar of the bicycle. A few hours later he had a perineal swelling and was unable to micturate. Diagnosis: ruptured
urethra. Which of the following is correct?
A. part of urethra commonly involved is the prostatic urethra
B. extravasated urine is the cause of swelling in the superficial perineal pouch
C. urine cannot possibly enter the anterior abdominal wall
D. urine can extend posteriorly to the ischiorectal fossa
(Clinically Oriented Anatomy by Keith Moore & Arthur Dalley, 4 th ed., page 396)
56. A 30 y.o. woman was found to have unstable right knee joint following a car accident. On examination it was possible to pull the
tibia excessively forward on the femur. Diagnosis: ruptured anterior cruciate ligament (ACL). Which of the following statements about
ACL is not correct?
A. more commonly torn than posterior cruciate ligament
B. passes upward, backward and laterally from the tibia
C. attached to the lateral surface of the medial femoral condyle
D. attached to the tibia in the anterior part of the intercondylar area
(Clinically Oriented Anatomy by Keith Moore & Arthur Dalley, 4 th ed., page 620)
57. During a fight a man was stabbed in the thigh which transected the femoral nerve just below the inguinal ligament. Which of the
following signs and symptoms will the patient have?
A. intact knee-jerk reflex
B. skin sensation is lost over the anterior and medial thigh
C. skin sensation is lost along the medial border of the big toe
D. patient cannot flex the knee joint
(Clinically Oriented Anatomy by Keith Moore & Arthur Dalley, 4th ed., page 529 & 563)
58. A 47 y.o. man was riding his motorcycle when a jaywalker suddenly crossed his path. His sudden brake and turn caused him to
lose control and he subsequently fell hitting the pavement with his head and shoulder. On examination his upper limb hung limply on
the side, medially rotated and forearm pronated. Diagnosis: Erb-Duchenne Palsy. Which statement is true regarding Erb-Duchenne
Palsy?
A. involves upper brachial plexus
B. caused by excessive abduction of the arm
C. there is no paralysis of the arm muscles
D. skin sensation of the whole arm is intact
(Clinically Oriented Anatomy by Keith Moore & Arthur Dalley, 4 th ed., page 716)
59. A data encoder complained of “pins and needles” sensation over the right palm. She also complained that she has difficulty
buttoning up her clothes. On examination, she pointed to her right thumb, index, middle and ring fingers as the areas where she felt
discomfort. Diagnosis: Carpal Tunnel Syndrome. The following are true about the median nerve , except:
A. thenar muscles are supplied by the median nerve
B. enters the palm through the carpal tunnel
C. occupies a large space between the tendons behind the flexor retinaculum
D. roots derived only from the lateral cord of the brachial plexus
MICROBIOLOGY
Page 5 of 137
(Clinically Oriented Anatomy by Keith Moore & Arthur Dalley, 4 th ed., page 774)
60. The dorsal scapular nerve innervates:
A. rhomboid major
C. splenius capitis
B. trapezius
D. supraspinatus
(Clinically Oriented Anatomy by Keith Moore & Arthur Dalley, 4 th ed., page 708)
61. Which muscle inserts to the lesser tubercle of the humerus?
A. pectoralis major
C. subscapularis
B. supraspinatus
D. teres minor
(Clinically Oriented Anatomy by Keith Moore & Arthur Dalley, 4th ed., page 691)
62. In movement of the arm, the coracobrachialis assists in:
A. lateral rotation
C. extension
B. adduction
D. supination
(Clinically Oriented Anatomy by Keith Moore & Arthur Dalley, 4 th ed., page 722)
63. Inability to extend the forearm would indicate a lesion of:
A. ulnar nerve
C. median nerve
B. musculocutaneous nerve
D. radial nerve
(Clinically Oriented Anatomy by Keith Moore & Arthur Dalley, 4 th ed., page 731)
64. Lesions of the median nerve in the hand is indicated by:
A. loss of ability to abduct the digits
B. clawhand
C. loss of opposability of the thumb
D. Dupytren’s contracture
(Clinically Oriented Anatomy by Keith Moore & Arthur Dalley, 4 th ed., page 776)
65. The lateral boundary of the cubital fossa:
A. brachiordialis
C. palmaris longus
B. pronator teres
D. interepicondylar line
(Clinically Oriented Anatomy by Keith Moore & Arthur Dalley, 4 th ed., page 731)
66. The glossopharyngeal nerve exits the skull through the:
A. foramen ovale
C. foramen rotundum
B. jugular foramen
D. internal acoustic meatus
(Clinically Oriented Anatomy by Keith Moore & Arthur Dalley, 4 th ed., page1104)
67. In a tracheostomy performed inferior to the thyroid isthmus, which vessel or nerve could not be injured:
A. external branch of superior laryngeal nerve
B. thyroidea ima artery
C. inferior thyroid vein
D. jugular arch
(Clinically Oriented Anatomy by Keith Moore & Arthur Dalley, 4 th ed., page 1049)
68. The muscular triangle of the neck:
A. is bounded superolaterally by the anterior belly of digastric
B. contains the thyroid gland
C. contains the carotid triangle
D. inferior boundary is clavicle
(Clinical Anatomy for Medical Students by Richard Snell, 6 th ed., page 645)
69. The part of the larynx located between the vestibular and vocal folds is the:
A. rima glottidis
C. laryngeal vestibule
B. laryngeal ventricle
D. infraglottis
(Clinically Oriented Anatomy by Keith Moore & Arthur Dalley, 4 th ed., page1043)
70. Damage to the facial nerve at the stylomastoid foramen would affect:
A. facial expression
C. salivation
B. taste on anterior 2/3 of tongue D. lacrimation
(Clinically Oriented Anatomy by Keith Moore & Arthur Dalley, 4 th ed., page 1098)
71. After a car accident the driver was noted to have fluid (apparently CSF) escape from the nose. The most likely cause is fracture of
the ______ bone.
A. frontal
C. nasal
B. ethmoid
D. lacrimal
(Clinical Anatomy for Medical Students by Richard Snell 6th ed., page 745)
72. The superior orbital fissure transmits all of the following, EXCEPT:
A. superior ophthalmic vein
C. optic nerve
B. oculomotor nerve
D. abducens nerve
(Clinical Anatomy for Medical Students by Richard Snell 6th ed., page 713)
73. The lacrimal gland:
A. receives sensory fibers from the optic nerve:
B. secretes directly to the lacrimal sac
C. located deep to the lateral portion of the upper eyelid
D. receives postganglionic parasympathetic fibers from the ciliary ganglion
(Clinical Anatomy for Medical Students by Richard Snell 6th ed., page 711)
74. If both lingual nerves are severed at the foramen ovale, there will be:
MICROBIOLOGY
Page 6 of 137
A. loss of taste from anterior 2/3 of tongue
B. loss of control of tongue muscles
C. loss of general sensation from anterior 2/3 of tongue
D. loss of speech
(Clinical Anatomy for Medical Students by Richard Snell 6th ed., page 684)
75. A patient with a lesion of the optic chiasm will most likely experience:
A. homonymous hemianopsia
C. bilateral anopsia
B. unilateral anopsia
D. bitemporal hemianopsia
(Clinically Oriented Anatomy by Keith Moore & Arthur Dalley, 4 th ed., page 1092)
76. The costodiaphragmatic recess extends inferiorly to the level of rib ____ at the midclavicular line, and rib ____ at the midaxillary line
A. 6, 8
C. 10, 12
B. 8, 10
D. 6, 10
(Clinically Oriented Anatomy by Keith Moore & Arthur Dalley, 4 th ed., page 98)
77. In the heart the papillary muscles are attached to the atrioventricular cusps via the:
A. dentate ligaments
C. moderator band
B. chordae tendinae
D. trabeculae carnae
(Clinically Oriented Anatomy by Keith Moore & Arthur Dalley, 4 th ed., page 127)
78. The small intestine derives its blood supply from the:
A. splenic artery
C. inferior mesenteric artery
B. left gastric artery
D. superior mesenteric artery
(Clinically Oriented Anatomy by Keith Moore & Arthur Dalley, 4 th ed., page 241 & 244)
79. The posterior boundary of the epiploic foramen of Winslow is formed by:
A. portal vein
C. caudate lobe of the liver
B. duodenum
D. inferior vena cava
(Clinically Oriented Anatomy by Keith Moore & Arthur Dalley, 4 th ed., page 217)
80. The cystic artery usually originates from the _____ artery.
A. right hepatic
C. common hepatic
B. left hepatic
D. proper hepatic
(Clinical Anatomy for Medical Students by Richard Snell, 6th ed., page 229 )
81. If the portal vein is obstructed, blood from the jejunum could return to the heart through the:
A. hepatic vein
C. splenic vein
B. esophageal vein
C. inferior mesenteric vein
(Clinically Oriented Anatomy by Keith Moore & Arthur Dalley, 4 th ed., page 277 & 278)
82. The gall bladder:
A. is located on the visceral surface of the liver between the caudate and quadrate lobes.
B. fundus lies against the anterior abdominal wall at the level of the 9th costal cartilage
C. always receives blood supply from the superior mesenteric artery
D. lies in the lesser sac
(Clinically Oriented Anatomy by Keith Moore & Arthur Dalley, 4 th ed., page 274)
83. A 60 y.o. man complains of recurring epigastric pain. A gastric consult revealed gastric ulcer. This type of ulcer is frequently
located in the pyloric part of the stomach, and consequently:
A. pain impulses from the stomach are carried by visceral afferent fibers that accompany parasympathetic nerves.
B. vagotomy is not used in the treatment of gastric ulcers.
C. posterior gastric ulcer may erode through the stomach wall into the pancreas.
D. it is easy to differentiate between gastric and duodenal ulcers by location of the pain.
(Clinically Oriented Anatomy by Keith Moore & Arthur Dalley, 4th ed., page 234)
84. If the gastroduodenal artery were occluded, the duodenum and pancreas would be provided with blood by the ______ artery.
A. superior mesenteric
C. common hepatic
B. inferior mesenteric
D. proper hepatic
(Clinically Oriented Anatomy by Keith Moore & Arthur Dalley, 4 th ed., page 241)
85. Placing your finger in the epiploic foramen of Winslow, the ventral and dorsal structures immediately palpated are:
A. duodenum and quadrate lobe of the liver
B. portal vein and inferior vena cava
C. hepatoduodenal ligament and caudate lobe of the liver
D. gastroduodenal artery and common bile duct
(Clinically Oriented Anatomy by Keith Moore & Arthur Dalley, 4 th ed., page 217)
86. The second part of the duodenum:
A. is located anterior to the superior mesenteric artery
B. lies within the peritoneal cavity
C. is the transverse segment
D. contains the greater duodenal papilla
(Clinically Oriented Anatomy by Keith Moore & Arthur Dalley, 4 th ed., page 237)
87. The cardiac vein which does not drain into the coronary sinus:
A. great cardiac vein
C. posterior cardiac vein
B. oblique vein
D. anterior cardiac vein
(Clinically Oriented Anatomy by Keith Moore & Arthur Dalley, 4th ed., page 136 & 137)
88. The septomarginal trabecula is seen in which chamber of the heart?
MICROBIOLOGY
Page 7 of 137
A. Left atrium
C. Left ventricle
B. Right atrium
D. Right ventricle
(Clinical Oriented Anatomy by Keith Moore & Arthur Dalley, 4th ed., page 127)
89. The azygos vein ultimately collects posterior intercostals venous drainage:
A. from the right side of the thorax only
B. before it empties into the inferior vena cava
C. from nearly the entire thorax
D. before it empties into the brachiocephalic vein
(Clinically Oriented Anatomy by Keith Moore & Arthur Dalley, 4 th ed., page 155)
90. Regarding the bifurcation of the trachea:
A. it is usually at about the level of T6
B. its position is affected by respiratory movements
C. the left bronchus is larger than the right
D. foreign bodies usually find their way into the left bronchus
(Clinical Anatomy for Medical Students by Richard Snell, 6th ed., page 82-83)
91. Which of the following is not true?
A. the left suprarenal gland is triangular in shape
B. the inferior mesenteric vein ascends behind the duodenal flexure
C. the tail of the pancreas crosses the left kidney
D. the left renal vein crosses in front of the aorta and receives the left testicular vein
(Clinically Oriented Anatomy by Keith Moore & Arthur Dalley, 4 th ed., page 285)
92. Anatomic narrowing of the ureter where renal calculi may be arrested:
A. before the ureter enters the bladder
B. at the renal pelvis
C. at the level of the pelvic brim
D. at the level of the iliac crest
(Clinically Oriented Anatomy by Keith Moore & Arthur Dalley, 4 th ed., page 280)
93. Most common type of intussuception:
A. ileocolic
C. ileoileal
B. colocolic
D. jejunoileal
(Clinical Anatomy for Medical Students by Richard Snell, 6 th ed., page 267)
94. Volvulus may be seen in which segment of the GIT?
A. ileum
C. descending colon
B. ascending colon
D. jejunum
(Clinically Oriented Anatomy by Keith Moore & Arthur Dalley, 4th ed., page 255)
95. The anterior rectus sheath:
A. is formed by the aponeurosis of internal oblique and transverses abdominis.
B. is formed by the aponeurosis of external and internal oblique abdominis.
C. includes scarpa’s fascia.
D. Ends at the semilunar fold of Douglas.
(Clinically Oriented Anatomy by Keith Moore & Arthur Dalley, 4 th ed., page 184)
96. Hesselbach’s triangle is a potential weak area of the abdomen where direct inguinal hernia may occur. Which of the following is not
a boundary of Hesselbach’s triangle?
A. semilunar fold of Douglas
C. inguinal ligament
B. rectus abdominis
D. inferior epigastric vessels
(Clinically Oriented Anatomy by Keith Moore & Arthur Dalley, 4 th ed., page 206)
97. Liver biopsy is a common diagnostic procedure. To avoid hitting the lungs, the needle is inserted into the:
A. 9th intercostals space, right midaxillary line
B. 9th intercostals space, right scapular line
C. 6th intercoastal space, right midclavicular line
D. 5th intercostals space, left midclavicular line
(Clinically Oriented Anatomy by Keith Moore & Arthur Dalley, 4 th ed., page 98)
98. Which of the following is a site of esophageal narrowing which may possibly offer resistance to insertion of a nasogastric tube?
A. level of the thyroid gland
B. when the arch of the aorta crosses in front of the esophagus
C. just before entering the esophageal opening of the diaphragm
D. level of C7
(Clinically Oriented Anatomy by Keith Moore & Arthur Dalley, 4 th ed., page 152)
99. Meckel’s diverticulum is a congenital anomaly of the:
A. stomach
C. jejunum
B. duodenum
D. ileum
(Clinically Oriented Anatomy by Keith Moore & Arthur Dalley, 4 th ed., page 248)
100. Cancer of the head of the pancreas often causes:
A. physiologic jaundice
C. hemorrhagic jaundice
B. obstructive jaundice
D. pathologic jaundice
(Clinically Oriented Anatomy by Keith Moore & Arthur Dalley, 4 th ed., page 262)
MICROBIOLOGY
Page 8 of 137
1.
2.
3.
4.
5.
6.
7.
8.
9.
10.
11.
12.
13.
14.
15.
16.
17.
18.
19.
20.
21.
22.
23.
24.
25.
26.
27.
28.
29.
30.
31.
32.
33.
34.
35.
36.
37.
38.
39.
40.
41.
42.
43.
44.
45.
46.
47.
48.
49.
50.
51.
52.
53.
54.
55.
56.
57.
58.
59.
60.
61.
62.
63.
64.
65.
66.
67.
68.
69.
70.
71.
72.
73.
74.
75.
76.
77.
78.
79.
A
C
A
C
B
A
D
A
D
C
A
A
D
B
B
B
C
C
B
B
C
D
A
D
C
C
A
C
A
C
B
B
C
A
D
B
A
B
B
C
B
A
B
A
C
A
A
D
A
B
D
A
B
B
D
C
D
C
A
C
C
B
D
A
C
D
C
B
C
B
D
C
D
C
D
D
C
C
B
80. D
81. A
82. D
83. B
84. D
85. D
86. B
87. A
88. D
89. C
90. D
91. D
92. B
93. A
94. C
95. B
96. B
97. D
98. C
99. D
100.B
MICROBIOLOGY
Page 9 of 137
Carlo has leukemia; unfortunately he was exposed to HBsAg positive blood. The best immediate management
which can be given to him is to give:
recombinant Hep B vaccine
passive immunization
anti-viral drugs
vitamins
Answer: B
Reference:
Medical Microbiology by Jawetz, 23rd ed. p.119
MPL:
1
Mario, a 45-year-old executive is a sickly guy. He always complains of coughs and colds. He used to be a
smoker but he had stopped since he was in his early thirty's. His doctor said that he is often attacked by
organisms which attack/ attach to the mucous membranes of his respiratory tract. These organism then do
which of the following actions against its host:
breakdown of IgA by producing proteases
evasion of phagocytosis by means of their capsule
excretion of hyaluronidase
neutralization of lysozymes
Answer: A
Reference:
Medical Microbiology by Jawetz, 23rd ed. p. 121
MPL:
0.25
A child who is breast fed is not usually prone to diarrhea, this is probably because of:
the presence of hydrolytic enzymes in the saliva which kills most bacteria
the acidity of the stomach
presence of proteolytic enzymes and active macrophages in the in the small intestines
all of the above
Answer: D
Reference:
Medical Microbiology by Jawetz, 23rd ed. p.122
MPL:
1
Luna was diagnosed to have a starting skin cancer, which somehow spontaneously resolved. Aside from a
possible miracle the doctor said it could be due to the Luna's natural killer cells. Which is NOT a
characteristic of natural killer (NK) cells)?
morphologically related to T cells
they can lyse malignant cells
they resemble large granular lymphocyte
all of the above
Answer: D
Reference:
Medical Microbiology by Jawetz, 23rd ed. p. 125
MPL:
1
Martin suffers from runny nose, itchy eyes and frequent sneezing especially during the months of March to May
and November to January. He probably has:
Type I hypersensitivity
Type II hypersensitivity
Type III hypersensitivity
Type IV hypersensitivity
Answer: A
Reference:
Medical Microbiology by Jawetz, 23rd ed. p. 140
MPL:
1
6. Vivian was rushed to the hospital for shortness of breath. She gave a history of taking penicillin for her sore
throat two days prior to admission. She gave no history of allergy to penicillin. Laboratory test done on her
blood showed the presence of antibodies which hemolyses her own RBC. She is probably suffering from:
A. Type I hypersensitivity
B. Type II hypersensitivity
C. Type III hypersensitivity
Type IV hypersensitivity
Answer: B
Reference:
Medical Microbiology by Jawetz, 23rd ed. p.141
MPL:
1
MICROBIOLOGY
Page 10 of 137
7. Athena was born December 2004. She has an older brother Bitoy who got sick of chicken pox when Athena
was one month old. Ikay, another older sister of Athena got sick of chicken pox while she did not. This is
because of:
Maternal IgA
Athena's IgA
Maternal IgG
Athena's IgG
Answer: C
Reference:
Medical Microbiology by Jawetz, 23rd ed. p. 128
MPL:
0.5
8. Isabel becomes a different person during the months of December. In these months she is grouchy because
of hay fever. Her doctor explained that she is allergic to pollen and the major culprit is the so-called IgE,
which is a homocytotropic immunoglobulin which binds to all of the following cells bellow except:
eosinophils
mast cells
basophils
neutrophils
Answer: D
Reference:
Medical Microbiology by Jawetz, 23rd ed. p 130
MPL:
0.25
9.
Betty has colds almost every three months. But she is not bothered because she noticed that even without
treatment she gets better The self-limiting nature of her colds which is often of viral etiology is partly
attributed to:
Gamma interferon
Alpha interferon
Beta interferon
All choices are correct except A
Answer: D
Reference:
Medical Microbiology by Jawetz, 23rd ed. p. 125
MPL:
0.25
10. Mike was sick of pneumonia, for which he almost died. Before his discharge from the hospital he asked the
doctor to explain to him his illness and how he was able to recover. The doctor talked of the germs having
slimly materials around them. That it was necessary for some substance to coat the germs so that the
scavenger's cells in the body can eat them up. The doctor was referring to opsonization. Which of the
following substances is involved in opsonization?
C3a
C3b
C2a
C2b
Answer: B
Reference:
Medical Microbiology by Jawetz, 23rd ed. p. 137
MPL:
0.25
11.
A 7-year-old girl was brought to the derma section of UPHR medical center for
swollen, weeping earlobes with some signs of crusting. One earlobe has an almost embedded tiny earring.
The doctor said the girl was probably allergic to the nickel component of the fancy earrings. This condition is
also termed as:
Contact hypersensitivity
Tuberculin hypersensitivity
Atopic hypersensitivity
Immune complex hypersensitivity
Answer: A
Reference:
MPL:
Medical Microbiology by Jawetz, 23rd ed. p. 141
0.25
Nonong was an OFW from Puerto Rico. He came home because he was diagnosed to have syphilis. He consulted
a local doctor for a second opinion. The doctor scrapped off the base of the ulcer found in his buccal mucosa.
MICROBIOLOGY
Page 11 of 137
He explained to Jojo that he will add to the specimen a substance which he called antibody to which will be
attached a certain dye which will emit light under a " dark microscope ".. This immunologic diagnostic test is
probably:
Immunoblotting
Immunofluorescence
ELISA
Radioimmunoassay
Answer: B
Reference:
Medical Microbiology by Jawetz, 23rd ed. p. 142
MPL:
1
Jake had typhoid fever. When seen by the doctor he has already on the third weeks and had started to feel better. If
you can enter Jake's body which of the following item below would show that he had achieved an adaptive
immune response against typhoid bacilli?
physical barriers
chemical barriers
clonal expansion of effector cells
phagocytosis
Answer: C
Reference:
Medical Microbiology by Jawetz, 23rd ed. p. 143
MPL:
1
Doy was diagnosed to have an infectious disease of unknown etiology. Examination of his blood showed that the
complement levels are normal. In infections, one defense against the agent is the activation of the complement.
What then among the item below is the best trigger for the of complement activation ?
IgG
Mannose-containing bacterial glycolipids
Microbial surfaces
IgM-antigen immune complexes
Answer: D
Reference:
Medical Microbiology by Jawetz, 23rd ed. p. 147
MPL:
0.25
A 13-year-old boy from Ilocos tested negative in the lepromin test. He presents with multiple , erythematous,
anesthetic lesions all over the body. The skin test shows which type of hypersensitivity:
Type I Hypersensitivity
Type II hypersensitivity
Type III hypersensitivity
Type IV hypersenitivity
Answer: C
Reference:
Medical Microbiology by Jawetz, 23rd ed. p. 141
MPL:
1
Roy was found to have antibodies to AIDS by the ELISA method. To confirm the diagnosis, antigens of the virus
must be detected in his blood. Which of the following methods will identify the HIV antigen mix in a complex
mixture of proteins :
Western blotting
Southern blotting
Eastern blotting
Northern blotting
Answer: A
Reference:
Medical Microbiology by Jawetz, 23rd ed. p. 143
MPL:
1
Subject: Microbiology (Virology)
Instruction: Choose the correct answer.
The human vaccine for this viral infection is made of killed virus grown in human diploid fibroblasts or chicken
fibroblasts:
Rubella
Rabies
Jap B encephalitis
MICROBIOLOGY
Page 12 of 137
Measles
Answer: B
Reference:
MPL:
Medical Microbiology by Jawetz, 23rd ed. p. 410
0.25
Luisa is an absent-minded medical intern. She was asked by the resident to get some influenza vaccines from
the refrigerator in the call room and to bring them to the OPD for immunization of the old nurses in the
section. Luisa followed the instructions. Unfortunately she did not find the resident so she placed the
vaccine on the table with a short note. After 2-4 hours, the resident came and found the vaccines.
Influenza is a killed viral vaccine, all of the following are its disadvantages EXCEPT:
high stability at room temperature
shorter duration of immunity
poor CMI produced
no IgA produced
Answer: A
Reference:
Medical Microbiology by Jawetz, 23rd ed. p. 410
MPL:
1
Allan was bitten by his dog in the lower leg. The wound is not deep but it bled. In the decision to give postexposure
prophylaxis the following are considered except which of the following :
manner of attack
nature of biting animal
severeity of the bite
none of the choices
Answer: D
Reference:
Medical Microbiology by Jawetz, 23rd ed. p. 580
MPL:
1
Laura was in her 2nd month of pregnancy when she tested positive for Rubella antibodies. Inspite of this she decided
to continue with the pregnancy. She delivered the baby 4 weeks earlier than the expected date. A t a glance the
baby seemed normal. She asked her doctor what could be the possible effect if her baby got congenital rubella.
Which of the following could explain any abnormality that may be later seen in Laura's baby ?
infected cells in the baby have reduced growth rate
hypoplastic organs results in abnormalities in the embryonic cells which have reduced growth rate
structural anomalies due to hypoplastic organs
all of the above
Answer: D
Reference:
Medical Microbiology by Jawetz, 23rd ed. p. 567
MPL:
0.25
A baby born with congenital rubella was found to be deaf. Defects in rubella are grouped into broad categories, i.e
temporary, developmental, permanent. Deafness is under :
temporary
developmental
permanent
none of the choices
Answer: C
Reference:
Medical Microbiology by Jawetz, 23rd ed. p. 568
MPL:
0.33
22.
Ana is getting married and thus was advised to get her Rubella vaccine shot. Which of the following is/are
true of the rubella vaccine ?
it is live attenuated vaccine
it is given to prevent congenital rubella infection
vaccinated children pose no threat to mothers who are susceptible and pregnant
all of the above
Answer: D
Reference:
Medical Microbiology by Jawetz, 23rd ed. p. 567
MPL:
1
MICROBIOLOGY
Page 13 of 137
23. Jay a 9-month old baby is rushed to the ER for fever and persistent cough. On examination, rales are heard
on his left chest and an infiltrate in the left lung is seen on the chest x-ray film. The diagnosis given is
pneumonia. Which of the following viruses is the most likely cause?
A. Rhinovirus
B. Respiratory syncytial virus
C. Adenovirus
D. Coronavirus
Answer: B
Reference:
Medical Microbiology by Jawetz, 23rd ed. p. 402
MPL:
1
24 In this viral infection, humans are dead-end hosts, thus virus shedding does not occur.
A. measles
B. Rabies
C. influenza
D. chicken pox
Answer: B
Reference:
Medical Microbiology by Jawetz, 23rd ed. p. 198
MPL:
1
25. A ten-year-old girl complained of difficulty and pain in swallowing. It was accompanied by runny nose and
fever. Upon examination, the intern in the OPD noted slightly swollen tonsils and erythematous pharynx.
No exudates were noted on the tonsillar surface. The diagnosis given was upper respiratory infection . The
most likely etiologic agent for this case is:
adenovirus
Respiratory syncytial virus
Rhinovirus
Coronavirus
Answer: A
Reference:
Medical Microbiology by Jawetz, 23rd ed. p. 402
MPL:
0.25
Ramon, a 4 year old boy was seen by a pediatrician for complains of low-grade
fever, cold, and rash on the
face which showed a typical " slapped cheek " appearance. Complains of flu-like symptoms such as
malaise and myalgias were also noted. The pediatrician gave a diagnosis of erythema infectiosum of the
fifth disease which is caused by which of the following viruses:
Hanta virus
B19 parvovirus
Herpes simplex
Coxsackie viruses
Answer: B
Reference:
Medical Microbiology by Jawetz, 23rd ed. p. 414
MPL:
0.25
Last summer, a community hospital in Laguna, recorded 25 cases of pharyngoconjunctivitis among the grade five
students from a private school in the nearby municipality. It was learned that all of them attended a
summer camp, which lasted for 2 weeks. Which of the following adenovirus serotypes are most likely
involved?
A. types 3 and 7
B. types 1 and 70
C. types 40 and 45
D. types 8 and 10
Answer: A
Reference:
Medical Microbiology by Jawetz, 23rd ed. p. 425
MPL:
0.25
28. A 2 ½-year-old boy was noticed by his mother to have less appetite for solid food, but when given ice cream
and soft drinks he was quite happy. This was unusual because he likes to eat and play with solid foods. The
mother examined him and found that he has some swollen nodules in his neck. Suspecting something in the
mouth, she examined the mouth and found vesicular lesions in the buccal mucosa. This is probably caused
by :
A. Candida albicans
MICROBIOLOGY
Page 14 of 137
B. Streptococcus pyogenes
Measles virus
Herpes simplex virus
Answer: D
Reference:
Medical Microbiology by Jawetz, 23rd ed. p. 435
MPL:
0.5
29.
Yoyong, a 15-year-old boy acquired chicken pox. When vesicular lesions became evident, he was "isolated"
by his mother from the rest of the family by confining him to his room. After a week , his pocks subsided
with some lesions already developing scabs. It was at this time that two other siblings started showing the
same signs and symptoms he had. Which of the following item below best describes the disease/etiologic
agent of involved in Yoyong's case?
A. the virus infects only humans
B. the disease is highly contagious
C. both are correct
neither one is correct
Answer: C
Reference:
Medical Microbiology by Jawetz, 23rd ed. p.438
MPL:
0.25
30. Idang is due two deliver in two weeks time. Unfortunately she had premature labor and deliverered a stillborn
baby boy. The baby was autopsied and the one finding was the presence of cells which look like "owls eye"
i.e. cells with large intranuclear inclusion bodies. The most probable diagnosis is:
A. Congenital measles
B. Congenital rubella
C. Congenital chicken pox
D. congenital cytomegalovirus infection
Answer: D
Reference:
Medical Microbiology by Jawetz, 23rd ed. p. 443
MPL:
0.25
31. JR a macho dancer from the red district did not report for work for almost three days. RJ his friend became
worried and went to his place. He found JR with fever, headache, and feels weak and tired. They went to
the clinic and was found to have enlarged lymph nodes and spleen and slightly icteric conjunctiva. Blood
exam showed large, atypical lymphocytes. The most probable diagnosis is:
A. Cytomegalovirus
infectious mononucleosis
B. Infectious mononucleosis
C. Herpes simplex infection
Mumps
Answer: B
Reference:
Medical Microbiology by Jawetz, 23rd ed. p. 447
MPL:
0.33
32. A 28-year-old woman has recurrent genital herpes. Which of the following about genital herpes infection is
true ?
A. cannot be transmitted in the absence of apparent lesions
reactivation of latent virus during pregnancy poses no threat to the newborn
C. can be caused by either herpes simplex type 1 or type 2
D. recurrent episodes are more severe than the primary infection
Answer: C
Reference:
Medical Microbiology by Jawetz, 23rd ed. p. 435
MPL:
0.25
33. MR consulted a dermatologist for the small, pink, wart-like lesions on her face, arms, back and buttocks. The
diagnosis given was moluscum contagiosum infection. Which of the following about this is NOT true?
A. virus has a wide range of host
B. transmission is by direct and indirect contact
C. it is considered a sexually transmitted disease
D. the virus has a DNA genome
Answer: A
Reference:
Medical Microbiology by Jawetz, 23rd ed. p. 463
MPL:
0.25
MICROBIOLOGY
Page 15 of 137
34.Motong is a leukemia patient. He was admitted for post transfusion hepatitis. Test showed the etiologic agent
to be flavivirus. This virus is:
A. Hepatitis A
B. Hepatitis B
C. Hepatitis C
Hepatitis E
Answer: C
Reference:
Medical Microbiology by Jawetz, 23rd ed. p.468
MPL:
1
35.
Ferdie was a diagnosed carrier of Hepatitis B. In one of his follow-ups in the clinic, anti-HBe antibodies were
detected in his serum. This means that:
A. he is now developing resistance
B. his HB virus titer is now low
C. it has no meaning
D. a mistake was committed because HBs Ag and anti-HBe do not exist together in the blood
Answer: B
Reference:
Medical Microbiology by Jawetz, 23rd ed. p. 468
MPL:
0.25
36. Lola, a middle-aged woman complained of acute onset fever, and nausea. Her eyes were "yellow" and she
was surprised to see her urine to be dark colored. She consulted a private clinic and the laboratory test
done on her blood was positive for HAV IgM antibody. What will you tell Lola if you are the physician?
A. That she acquired the disease through sexual contact that probably it was transmitted to her
when she had blood transfusion
to be very careful, because she can transmit the infection to other members of her family
by person-to-person spread
D. that she might develop liver carcinoma in the future
Answer: C
Reference:
Medical Microbiology by Jawetz, 23rd ed. p. 483
MPL:
1
37 A young executive from Makati was hospitalized for myopericarditis with mild congestive heart failure that
increases over several weeks. The attending physician gave a diagnosis of Coxsackie B5 infection. Other
than his conditions what other syndrome/s is/ are associated with this virus?
A. herpangina
B. aseptic meningitis
C. acute hemorrhagic conjunctivitis
D. all of the above
Answer: D
Reference:
Medical Microbiology by Jawetz, 23rd ed. p. 495
MPL:
0.25
38. After getting his first salary, Bitoy went to the seaside restaurant and feasted on seafoods, particularly raw
oyster with his favorite beer. After 24 hours, he became ill with sudden onset of vomiting, diarrhea and
headache. He consulted a private clinic and the physician, after thorough examination, ruled out a bacterial
etiology. To him the most probable viral etiologic agent is:
A. Astrovirus
B. Norwalk virus
C. Rotavirus
D. Hepatitis A virus
Answer: B
Reference:
Medical Microbiology by Jawetz, 23rd ed. p. 512
MPL:
0.25
39.
A 45-year-old woman from a ranch in Mindanao, complained of fever, malaise and sore throat, followed by
nausea, vomiting and then sudden stupor. She was diagnosed to be suffering from eastern equine
encephalitis. The control of this disease in humans could be accomplished by eradication/ control of:
A. ticks
B. birds
C. mosquitoes
MICROBIOLOGY
Page 16 of 137
D. sandflies
Answer: B
Reference:
Medical Microbiology by Jawetz, 23rd ed. p. 522
MPL:
0.25
Subject: Microbiology (Mycology)
Instruction: Choose the correct answer.
40. DJ went to Boracay last summer. He had a good tan except on some portion of his neck down to his upper
back which showed minimal scaling . After a week, the tan portion of his skin started to peel. To his
surprise the " untanned" portion of his skin seemed to have become bigger. His skin started to return to its
normal color but the " untanned" portion seemed to stand out this time looking more whitish that his normal
light brown skin. The most likely diagnosis is :
A. leprosy
B. superficial mycoses
C. avitaminosis
D. none of the choices
Answer: B
Reference:
Medical Microbiology by Jawetz, 23rd ed. p. 628
MPL:
0.25
41. DJ consulted a dermatologist for the whitish discoloration which showed after his Boracay escapade. The
doctor made a KOH mount of the skin scrapings and found, short, stout, angular hyphal elements. The
skin lesions also fluoresce under the Wood's lamp. The most likely etiologic agent is:
Tinea corporis
Pityriasis versicolor
White piedra
candidiasis
Answer: B
Reference:
Medical Microbiology by Jawetz, 23rd ed. p. 628
MPL:
1
42.
Melanie is a cook in a small restaurant responsible for the barbecue stand. Later, because of arthritis she
was reassigned to the counter as the one taking the orders. She noticed that her left hand still showed the
blackish discoloration caused by handling charcoal. She had not noticed it before because after cooking
she does not wash her hand often because of fear of " pasma". This time she used alcohol to clean the
discoloration, but to her surprised it had no effect on the " dirt." After trying to remove it to no avail for a
week, she decided to consult a doctor. The diagnosis given was tinea nigra. The etiologic agent of this is :
A. Hortaea werneckii
B. Cladosporium spp.
C. Trichophyton spp.
D. Exophiala hortai
Answer: A
Reference:
Medical Microbiology by Jawetz, 23rd ed. p. 628
MPL:
0.25
A group of " street oldies" were taken to the DSWD. They were given bath, fresh clothes and something to eat.
A medical team was also present who did some quickie check up on them. One man became the center
of attraction of the doctors and interns because of the presence of soft, yellowish nodules on the hair of his
axilla and beard. The interns thought of them as the eggs and nits of lice. The were however proven
wrong after they had squeezed some of it. The most likely diagnosis for this is :
Black piedra
Trichophyton hair infection
White piedra
Excess soap suds
Answer: C
Reference:
Medical Microbiology by Jawetz, 23rd ed. p. 628
MPL:
0.33
Vic plays basketball almost everyday after school for the last 6 weeks, until the vesicular lesions in his both feet
particularly in the toe webs became ulcerative and painful. Also, he gets embarrassed in the shower room
because his feet smell. The obvious diagnosis is athlete's foot. Three of the following dermatophytes
below are common etiologic agents of athlete's foot except :
MICROBIOLOGY
Page 17 of 137
T.mentagrophytes
T. rubrum
E. flocossum
M. canis
Answer: D
Reference:
Medical Microbiology by Jawetz, 23rd ed. pp.631-632
MPL:
0.25
45. RC, a 40-year-old farmer consulted a dermatologist for the multiple sinuses in his left arm that seem to follow
a line. According to him, it started as small wounds caused by rose's thorns, when he was pruning the
ground roses. The discharge from the sinuses showed oval budding cells that are variable in shape, but
often fusiform. The discharge was cultured in SAB. Growth in SAB yielded septate hyphae with small
conidia clustered at the tip of conidiophores, appearing like daisies with stem. The most likely identification
of this organism is:
A. Trichophyton mentagrophytes
B. Phialophora species
C. Sporothrix schenckii
D. Cladosporium carrionii
Answer: C
Reference:
Medical Microbiology by Jawetz, 23rd ed. p. 632
MPL:
0.25
46. A farmer from Laguna consulted the district hospital for his " big foot " with draining sinuses. He has been with
it for almost three years now. He was forced to see a doctor because he could no longer do his job in the
farm. The doctor noted that black granules are coming out of the sinuses . The most probable diagnosis
is:
A. sporothrichosis
B. mycetoma
C. elephantiasis
D. chromomycosis
Answer: B
Reference:
Medical Microbiology by Jawetz, 23rd ed. p.636
MPL:
0.5
47.The black granules coming out of the sinuses in the " big foot" of a farmer are hard and showed intertwined
septate hyphae. Which of the following is the most likely etiologic agent?
Madurella grisea
Actinomyces
Nocardia spp.
Trichophyton rubrum
Answer: A
Reference:
Medical Microbiology by Jawetz, 23rd ed. p. 630
MPL:
0.5
48. Jojo a Filipino migrant in California came home for a two-month vacation. He thought he can relax, but on his
second week in the Philippines he had no choice but to seek medical help for small draining sinus on his
left chest at the level of the axilla. Other symptoms were cough and fever . Examination of the discharge
using 10% KOH showed sac like structure with spore like structures inside. The most likely diagnosis is:
Histoplasmosis
Coccidioidomycosis
Blastomycosis
Tuberculosis
Answer: B
Reference:
Medical Microbiology by Jawetz, 23rd ed. p. 638
MPL:
0.33
Subject: Microbiology (Bacteriology)
Instruction: Choose the correct answer.
49.
An elderly woman with pneumonia developed bloody diarrhea on the 7th hospital day, accompanied by
abdominal cramps and fever. The doctors got worried because her fever caused by pneumonia had lysed
as early as her 3rd hospital day. She was immediately placed in the ICU and all antibiotics were stopped.
MICROBIOLOGY
Page 18 of 137
She has been on ampicillin for 5 days prior to admission and on admission was started with clindamycin and
augmentin Her condition is probably caused by:
A. Enterohemorrhagic Escherichia coli
B. Shigella dysenteriae
C. Clostridium difficile
D. Vibrio parahemolyticus
Answer: C
Reference:
Medical Microbiology by Jawetz, 23rd ed. p. 210
MPL:
0.33
Severino had an accident that resulted in the amputation of his right leg, two inches above the knee. About five
days post-op, signs of wound infection was evident. Local wound care was given and the exudate was sent to
the lab. The doctors were willing to wait for the lab results before changing antibiotics. After two days the
culture came with no organism isolated from the specimen as reported. They suspected anaerobic infection.
Anaerobic infection is usually suspected when:
A. there is presence of foul smell
B. the site of infection is near a mucosal surface
C. there is gas in tissues
D. all of the above
Answer: D
Reference:
Medical Microbiology by Jawetz, 23rd ed. p. 309
MPL:
1
51. George was rushed to the hospital for fever and pain in the right lower abdomen. Rectal exam was compatible
with the diagnosis of appendicitis. He was again rushed to the operating table because of unstable vital
signs. On opening up, he was found to have a ruptured appendicitis with abscess formation around it. The
exudates later yielded Bacteroides fragilis. Which of the following factors promote abscess formation by B.
fragilis?
A. capsule
B. lipopolysaccharide
C. pili
D. superoxide dismutase
Answer: A
Reference:
Medical Microbiology by Jawetz, 23rd ed. p. 309
MPL:
0.33
52. GC, a 20-year-old girl working in the call center, consulted a gynecologist because of whitish-gray vaginal
discharge with a bad odor of about a week duration. Previous to it, she had several sexual relationships
with her boyfriend of two months. With her condition, she felt guilty and decided to lead a clean life as soon
as the she gets well. The doctor got some fluid from the vaginal canal, placed some on the slide and some
she tested for pH which was 5.5 against the normal value of 4.5. Wet mount showed many epithelial cells
which appear granular. No PMN was noted. Which of the following is the most likely diagnosis?
A, Trichomonas vaginalis
B. Yeast vaginitis
C. Gonorrhea
D. Bacterial vaginosis
Answer: D
Reference:
Medical Microbiology by Jawetz, 23rd ed. p. 316
MPL:
1
53. A 67-year-old man was taken to the ER for difficulty of breathing. He has been feeling weak since 4 days ago.
He has a chronic cigarette cough since he was in his 40s, but did not bother to see a doctor for his
complaints until his son noticed that he looked worst. On physical examination inspiratory and expiratory
wheezes and rales were noted over the right lower lung . His chest x-ray further revealed patchy lower right
lung infiltrates. The differential diagnosis for this patient is:
A. Streptococcus pneumoniae
B. Legionella pneumophilia
C. Mycoplasma pneumoniae
D. all of the above
Answer: D
Reference:
Medical Microbiology by Jawetz, 23rd ed. p. 318
MPL:
0.25
MICROBIOLOGY
Page 19 of 137
Joe was diagnosed with lung cancer and in his bronchial washing acid-fast bacilli were seen mixed with the cancer
cells. Joe had been given the full short course of the TB regimen before and so the doctor suspected a drug
resistant strain. The bronchial washing was cultured and it grew orange colored colonies when exposed to
light. The most probable etiologic agent is:
Mycobacterium tuberculosis
Mycobacterium smegmatis
Mycobacterium kansasii
Mycobacterium avium complex
Answer: C
Reference:
Medical Microbiology by Jawetz, 23rd ed. p. 327
MPL:
0.5
You are giving lectures to TB patients about the characteristics of Mycobacterium tuberculosis. This is to convince
them that they should cover their mouths when coughing and to avoid spitting in the environment. Which of
the following characteristics should NOT be emphasized to them?
The organism is grown in complex media
The organism is resistant to drying and survive for long periods in dried sputum
The organism is resistant to acid and alkali
The organism are found in the droplet nuclei discharge by the infected person.
Answer: A
Reference:
Medical Microbiology by Jawetz, 23rd ed. p. 321
MPL:
0.25
A visiting media team conducted the " search for the child with primary complex " in an elementary school in Tanay.
After a thorough physical examination, the children in two grade one sections were skin tested using
tuberculin at a dose of 5TU. Which of the following statements on PPD is correct ?
The tuberculin skin test becomes positive within 4-6 weeks after primary infection.
Immunization with BCG will result in a positive PPD which may last for 3-7 years.
Persons who had been PPD positives before but are healthy may fail to give a positive PPD test when
tested again.
All are correct
Answer: D
Reference:
Medical Microbiology by Jawetz, 23rd ed. p. 323
MPL:
0.25
57. About fifty percent of the grade one students from Barrio Tubibi was found to have primary tuberculosis.
Which of the following features of tuberculosis is most correct?
characterized by acute exudative lesions which rapidly spreads to the lymphatics and
regional lymph nodes
all organism are killed when by the immune response that develop during primary infection
It is often accompanied by severe pulmonary edema
Caseous materials streaked with blood often are seen in expectorated sputum
Answer: A
Reference:
Medical Microbiology by Jawetz, 23rd ed. p. 322
MPL:
1
58. Paul, a 33-year-old lawyer from Ilocos was seen in the OPD with maculopapular rash over his trunk but not
in his mouth or on his palms. He is a bachelor and travels around the Philippines a lot. The attending
doctor's impression was secondary syphilis. He ordered RPR test to be done which turns out to be
reactive. Paul was advised to send his serum to a diagnostic test for TPHA or FTA-ABS test, which later
turned out to be positive. Which of the following diseases can be ruled out ?
A. atypical measles
B. Secondary syphilis
C. Coxsackie virus infection
D. German measles
Answer: B
Reference:
MPL:
Medical Microbiology by Jawetz, 23rd ed. pp.332-333
1
MICROBIOLOGY
Page 20 of 137
59. You Ming, a 14-year-old exchange student from China joined the annual boy's scout camping. They stayed in
the province for two weeks. On their second week, You Ming together with some boys went to the forest
and came back with many insect bites. You was so afraid because they even extracted a fat-bellied tick in
his right leg. At the end of the two weeks , he noticed the tick bite to be reddish but flat. It grew bigger
every day, with clearing of the central area. With the expansion of the lesion was fever, chills, myalgias and
headache. At this point his guardian brought him to the hospital. The most likely diagnosis is:
A. leptospirosis
B. lyme disease
C. Scrub typhus
D. Rat bite fever
Answer: B
Reference:
Medical Microbiology by Jawetz, 23rd ed. p. 336
MPL:
0.5
An MMDA traffic enforcer was admitted to the hospital for sudden onset of high fever (39 C), headache and pain
in his calf muscles. On admission his blood tests showed, increased PMN, abnormal liver and renal function
test . On physical examination he also had subconjunctival hemorrhage. He also had a history of wading in
the floodwaters of Tondo, about 10 days prior to the illness. The diagnosis of the attending medical intern
was leptospirosis. Which of the following would be most likely to confirm the diagnosis?
Test acute and convalescent sera using the RPR
Culture the blood on Thayer Martin agar
Test acute and convalescent serum for antileptospiral antibodies
Do darkfield examination of serum and urine for leptospires
Answer: C
Reference:
Medical Microbiology by Jawetz, 23rd ed. p. 340
MPL:
0.5
Topic: Immunology
Instruction: Choose the best answer.
1. Carlo has leukemia; unfortunately he was exposed to HBsAg positive blood. The best
management which can be given to him is to give:
A.
recombinant Hep B vaccine
B.
passive immunization
C.
anti-viral drugs
D.
vitamins
Answer: B
Reference:
Medical Microbiology by Jawetz, 23rd ed. p.119
MPL:
1
immediate
2.
Mario, a 45-year-old executive is a sickly guy. He always complains of coughs and colds. He used to be a
smoker but he had stopped since he was in his early thirty’s. His doctor said that he is often attacked by
organisms which attack/ attach to the mucous membranes of his respiratory tract. These organism then do
which of the following actions against its host:
A. breakdown of IgA by producing proteases
B. evasion of phagocytosis by means of their capsule
C. excretion of hyaluronidase
D. neutralization of lysozymes
Answer: A
Reference:
Medical Microbiology by Jawetz, 23rd ed. p. 121
MPL:
0.25
3.
A child who is breast fed is not usually prone to diarrhea, this is probably because of:
A. the presence of hydrolytic enzymes in the saliva which kills most bacteria
B. the acidity of the stomach
C. presence of proteolytic enzymes and active macrophages in the in the small intestines
D. all of the above
Answer: D
Reference:
Medical Microbiology by Jawetz, 23rd ed. p.122
MPL:
1
MICROBIOLOGY
Page 21 of 137
4.
Luna was diagnosed to have a starting skin cancer, which somehow spontaneously resolved. Aside from a
possible miracle the doctor said it could be due to the Luna’s natural killer cells. Which is NOT a
characteristic of natural killer (NK) cells)?
A. morphologically related to T cells
B. they can lyse malignant cells
C. they resemble large granular lymphocyte
D. all of the above
Answer: D
Reference:
Medical Microbiology by Jawetz, 23rd ed. p. 125
MPL:
1
5.
Martin suffers from runny nose, itchy eyes and frequent sneezing especially during the months of March to
May and November to January. He probably has:
A. Type I hypersensitivity
B. Type II hypersensitivity
C. Type III hypersensitivity
D. Type IV hypersensitivity
Answer: A
Reference:
Medical Microbiology by Jawetz, 23rd ed. p. 140
MPL:
1
6. Vivian was rushed to the hospital for shortness of breath. She gave a history of taking penicillin for her sore
throat two days prior to admission. She gave no history of allergy to penicillin. Laboratory test done on her
blood showed the presence of antibodies which hemolyses her own RBC. She is probably suffering from:
A. Type I hypersensitivity
B. Type II hypersensitivity
C. Type III hypersensitivity
D.
Type IV hypersensitivity
Answer: B
Reference:
Medical Microbiology by Jawetz, 23rd ed. p.141
MPL:
1
7. Athena was born December 2004. She has an older brother Bitoy who got sick of chicken pox when Athena
was one month old. Ikay, another older sister of Athena got sick of chicken pox while she did not. This is
because of:
A.
Maternal IgA
B.
Athena’s IgA
C.
Maternal IgG
D.
Athena’s IgG
Answer: C
Reference:
Medical Microbiology by Jawetz, 23rd ed. p. 128
MPL:
0.5
8. Isabel becomes a different person during the months of December. In these months she is grouchy because
of hay fever. Her doctor explained that she is allergic to pollen and the major culprit is the so-called IgE,
which is a homocytotropic immunoglobulin which binds to all of the following cells bellow EXCEPT:
A. eosinophils
B. mast cells
C. basophils
D. neutrophils
Answer: D
Reference:
Medical Microbiology by Jawetz, 23rd ed. p 130
MPL:
0.25
9.
Betty has colds almost every three months. But she is not bothered because she noticed that even without
treatment she gets better The self-limiting nature of her colds which is often of viral etiology is partly
attributed to:
A. Gamma interferon
B. Alpha interferon
C. Beta interferon
D. All choices are correct except A
Answer: D
MICROBIOLOGY
Page 22 of 137
Reference:
MPL:
Medical Microbiology by Jawetz, 23rd ed. p. 125
0.25
10. Mike was sick of pneumonia, for which he almost died. Before his discharge from the hospital he asked the
doctor to explain to him his illness and how he was able to recover. The doctor talked of the germs having
slimly materials around them. That it was necessary for some substance to coat the germs so that the
scavenger’s cells in the body can eat them up. The doctor was referring to opsonization. Which of the
following substances is involved in opsonization?
A. C3a
B. C3b
C. C2a
D. C2b
Answer: B
Reference:
Medical Microbiology by Jawetz, 23rd ed. p. 137
MPL:
0.25
11. A 7-year-old girl was brought to the derma section of UPHR medical center for swollen, weeping earlobes with
some signs of crusting. One earlobe has an almost embedded tiny earring. The doctor said the girl was probably allergic to the
nickel component of the fancy earrings. This condition is also termed as:
A. Contact hypersensitivity
B. Tuberculin hypersensitivity
C. Atopic hypersensitivity
D. Immune complex hypersensitivity
Answer: A
Reference:
Medical Microbiology by Jawetz, 23rd ed. p. 141
MPL:
0.25
12. Nonong was an OFW from Puerto Rico. He came home because he was diagnosed to have syphilis. He
consulted a local doctor for a second opinion. The doctor scrapped off the base of the ulcer found in his buccal
mucosa. He explained to Jojo that he will add to the specimen a substance which he called antibody to which
will be attached a certain dye which will emit light under a dark microscope “.. This immunologic diagnostic test
is probably:
A. Immunoblotting
B. Immunofluorescence
C. ELISA
D. Radioimmunoassay
Answer: B
Reference:
Medical Microbiology by Jawetz, 23rd ed. p. 142
MPL:
1
13. Jake had typhoid fever. When seen by the doctor he has already on the third weeks and had started to feel
better. If you can enter Jake’s body which of the following item below would show that he had achieved an
adaptive immune response against typhoid bacilli?
A. physical barriers
B. chemical barriers
C. clonal expansion of effector cells
D. phagocytosis
Answer: C
Reference:
Medical Microbiology by Jawetz, 23rd ed. p. 143
MPL:
1
14. Doy was diagnosed to have an infectious disease of unknown etiology. Examination of his blood showed that the
complement levels are normal. In infections, one defense against the agent is the activation of the complement.
What then among the item below is the best trigger for the of complement activation ?
A. IgG
B. Mannose-containing bacterial glycolipids
C. Microbial surfaces
D. IgM-antigen immune complexes
Answer: D
Reference:
Medical Microbiology by Jawetz, 23rd ed. p. 147
MPL:
0.25
MICROBIOLOGY
Page 23 of 137
15. A 13-year-old boy from Ilocos tested negative in the lepromin test. He presents with multiple , erythematous,
anesthetic lesions all over the body. The skin test shows which type of hypersensitivity:
A. Type I Hypersensitivity
B. Type II hypersensitivity
C. Type III hypersensitivity
D. Type IV hypersenitivity
Answer: C
Reference:
Medical Microbiology by Jawetz, 23rd ed. p. 141
MPL:
1
16. Roy was found to have antibodies to AIDS by the ELISA method. To confirm the diagnosis, antigens of the virus
must be detected in his blood. Which of the following methods will identify the HIV antigen mix in a complex
mixture of proteins :
A. Western blotting
B. Southern blotting
C. Eastern blotting
D. Northern blotting
Answer: A
Reference:
Medical Microbiology by Jawetz, 23rd ed. p. 143
MPL:
1
Subject: Microbiology (Virology)
Instruction: Choose the correct answer.
17. The human vaccine for this viral infection is made of killed virus grown in human diploid fibroblasts or chicken
fibroblasts:
A. Rubella
B. Rabies
C. Jap B encephalitis
D. Measles
Answer: B
Reference:
Medical Microbiology by Jawetz, 23rd ed. p. 410
MPL:
0.25
18. Luisa is an absent-minded medical intern. She was asked by the resident to get some influenza vaccines from the
refrigerator in the call room and to bring them to the OPD for immunization of the old nurses in the section. Luisa
followed the instructions. Unfortunately she did not find the resident so she placed the vaccine on the table with a
short note. After 2-4 hours, the resident came and found the vaccines. Influenza is a killed viral vaccine, all of
the following are its disadvantages EXCEPT:
A. high stability at room temperature
B. shorter duration of immunity
C. poor CMI produced
D. no IgA produced
Answer: A
Reference:
Medical Microbiology by Jawetz, 23rd ed. p. 410
MPL:
1
19. Allan was bitten by his dog in the lower leg. The wound is not deep but it bled. In the decision to give
postexposure prophylaxis the following are considered except which of the following :
A. manner of attack
B. nature of biting animal
C. severeity of the bite
D. none of the choices
Answer: D
Reference:
Medical Microbiology by Jawetz, 23rd ed. p. 580
MPL:
1
20. Laura was in her 2nd month of pregnancy when she tested positive for Rubella antibodies. Inspite of this she
decided to continue with the pregnancy. She delivered the baby 4 weeks earlier than the expected date. A t a
glance the baby seemed normal. She asked her doctor what could be the possible effect if her baby got
congenital rubella. Which of the following could explain any abnormality that may be later seen in Laura’s
baby ?
A. infected cells in the baby have reduced growth rate
MICROBIOLOGY
Page 24 of 137
B.
hypoplastic organs results in abnormalities in the embryonic cells which have
reduced growth rate
C. structural anomalies due to hypoplastic organs
D.
all of the above
Answer: D
Reference:
Medical Microbiology by Jawetz, 23rd ed. p. 567
MPL:
0.25
21. A baby born with congenital rubella was found to be deaf. Defects in rubella are grouped into broad
categories, i.e temporary, developmental, permanent. Deafness is under :
A. temporary
B. developmental
C. permanent
D. none of the choices
Answer: C
Reference:
MPL:
22.
Medical Microbiology by Jawetz, 23rd ed. p. 568
0.33
Ana is getting married and thus was advised to get her Rubella vaccine shot. Which of the following is/are
true of the rubella vaccine ?
A. it is live attenuated vaccine
B. it is given to prevent congenital rubella infection
C.
vaccinated children pose no threat to mothers who are susceptible and pregnant
D. all of the above
Answer: D
Reference:
MPL:
Medical Microbiology by Jawetz, 23rd ed. p. 567
1
23. Jay a 9-month old baby is rushed to the ER for fever and persistent cough. On examination, rales are heard
on his left chest and an infiltrate in the left lung is seen on the chest x-ray film. The diagnosis given is
pneumonia. Which of the following viruses is the most likely cause?
A. Rhinovirus
B. Respiratory syncytial virus
C. Adenovirus
D. Coronavirus
Answer: B
Reference:
MPL:
Medical Microbiology by Jawetz, 23rd ed. p. 402
1
24 In this viral infection, humans are dead-end hosts, thus virus shedding does
not occur. A. measles
B. Rabies
C. influenza
D. chicken pox
Answer: B
Reference:
MPL:
Medical Microbiology by Jawetz, 23rd ed. p. 198
1
25. A ten-year-old girl complained of difficulty and pain in swallowing. It was accompanied by runny nose and
fever. Upon examination, the intern in the OPD noted slightly swollen tonsils and erythematous pharynx.
No exudates were noted on the tonsillar surface. The diagnosis given was upper respiratory infection . The
most likely etiologic agent for this case is:
A. adenovirus
B. Respiratory syncytial virus
MICROBIOLOGY
Page 25 of 137
C. Rhinovirus
D. Coronavirus
Answer: A
Reference:
Medical Microbiology by Jawetz, 23rd ed. p. 402
MPL:
0.25
26. Ramon, a 4 year old boy was seen by a pediatrician for complains of low-grade
fever, cold, and rash
on the face which showed a typical “ slapped cheek ” appearance. Complains of flu-like symptoms such as
malaise and myalgias were also noted. The pediatrician gave a diagnosis of erythema infectiosum of the
fifth disease which is caused by which of the following viruses:
A. Hanta virus
B. B19 parvovirus
C. Herpes simplex
D. Coxsackie viruses
Answer: B
Reference:
Medical Microbiology by Jawetz, 23rd ed. p. 414
MPL:
0.25
27.
Last summer, a community hospital in Laguna, recorded 25 cases of pharyngoconjunctivitis among the
grade five students from a private school in the nearby municipality. It was learned that all of them attended
a summer camp, which lasted for 2 weeks. Which of the following adenovirus serotypes are most likely
involved?
A. types 3 and 7
B. types 1 and 70
C. types 40 and 45
D. types 8 and 10
Answer: A
Reference:
Medical Microbiology by Jawetz, 23rd ed. p. 425
MPL:
0.25
28. A 2 ½-year-old boy was noticed by his mother to have less appetite for solid food, but when given ice cream
and soft drinks he was quite happy. This was unusual because he likes to eat and play with solid foods. The
mother examined him and found that he has some swollen nodules in his neck. Suspecting something in the
mouth, she examined the mouth and found vesicular lesions in the buccal mucosa. This is probably caused
by :
A. Candida albicans
B. Streptococcus pyogenes
C. Measles virus
D. Herpes simplex virus
Answer: D
Reference:
Medical Microbiology by Jawetz, 23rd ed. p. 435
MPL:
0.5
29.
Yoyong, a 15-year-old boy acquired chicken pox. When vesicular lesions became evident, he was “isolated”
by his mother from the rest of the family by confining him to his room. After a week , his pocks subsided
with some lesions already developing scabs. It was at this time that two other siblings started showing the
same signs and symptoms he had. Which of the following item below best describes the disease/etiologic
agent of involved in Yoyong’s case?
A. the virus infects only humans
B. the disease is highly contagious
C. both are correct
D. neither one is correct
Answer: C
Reference:
Medical Microbiology by Jawetz, 23rd ed. p.438
MICROBIOLOGY
Page 26 of 137
MPL:
0.25
30. Idang is due two deliver in two weeks time. Unfortunately she had premature labor and deliverered a stillborn
baby boy. The baby was autopsied and the one finding was the presence of cells which look like “owls eye”
i.e. cells with large intranuclear inclusion bodies. The most probable diagnosis is:
A. Congenital measles
B. Congenital rubella
C. Congenital chicken pox
D. congenital cytomegalovirus infection
Answer: D
Reference:
Medical Microbiology by Jawetz, 23rd ed. p. 443
MPL:
0.25
31. JR a macho dancer from the red district did not report for work for almost three days. RJ his friend became
worried and went to his place. He found JR with fever, headache, and feels weak and tired. They went to
the clinic and was found to have enlarged lymph nodes and spleen and slightly icteric conjunctiva. Blood
exam showed large, atypical lymphocytes. The most probable diagnosis is:
A.
Cytomegalovirus
infectious mononucleosis
B. Infectious mononucleosis
C. Herpes simplex infection
D. Mumps
Answer: B
Reference:
MPL:
Medical Microbiology by Jawetz, 23rd ed. p. 447
0.33
32. A 28-year-old woman has recurrent genital herpes. Which of the following about genital herpes infection is
true ?
A. cannot be transmitted in the absence of apparent lesions
B. reactivation of latent virus during pregnancy poses no threat to the
newborn
C. can be caused by either herpes simplex type 1 or type 2
D. recurrent episodes are more severe than the primary infection
Answer: C
Reference:
MPL:
Medical Microbiology by Jawetz, 23rd ed. p. 435
0.25
33. MR consulted a dermatologist for the small, pink, wart-like lesions on her face, arms, back and buttocks. The
diagnosis given was moluscum contagiosum infection. Which of the following about this is NOT true?
A. virus has a wide range of host
B. transmission is by direct and indirect contact
C. it is considered a sexually transmitted disease
D. the virus has a DNA genome
Answer: A
Reference:
MPL:
Medical Microbiology by Jawetz, 23rd ed. p. 463
0.25
34.Motong is a leukemia patient. He was admitted for post transfusion hepatitis. Test showed the etiologic agent
to be flavivirus. This virus is:
A. Hepatitis A
B. Hepatitis B
C. Hepatitis C
D. Hepatitis E
Answer: C
Reference:
MPL:
Medical Microbiology by Jawetz, 23rd ed. p.468
1
MICROBIOLOGY
Page 27 of 137
35.
Ferdie was a diagnosed carrier of Hepatitis B. In one of his follow-ups in the clinic, anti-HBe antibodies were
detected in his serum. This means that:
A. he is now developing resistance
B. his HB virus titer is now low
C. it has no meaning
D. a mistake was committed because HBs Ag and anti-HBe do not exist
together in the blood
Answer: B
Reference:
MPL:
36.
Medical Microbiology by Jawetz, 23rd ed. p. 468
0.25
Lola, a middle-aged woman complained of acute onset fever, and nausea. Her eyes were “yellow” and she
was surprised to see her urine to be dark colored. She consulted a private clinic and the laboratory test
done on her blood was positive for HAV IgM antibody. What will you tell Lola if you are the physician?
A. That she acquired the disease through sexual contact
B. that probably it was transmitted to her when she had blood
transfusion
C. to be very careful, because she can transmit the infection to
other members of her family by person-to-person spread
D. that she might develop liver carcinoma in the future
Answer: C
Reference:
Medical Microbiology by Jawetz, 23rd ed. p. 483
MPL:
1
37 A young executive from Makati was hospitalized for myopericarditis with mild congestive heart failure that
increases over several weeks. The attending physician gave a diagnosis of Coxsackie B5 infection. Other
than his conditions what other syndrome/s is/ are associated with this virus?
A. herpangina
B. aseptic meningitis
C. acute hemorrhagic conjunctivitis
D. all of the above
Answer: D
Reference:
Medical Microbiology by Jawetz, 23rd ed. p. 495
MPL:
0.25
38.
After getting his first salary, Bitoy went to the seaside restaurant and feasted on seafoods, particularly raw
oyster with his favorite beer. After 24 hours, he became ill with sudden onset of vomiting, diarrhea and
headache. He consulted a private clinic and the physician, after thorough examination, ruled out a bacterial
etiology. To him the most probable viral etiologic agent is:
A. Astrovirus
B. Norwalk virus
C. Rotavirus
D. Hepatitis A virus
Answer: B
Reference:
Medical Microbiology by Jawetz, 23rd ed. p. 512
MPL:
0.25
39.
A 45-year-old woman from a ranch in Mindanao, complained of fever, malaise and sore throat, followed by
nausea, vomiting and then sudden stupor. She was diagnosed to be suffering from eastern equine
encephalitis. The control of this disease in humans could be accomplished by eradication/ control of:
A. ticks
B. birds
C. mosquitoes
D. sandflies
MICROBIOLOGY
Page 28 of 137
Answer: B
Reference:
MPL:
Medical Microbiology by Jawetz, 23rd ed. p. 522
0.25
Subject: Microbiology (Mycology)
40. DJ went to Boracay last summer. He had a good tan except on some portion of his neck down to his upper
back which showed minimal scaling . After a week, the tan portion of his skin started to peel. To his
surprise the “ untanned” portion of his skin seemed to have become bigger. His skin started to return to its
normal color but the “ untanned” portion seemed to stand out this time looking more whitish that his normal
light brown skin. The most likely diagnosis is :
A. leprosy
B. superficial mycoses
C. avitaminosis
D. none of the choices
Answer: B
Reference:
Medical Microbiology by Jawetz, 23rd ed. p. 628
MPL:
0.25
41.
DJ consulted a dermatologist for the whitish discoloration which showed after his Boracay escapade. The
doctor made a KOH mount of the skin scrapings and found, short, stout, angular hyphal elements. The
skin lesions also fluoresce under the Wood’s lamp. The most likely etiologic agent is:
A. Tinea corporis
B. Pityriasis versicolor
C. White piedra
D. candidiasis
Answer: B
Reference:
Medical Microbiology by Jawetz, 23rd ed. p. 628
MPL:
1
42.
Melanie is a cook in a small restaurant responsible for the barbecue stand. Later, because of arthritis she
was reassigned to the counter as the one taking the orders. She noticed that her left hand still showed the
blackish discoloration caused by handling charcoal. She had not noticed it before because after cooking
she does not wash her hand often because of fear of “ pasma”. This time she used alcohol to clean the
discoloration, but to her surprised it had no effect on the “ dirt.” After trying to remove it to no avail for a
week, she decided to consult a doctor. The diagnosis given was tinea nigra. The etiologic agent of this is :
A. Hortaea werneckii
B. Cladosporium spp.
C. Trichophyton spp.
D. Exophiala hortai
Answer: A
Reference:
MPL:
Medical Microbiology by Jawetz, 23rd ed. p. 628
0.25
43. A group of “ street oldies” were taken to the DSWD. They were given bath, fresh clothes and something to
eat. A medical team was also present who did some quickie check up on them. One man became the
center of attraction of the doctors and interns because of the presence of soft, yellowish nodules on the hair
of his axilla and beard. The interns thought of them as the eggs and nits of lice. The were however
proven wrong after they had squeezed some of it. The most likely diagnosis for this is :
A. Black piedra
B. Trichophyton hair infection
C. White piedra
D. Excess soap suds
Answer: C
Reference:
MPL:
44.
Medical Microbiology by Jawetz, 23rd ed. p. 628
0.33
Vic plays basketball almost everyday after school for the last 6 weeks, until the vesicular lesions in his both
feet particularly in the toe webs became ulcerative and painful. Also, he gets embarrassed in the shower
MICROBIOLOGY
Page 29 of 137
room because his feet smell. The obvious diagnosis is athlete’s foot. Three of the following dermatophytes
below are common etiologic agents of athlete’s foot EXCEPT :
A. T.mentagrophytes
B. T. rubrum
C. E. flocossum
D. M. canis
Answer: D
Reference:
MPL:
Medical Microbiology by Jawetz, 23rd ed. pp.631-632
0.25
45. RC, a 40-year-old farmer consulted a dermatologist for the multiple sinuses in his left arm that seem to follow
a line. According to him, it started as small wounds caused by rose’s thorns, when he was pruning the
ground roses. The discharge from the sinuses showed oval budding cells that are variable in shape, but
often fusiform. The discharge was cultured in SAB. Growth in SAB yielded septate hyphae with small
conidia clustered at the tip of conidiophores, appearing like daisies with stem. The most likely identification
of this organism is:
A. Trichophyton mentagrophytes
B. Phialophora species
C. Sporothrix schenckii
D. Cladosporium carrionii
Answer: C
Reference:
MPL:
Medical Microbiology by Jawetz, 23rd ed. p. 632
0.25
46. A farmer from Laguna consulted the district hospital for his “ big foot ” with draining sinuses. He has been with
it for almost three years now. He was forced to see a doctor because he could no longer do his job in the
farm. The doctor noted that black granules are coming out of the sinuses . The most probable diagnosis
is:
A. sporothrichosis
B. mycetoma
C. elephantiasis
D. chromomycosis
Answer: B
Reference:
MPL:
Medical Microbiology by Jawetz, 23rd ed. p.636
0.5
47.The black granules coming out of the sinuses in the “ big foot” of a farmer are hard and showed intertwined
septate hyphae. Which of the following is the most likely etiologic agent?
A. Madurella grisea
B. Actinomyces
C. Nocardia spp.
D. Trichophyton rubrum
Answer: A
Reference:
MPL:
Medical Microbiology by Jawetz, 23rd ed. p. 630
0.5
48. Jojo a Filipino migrant in California came home for a two-month vacation. He thought he can relax, but on his
second week in the Philippines he had no choice but to seek medical help for small draining sinus on his
left chest at the level of the axilla. Other symptoms were cough and fever . Examination of the discharge
using 10% KOH showed sac like structure with spore like structures inside. The most likely diagnosis is:
A. Histoplasmosis
MICROBIOLOGY
Page 30 of 137
B.
C.
D.
Coccidioidomycosis
Blastomycosis
Tuberculosis
Answer: B
Reference:
MPL:
Medical Microbiology by Jawetz, 23rd ed. p. 638
0.33
Subject: Microbiology (Bacteriology)
49.
An elderly woman with pneumonia developed bloody diarrhea on the 7th hospital day, accompanied by
abdominal cramps and fever. The doctors got worried because her fever caused by pneumonia had lysed
as early as her 3rd hospital day. She was immediately placed in the ICU and all antibiotics were stopped.
She has been on ampicillin for 5 days prior to admission and on admission was started with clindamycin and
augmentin Her condition is probably caused by:
A. Enterohemorrhagic Escherichia coli
B. Shigella dysenteriae
C. Clostridium difficile
D. Vibrio parahemolyticus
Answer: C
Reference:
Medical Microbiology by Jawetz, 23rd ed. p. 210
MPL:
0.33
50. Severino had an accident that resulted in the amputation of his right leg, two inches above the knee. About
five days post-op, signs of wound infection was evident. Local wound care was given and the exudate was
sent to the lab. The doctors were willing to wait for the lab results before changing antibiotics. After two days
the culture came with no organism isolated from the specimen as reported. They suspected anaerobic
infection. Anaerobic infection is usually suspected when:
A. there is presence of foul smell
B. the site of infection is near a mucosal surface
C. there is gas in tissues
D. all of the above
Answer: D
Reference:
Medical Microbiology by Jawetz, 23rd ed. p. 309
MPL:
1
51. George was rushed to the hospital for fever and pain in the right lower abdomen. Rectal exam was compatible
with the diagnosis of appendicitis. He was again rushed to the operating table because of unstable vital
signs. On opening up, he was found to have a ruptured appendicitis with abscess formation around it. The
exudates later yielded Bacteroides fragilis. Which of the following factors promote abscess formation by B.
fragilis?
A. capsule
B. lipopolysaccharide
C. pili
D. superoxide dismutase
Answer: A
Reference:
MPL:
Medical Microbiology by Jawetz, 23rd ed. p. 309
0.33
52. GC, a 20-year-old girl working in the call center, consulted a gynecologist because of whitish-gray vaginal
discharge with a bad odor of about a week duration. Previous to it, she had several sexual relationships
with her boyfriend of two months. With her condition, she felt guilty and decided to lead a clean life as soon
MICROBIOLOGY
Page 31 of 137
as the she gets well. The doctor got some fluid from the vaginal canal, placed some on the slide and some
she tested for pH which was 5.5 against the normal value of 4.5. Wet mount showed many epithelial cells
which appear granular. No PMN was noted. Which of the following is the most likely diagnosis?
A, Trichomonas vaginalis
B. Yeast vaginitis
C. Gonorrhea
D. Bacterial vaginosis
Answer: D
Reference:
MPL:
Medical Microbiology by Jawetz, 23rd ed. p. 316
1
53. A 67-year-old man was taken to the ER for difficulty of breathing. He has been feeling weak since 4 days ago.
He has a chronic cigarette cough since he was in his 40s, but did not bother to see a doctor for his
complaints until his son noticed that he looked worst. On physical examination inspiratory and expiratory
wheezes and rales were noted over the right lower lung . His chest x-ray further revealed patchy lower right
lung infiltrates. The differential diagnosis for this patient is:
A. Streptococcus pneumoniae
B. Legionella pneumophilia
C. Mycoplasma pneumoniae
D. all of the above
Answer: D
Reference:
MPL:
Medical Microbiology by Jawetz, 23rd ed. p. 318
0.25
54. Joe was diagnosed with lung cancer and in his bronchial washing acid-fast bacilli were seen mixed with the
cancer cells. Joe had been given the full short course of the TB regimen before and so the doctor suspected
a drug resistant strain. The bronchial washing was cultured and it grew orange colored colonies when
exposed to light. The most probable etiologic agent is:
A. Mycobacterium tuberculosis
B. Mycobacterium smegmatis
C. Mycobacterium kansasii
D. Mycobacterium avium complex
Answer: C
Reference:
MPL:
55.
You are giving lectures to TB patients about the characteristics of Mycobacterium tuberculosis. This is to
convince them that they should cover their mouths when coughing and to avoid spitting in the environment.
Which of the following characteristics should NOT be emphasized to them?
A. The organism is grown in complex media
B. The organism is resistant to drying and survive for long periods in dried sputum
C. The organism is resistant to acid and alkali
D. The organism are found in the droplet nuclei discharge by the infected person.
Answer: A
Reference:
MPL:
56.
Medical Microbiology by Jawetz, 23rd ed. p. 327
0.5
Medical Microbiology by Jawetz, 23rd ed. p. 321
0.25
A visiting media team conducted the “ search for the child with primary complex ” in an elementary school in
Tanay. After a thorough physical examination, the children in two grade one sections were skin tested using
tuberculin at a dose of 5TU. Which of the following statements on PPD is correct ?
A. The tuberculin skin test becomes positive within 4-6 weeks after primary infection.
B. Immunization with BCG will result in a positive PPD which may last for 3-7 years.
MICROBIOLOGY
Page 32 of 137
C.
D.
Answer: D
Reference:
MPL:
57.
Persons who had been PPD positives before but are healthy may fail to give a positive
PPD test when tested again.
All are correct
Medical Microbiology by Jawetz, 23rd ed. p. 323
0.25
About fifty percent of the grade one students from Barrio Tubibi was found to have primary tuberculosis.
Which of the following features of tuberculosis is most correct?
A. characterized by acute exudative lesions which rapidly spreads to the lymphatics and regional
lymph nodes
B.
all organism are killed when by the immune response that develop during primary infection
C. It is often accompanied by severe pulmonary edema
D. Caseous materials streaked with blood often are seen in expectorated sputum
Answer: A
Reference:
MPL:
Medical Microbiology by Jawetz, 23rd ed. p. 322
1
58. Paul, a 33-year-old lawyer from Ilocos was seen in the OPD with maculopapular rash over his trunk but not
in his mouth or on his palms. He is a bachelor and travels around the Philippines a lot. The attending
doctor’s impression was secondary syphilis. He ordered RPR test to be done which turns out to be
reactive. Paul was advised to send his serum to a diagnostic test for TPHA or FTA-ABS test, which later
turned out to be positive. Which of the following diseases can be ruled out ?
A. atypical measles
B. Secondary syphilis
C. Coxsackie virus infection
D. German measles
Answer: B
Reference:
MPL:
Medical Microbiology by Jawetz, 23rd ed. pp.332-333
1
59. You Ming, a 14-year-old exchange student from China joined the annual boy’s scout camping. They stayed in
the province for two weeks. On their second week, You Ming together with some boys went to the forest
and came back with many insect bites. You were so afraid because they even extracted a fat-bellied tick in
his right leg. At the end of the two weeks, he noticed the tick bite to be reddish but flat. It grew bigger
every day, with clearing of the central area. With the expansion of the lesion was fever, chills, myalgias and
headache. At this point his guardian brought him to the hospital. The most likely diagnosis is:
A. leptospirosis
B. lyme disease
C. Scrub typhus
D. Rat bite fever
Answer: B
Reference:
MPL:
Medical Microbiology by Jawetz, 23rd ed. p. 336
0.5
60. An MMDA traffic enforcer was admitted to the hospital for sudden onset of high fever (39 C), headache and
pain in his calf muscles. On admission his blood tests showed, increased PMN, abnormal liver and renal
function test. On physical examination he also had subconjunctival hemorrhage. He also had a history of
wading in the floodwaters of Tondo, about 10 days prior to the illness. The diagnosis of the attending medical
intern was leptospirosis. Which of the following would be most likely to confirm the diagnosis?
A. Test acute and convalescent sera using the RPR
MICROBIOLOGY
Page 33 of 137
B.
C.
D.
Culture the blood on Thayer Martin agar
Test acute and convalescent serum for antileptospiral antibodies
Do darkfield examination of serum and urine for leptospires
Answer: C
Reference:
MPL:
Medical Microbiology by Jawetz, 23rd ed. p. 340
0.5
Subject: Parasitology (40)
MPL = 23.25
Choose the correct answer.
61. A 31-year old female from Southern Mindanao is brought in for chronic diarrhea of two months duration.
Stool examination reveals larvae with prominent genital primodium. What is your probable diagnosis?
a. Capillariasis
b. Hookworm infection
c. Strongyloidiasis
d. Ascariasis
Answer: C
Ref. Clinical Parasitology by Beaver and Jung 9th edition, pp. 260-261
MPL: 0.5
62. A 10-year-old girl, with perianal pruritus, was brought by her mother to her pediatrician. What is your most
probable diagnosis?
a.
Ascariasis
b.
Trichuriasis
c.
Enterobiasis
d.
Hookworm infection
Ans: C
Ref. Clinical Parasitology by Beaver and Jung 9th edition, pp. 304-305
MPL: 0.75
63. A 10-year old girl with perianal pruritus was brought by her mother to her pediatrician. Based from your
probable diagnosis, what diagnostic procedure will you request to support your diagnosis?
a. Stool concentration technique
b. Direct fecal smear
c. Harada-mori stool culture
d. Cellulose tape swab
Ans: D
Ref. Clinical Parasitology by Beaver and Jung 9th edition, pp. 304-306
MPL: 0.75
64. A 15-year old boy from Davao del Norte is noted to have pallor and malnutrition. Stool examination reveals
an ovum with thin colorless cell wall. What is your diagnosis?
a. Ascariasis
b. Trichuriasis
c. Enterobiasis
d. Hookworm infection
Ans: D
Ref. Clinical Parasitology by Beaver and Jung 9th edition, p.285
MPL: 0.5
65. A 15-year old boy from Davao del Norte is noted to have pallor and malnutrition. Stool examination reveals
an ovum with thin colorless cell wall. Probable diagnosis is hookworm infection. Name the portal of entry of
this worm.
MICROBIOLOGY
Page 34 of 137
a.
b.
c.
d.
oral ingestion
inhalation
skin penetration
mosquito bite
Ans: C
Ref. Clinical Parasitology by Beaver and Jung 9th edition, pp. 280-285
MPL: 0.5
66. Ova with flat bi-polar plugs were seen in the stool specimen of a 16-year old female with chronic diarrhea
from Compostela Valley province. What is your diagnosis?
a. Capillariasis
b. Hookworm infection
c. Strongyloidiasis
d. Ascariasis
Ans: A
Ref. Clinical Parasitology by Beaver and Jung 9th edition, p. 246
MPL: 0.5
67. Ova with flat bi-polar plugs were seen in the stool specimen of a 16-year old female with chronic diarrhea
from Compostela Valley province. If your diagnosis is correct, how did this patient acquire the infection?
a. Eating of salad
b. Eating raw infected snail
c. Eating raw infected fresh water fish
d. Eating raw or improperly cooked meat
Ans: C
Ref. Clinical Parasitology by Beaver and Jung 9th edition, pp. 246-247
MPL: 0.5
68. A 24-year old abaca farmer from Sorsogon with fever associated with signs and symptoms of inflammation
of the lymph glands was brought to the Out Patient Department of the Philippine General Hospital. What
parasitic diagnostic procedure will you request in the laboratory?
a. Stool examination
b. Sputum examination
c. Thick blood film
d. Urine examination
Ans: C
Ref. Clinical Parasitology by Beaver and Jung 9th edition, pp.752-753
MPL: 0.75
69. A 24-year old abaca farmer from Sorsogon with fever associated with signs and symptoms of inflammation
of the lymph glands was brought to the Out Patient Department of the Philippine General Hospital.
Microscopic finding in the blood taken at 10:00 pm revealed presence of larvae. What is your diagnosis?
a. Malaria
b. Toxoplasmosis
c. Filariasis
d. Angiostrongiloidiasis
Ans: C
Ref. Clinical Parasitology by Beaver and Jung 9th edition, pp.749-750
MPL: 0.25
70. A 24-year old abaca farmer from Sorsogon with fever associated with signs and symptoms of inflammation
of the lymph glands was brought to the Out Patient Department of the Philippine General Hospital.
MICROBIOLOGY
Page 35 of 137
Microscopic finding in the blood taken at 10:00 pm revealed presence of microfilariae. What drug can you
prescribe to this patient?
a. Mebendazole
b. Metronidazole
c. Di-ethyl carbamazine
d. Praziquantel
Ans: C
Ref. Clinical Parasitology by Beaver and Jung 9th edition, p.362
MPL: 0.75
71. A 40-year old male overseas worker complaining of muscle pains seek medical attention upon his arrival to
the Philippines. Biochemical tests showed elevated creatinine, phosphokinase, lactate dehydrogenase and
myokinase levels. Results of complete blood count showed high blood eosinophilia. What is your most
probable diagnosis?
a. Taeniasis
b. Capillariasis
c. Trichinosis
d. Filariasis
Ans: C
Ref. Clinical Parasitology by Beaver and Jung 9th edition, pp.238-239
MPL: 0.5
72. A 40-year old male overseas worker complaining of muscle pains seek medical attention upon his arrival to
the Philippines. Biochemical tests showed elevated creatinine, phosphokinase, lactate dehyrogenase and
myokinase levels. Results of complete blood count showed high blood eosinophilia. What other laboratory
test would you recommend to confirm your probable diagnosis?
a. Bentonite flocculation test
b. Montenegro test
c. Sabin Feldman test
d. Knott’s blood concentration test
Ans: A
Ref. Clinical Parasitology by Beaver and Jung 9th edition, p. 768
MPL: 0.25
73. A 40-year old male overseas worker complaining of muscle pains seek medical attention upon his arrival to
the Philippines. Biochemical tests showed elevated creatinine, phosphokinase, lactate dehyrogenase and
myokinase levels. Results of complete blood count showed high blood eosinophilia. Serological test may
confirm your diagnosis, but since there is no available serological test, what would you request to confirm
your diagnosis?
a. Skin test
b. Muscle biopsy
c. Rectal biopsy
d. Skin scrapping
Ans: B
Ref. Clinical Parasitology by Beaver and Jung 9th edition, p.754
MPL: 0.5
74. A flat whitish worm, measuring 1.5 c.m. long was submitted to the Diagnostic Parasitology Laboratory for
identification. It was found in the underwear of a 7-year old girl complaining of abdominal discomfort and
occasional itchiness of the perianal area. What parasitic infection would you consider?
a. Enterobiasis
b. Taeniasis
c. Strongyloidiasis
d. Heterophydiasis
MICROBIOLOGY
Page 36 of 137
Ans: B
Ref. Clinical Parasitology by Beaver and Jung 9th edition, p.521
MPL: 0.5
75. A flat whitish worm measuring 1.5cm long was submitted to the Diagnostic Parasitology Laboratory for
identification. It was found in the underwear of a 7-year old girl complaining of abdominal discomfort and
occasional itchiness of the perianal area. Laboratory results identified the worm as segment of a Taenia
saginata. How did this patient acquire the infection?
a. Eating of infected pork salad
b. Eating raw infected snail
c. Eating raw infected fresh water fish
d. Eating raw infected beef
Ans: D
Ref. Clinical Parasitology by Beaver and Jung 9th edition, p.521
MPL: 0.75
76. A flat whitish worm measuring 1.5 cm long was submitted to the Diagnostic Parasitology Laboratory for
identification. It was found in the underwear of a 7-year old girl complaining of abdominal discomfort and
occasional itchiness of the perianal area. Laboratory results identified the worm as segment of a Taenia
saginata. What is the drug of choice for this parasitic infection?
a. mebendazole
b. albendazole
c. praziquantel
d. pipperazine
Ans: C
Ref. Clinical Parasitology by Beaver and Jung 9th edition, p.522
MPL: 0.5
77. Adult roundworm, measuring 27 cm. in length, was seen in the colon of a 10-year old boy who died of
pneumonia. Manifestations such as lung infiltration, asthmatic attacks, and edema of the lips were
documented before the patient died. What is the most probable parasitic infection can you identify in this
case?
a. Trichuriasis.
b. Ascariasis
c. Capillariasis
d. Hookworm infection
Ans: B
Ref. Clinical Parasitology by Beaver and Jung 9th edition, pp. 226-228
MPL: 0.75
78. A fisherman from Davao Oriental was rushed to a nearby hospital with chest pain, persistent cough, and
hemoptysis as his main complains. Patient interview revealed excessive drinking of native alcohol together
with raw mountain crabs as their “pulutan”. What is the Diagnostic work-up you should include for this
patient?
a. chest x-ray
b. sputum examination
c. stool examination
MICROBIOLOGY
Page 37 of 137
d.
all of the above
Ans: B
Ref. Clinical Parasitology by Beaver and Jung 9th edition, pp.466-468
MPL: 0.75
79. A fisherman from Davao Oriental was rushed to a nearby hospital with chest pain, persistent cough, and
hemoptysis as his main complains. Patient interview revealed excessive drinking of native alcohol together
with raw mountain crabs as their “pulutan”. Chest x-ray demonstrated patchy, cloudy infiltration of the lungs
with nodular shadows and calcified spots. What parasitic infection can you consider?
a. Pulmonary tuberculosis
b. Paragonimiasis
c. Ascariasis
d. Strongyloidiasis
Ans: B
Ref. Clinical Parasitology by Beaver and Jung 9th edition, pp. 464-468
MPL: 0.75
80. A fisherman from Davao Oriental was rushed to a nearby hospital with chest pain, persistent cough, and
hemoptysis as his main complains. Patient interview revealed excessive drinking of native alcohol together
with raw mountain crabs as their “pulutan”. Chest x-ray demonstrated patchy, cloudy infiltration of the lungs
with nodular shadows and calcified spots. Paragonimiasis is the parasitic infection you are considering?
What social factor strongly contributes to your diagnosis?
a. drinking habits
b. occupation
c. eating habits
d. gender
Ans: C
Ref. Clinical Parasitology by Beaver and Jung 9th edition, p.468
MPL: 0.75
81. A 51-year old farmer from Surigao, Mindanao was brought to a hospital with abdominal pain and frequent
loose stools. Further examination revealed hepatomegaly with tenderness in the upper right quadrant of the
abdomen. Laboratory results showed increase of eosinophilia. Direct fecal smear showed no ova or parasite
seen, while rectal biopsy demonstrated eggs with miracidia. Based from clinical and laboratory results, what
parasitic disease can you suggest?
a. schistosomiasis
b. paragonimiasis
c. leishmaniasis
d. heterophydiasis
Ans: A
Ref. Clinical Parasitology by Beaver and Jung 9th edition, pp.416-425
MPL: 0.5
82. A 51-year old farmer from Surigao, Mindanao was brought to a hospital with abdominal pain and frequent
loose stools. Further examination revealed hepatomegaly with tenderness in the upper right quadrant of the
MICROBIOLOGY
Page 38 of 137
abdomen. Laboratory results showed increase of eosinophilia. Direct fecal smear showed no ova or parasite
seen, while rectal biopsy demonstrated eggs with miracidia. What factor should you consider to make your
specific diagnosis?
a. clinical signs and symptoms
b. recovery of characteristic eggs
c. increase of eosinophilia
d. evidence of exposure to endemic area
Ans: B
Ref. Clinical Parasitology by Beaver and Jung 9th edition, pp. 416-440
MPL: 0.75
83. A 51- year old farmer from Surigao, Mindanao was brought to a hospital with abdominal pain and frequent
loose stools. Further examination revealed hepatomegaly with tenderness in the upper right quadrant of the
abdomen. Laboratory results showed increase of eosinophilia. Direct fecal smear showed no ova or parasite
seen, while rectal biopsy demonstrated eggs with miracidia. What is/are the possible reason for the false
negative result of stool examination?
a. early infection
b. all male infection
c. all female infection
d. all of the above
Ans: D
Ref. Clinical Parasitology by Beaver and Jung 9th edition, pp. 416-425
MPL: 0.75
84. A 51-year old farmer from Surigao, Mindanao was brought to a hospital with abdominal pain and frequent
loose stools. Further examination revealed hepatomegaly with tenderness in the upper right quadrant of the
abdomen. Laboratory results showed increase of eosinophilia. Direct fecal smear showed no ova or parasite
seen, while rectal biopsy demonstrated eggs with miracidia, which lead you to diagnose schistosomiasis.
This parasitic infection is also known as:
a. Bilharziasis
b. Katayama fever
c. Snail fever
d. All of the above
Ans: D
Ref. Clinical Parasitology by Beaver and Jung 9th edition, pp.434-440
MPL: 0.75
85. A 62-year old farmer from Sorsogon was diagnosed for schistosomiasis japonicum. Its main pathology and
disease manifestation are due to host granulomatous reaction. The underlying mechanism for its granuloma
formation is due to:
a. acute inflammatory reaction
b. chronic vascular obstruction
c. immediate allergic reaction
d. delayed hypersensitivity
Ans: D
Ref. Clinical Parasitology by Beaver and Jung 9th edition, pp.416-425
MPL: 0.75
86. A 51-year old farmer from Surigao, Mindanao was brought to a hospital with abdominal pain and frequent
loose stools. Further examination revealed hepatomegaly with tenderness in the upper right quadrant of the
abdomen. Laboratory results showed increase of eosinophilia. Direct fecal smear showed no ova or parasite
MICROBIOLOGY
Page 39 of 137
seen, while rectal biopsy demonstrated eggs with miracidia, which lead you to diagnose schistosomiasis.
What is the drug of choice for this parasitic infection?
a. mebendazole
b. albendazole
c. praziquantel
d. pipperazine
Ans: C
Ref. Clinical Parasitology by Beaver and Jung 9th edition, p.424
MPL: 0.75
87. A 51-year old farmer from Surigao, Mindanao was brought to a hospital with abdominal pain and frequent
loose stools. Further examination revealed hepatomegaly with tenderness in the upper right quadrant of the
abdomen. Laboratory results showed increase of eosinophilia. Direct fecal smear showed no ova or parasite
seen, while rectal biopsy demonstrated eggs with miracidia, which lead you to diagnose schistosomiasis.
Serious complications of this parasitic disease include:
a. renal failure
b. intestinal lesions
c. bone metastases
d. cerebral involvement
Ans: D
Ref. Clinical Parasitology by Beaver and Jung 9th edition, pp. 421-423
MPL: 0.5
88. Sexually active 26-year old woman consulted a VD control clinic for vaginal itching and purulent discharge.
Your tentative diagnosis should include the following:
a. gonorrhea
b. trichomoniasis
c. candidiasis
d. all of the above
Ans: D
Ref. Clinical Parasitology by Beaver and Jung 9th edition, pp. 50
MPL: 0.25
89. Sexually active 26-year old woman consulted a VD control clinic for vaginal itching and purulent discharge.
Trichomoniasis is your tentative diagnosis. What parasitologic examination should you include in your workup?
a. stool culture
b. wet mount of vaginal fluid
c. pH test of blood sample
d. ova and parasite fecal smear
Ans: B
Ref. Clinical Parasitology by Beaver and Jung 9th edition, p.51
MPL: 0.5
90. Sexually active 26-year old woman consulted a VD control clinic for vaginal itching and purulent discharge.
Trichomoniasis is your tentative diagnosis, which is best treated with:
a. albendazole
b. mebendazole
c. metronidazole
d. praziquantel
MICROBIOLOGY
Page 40 of 137
Ans: C
Ref. Clinical Parasitology by Beaver and Jung 9th edition, p. 51
MPL: 0.75
91. Sexually active 26-year old woman consulted a VD control clinic for vaginal itching and purulent discharge.
Trichomoniasis is your tentative diagnosis, which is contracted almost exclusively through sexual
intercourse. Control and prevention require
a. prompt treatment of the patient
b. prompt treatment of sexual partner
c. treatment of both
d. require no treatment, since it is self-limiting
Ans: C
Ref. Clinical Parasitology by Beaver and Jung 9th edition, p. 51
MPL: 0.75
92. A 30-year old woman, native of Bulacan, traveled to Palawan for a vacation. After 2 weeks she consulted her
doctor with chills and fever as her main complains. What parasitologic test would you request to establish
your diagnosis?
a. sputum examination
b. stool examination
c. thick blood smear
d. thick and thin blood smear
Ans: D
Ref. Clinical Parasitology by Beaver and Jung 9th edition, pp. 180-181
MPL: 0.5
93. A 30-year old woman, native of Bulacan, traveled to Palawan for a vacation. After 2 weeks she consulted her
doctor with chills and fever as her main complaints. Microscopic examination of thick and thin blood smear
showed infected RBC with double chromatin dot. What is your most probable diagnosis?
a. Not malaria
b. Plasmodium falciparum
c. Plasmodium vivax
d. Plasmodium malariae
Ans: B
Ref. Clinical Parasitology by Beaver and Jung 9th edition, pp.185-188
MPL: 0.75
94. A 6-month old baby was brought in for a fever of one-day duration. She is found to have a temperature of
38.5°C and with hepatomegaly (3 cm) and splenomegaly (3 cm). Microscopic examination revealed infected
RBC with presence of band form stages. What is your most probable diagnosis?
a. Not malaria
b. Plasmodium falciparum
c. Plasmodium vivax
d. Plasmodium malariae
Ans: D
Ref. Clinical Parasitology by Beaver and Jung 9th edition, pp. 184-185
MPL: 0.75
95. A 49-year-old man from Manila received 4 units of packed red blood cells (PRBCs) on January 15 while
undergoing hip replacement surgery. He was again hospitalized on February 1 with fever, hypotension, and
renal failure. Peripheral blood smears confirmed malarial infection. The patient has never traveled outside
Manila for 20 years. How did this patient acquire the infection?
a. bite of mosquito
b. blood transfusion
c. relapse
MICROBIOLOGY
Page 41 of 137
d. infected syringe
Ans: B
Ref. Clinical Parasitology by Beaver and Jung 9th edition, pp.174-180
MPL: 0.75
96. A company plans to send a large number of employees to work in Afghanistan, for variable periods of time.
Medical examination by their employees’ health service accidentally found this banana shape organism
while doing routine CBC. Will there be a malaria risk in Afghanistan?
a. No risk
b. Risk in all areas
c. Risk in areas where mosquito vectors are present
d. Risk in all areas, during rainy season only
Ans: C
Ref. Clinical Parasitology by Beaver and Jung 9th edition, pp. 188-193
MPL: 0.75
97. A 6-month old baby was brought in for a fever of one-day duration. She is found to have a temperature of
38.5°C and with hepatomegaly (3 cm) and splenomegaly (3 cm). Microscopic examination revealed infected
RBC with presence of ring form stage only. What is your most probable diagnosis?
a. Not malaria
b. Plasmodium falciparum
c. Plasmodium vivax
d. Plasmodium ovale
Ans:
Reference:
MPL: 0.75
B
Clinical Parasitology by Beaver and Jung 9th edition, pp. 185-188
98. A 40-year old man, resident of the Manila, presents to an emergency room with a 5-day history of fever,
chills, nausea, vomiting, and myalgias. He returned 2 weeks ago from a 2-year job contract from Zambia.
Microscopic examination revealed oval shaped infected RBC. What is your most probable diagnosis?
a. Plasmodium falciparum
b. Plasmodium vivax
c. Plasmodium ovale
d. Plasmodium malariae
Ans:
Reference:
MPL: 0.75
C
Clinical Parasitology by Beaver and Jung 9th edition, pp. 182-184
99. An overseas worker with febrile illness that began in December 2003, approximately 3 months and 14
months, respectively, after leaving Africa, consulted the hospital. During the course of his illness, the patient
experienced fluctuating temperatures and lost 13 pounds of body weight. Light-microscopic examinations or
cultures of bone marrow and liver-biopsy specimens noted group of intracellular small rounded organisms.
Laboratory results identify it as amastigote stage. What is the most likely parasitic infection present in this
patient?
a. Malaria
b. Leishmaniasis
c. Toxoplasmosis
d. Trypanosomiasis
Ans:
Reference:
MPL: 0.5
B
Clinical Parasitology by Beaver and Jung 9th edition, pp.71-74
100. An overseas worker coming home from Uganda was rushed to a nearby hospital for seizure attack. His
relatives informed the medical officer on duty that the patient is experiencing progressive confusion and
MICROBIOLOGY
Page 42 of 137
personality changes. Laboratory examination of the CSF revealed presence of polymorphic with undulating
membrane. What is the most likely parasitic infection present in this patient?
a. Malaria
b. Leishmaniasis
c. Toxoplasmosis
d. Trypanosomiasis
Ans:
Reference:
MPL: 0.5
D
Clinical Parasitology by Beaver and Jung 9th edition, pp. 77-87
1.
Which is true about the bacterial cell?
A. Prokaryotic, unicellular
B. Gram positive bacteria have outer membrane
C. Gram negative bacteria have thicker cell wall than gram positive bacteria
D. Eukaryotic, muliticellular
Answer: A
Source: Jawetz 23rd edition; pages 11-31
MPL: 0.33
2.
Which structure is found in gram-negative bacteria but not in gram positive bacteria?
A. outer membrane
C. nucleoid
B. peptidoglycan
D. plasma membrane
Answer: A
Source: Jawetz 23rd edition; page 17
MPL: 0.50
3.
True about transfer of genes from one bacterium to another.
A. Gram negative bacteria that have sex pill transfer plasmids by conjugation
B. Free DNA fragments are transferred by transformation
C. Bacteriophages are involved in transduction
D. Involves participation of flagella
Answer: A
Source: Jawetz 23rd edition; pages 101-103
MPL: 0.33
4.
Which of the following bacteria is gram negative
A. Staphylococcus
B. Vibrio
Answer: B
Source: Jawetz 23rd edition; pages 45-46
MPL: 0.50
C. Corynebacterium
D. Lactobacillus
5.
True about obligate anaerobes
A. Use sugars by oxidative pathway
B. Infections are limited to muscle tissues
C. Lack superoxide dismutase
D. Can use oxygen in small amounts
Answer: C
Source: Jawetz 23rd edition; page 66
MPL: 0.50
6.
Which structure is associated with virulence of a bacterium
A. Cell wall
C. Flagella
B. Lipopolysaccharide
D. Endospores
Answer: B
Source: Jawetz 23rd edition; pages 31-32
MPL: 0.50
7.
This mechanism of gene transfer is characterized by the requirement for cell-to-cell contact
A. Conjugation
C. Transformation
B. Transduction
D. Transfection
MICROBIOLOGY
Page 43 of 137
Answer: A
Source: Jawetz 23rd edition; page 103
MPL: 0.25
8.
Action of lysozyme
A. Involved in the formation of peptide bond between two peptidoglycan subunits
B. Hydrolyzes peptidoglycan by cleaving the glycosyl bonds between N-acetylmuramic acid and Nacetyl glucosamine
C. An enxyme that dissolves the peptidoglycan layer
D. Located in the periplasmic space and is involved in the hydrolysis of foreign DNA
Answer: B
Source: Jawetz 23rd edition; page 121
MPL: 0.33
9.
True about gram negative bacteria
A. Have simpler but thicker cell wall than gram positive bacteria
B. Has an outer membrane located outside the peptidoglycan layer
C. Contains teichoic acid polymers dispersed throughout the peptidoglycan
D. Removal of the cell wall results in the formation of protoplasis
Answer: B
Source: Jawetz 23rd edition; page 17
MPL: 0.50
10. This type of antibiotic should be used with caution because of renal and ototoxicity.
A. Penicillins
C. Aminoglycosides
B. Macrolides
D. Tetracycyclines
Answer: C
Source: Jawetz 23rd edition; page 188
MPL: 0.50
11. Lack/s the ability to develop catalase and superoxide dismutase:
A. Obligate aerobes
C. Facultative anaerobes
B. Obligate anaerobes
D. Aerotolerant anaerobes
Answer: B
Source: Jawetz 23rd edition; page 66
MPL: 1.00
12. Which bacterial structures is not essential for growth but may confer survival advantage under certain
conditions?
A. Cell wall
C. Endospore
B. Outer membrane
D. Flagella
Answer: D
Source: Jawetz 23rd edition; page 35
MPL: 1.00
13. Serves as an anchor to bind and pull apart daughter chromosomes during cell division
A. Pill
C. Flagella
B. Mesosome
D. Outer membrane
Answer: B
Source: Jawetz 23rd edition; page 11
MPL: 0.33
14. Responsible for the endotoxin activity of lipopolysaccharide
A. Somatic O polysaccharide
C. Lipid A
B. Core polysaccharide
D. protein A
Answer: C
Source: Jawetz 23rd edition; page 28
MPL: 0.50
15. Use organic compounds as carbon source and chemical reactions as energy source
A. Chemolithotrophs
C. photoorganotrophs
B. Chemoorganotrophs
D. photolithotrophs
MICROBIOLOGY
Page 44 of 137
Answer: B
Source: Jawetz 23rd edition; page 63
MPL: 1.00
16. A mutant bacterium lacks the ability to produce the amino acid leucine and grows only in a culture medium
containing leucine. This bacterium was exposed to a preparation of DNA from Pseudomonas aeruginosa
and was able to grow in a medium without leucine. What mechanism or event may have taken place?
A. Transduction
C. Conjugation
B. Transformation
D. Chromosomal mutation
Answer: B
Source: Jawetz 23rd edition; page 106
MPL: 0.33
17. Normally sterile site in the body
A. Mouth
B. Anus
Answer: D
Source: Jawetz 23rd edition; page 196
MPL: 1.00
C. Vagina
D. CSF
18. A strain of Streptococcus pyogenes is able to produce pyrogenic exotoxin encoded by a prophage is an
example of:
A. Transposition
C. Conjugation
B. Lysogeny
D. Chromosomal mutation
Answer: B
Source: Jawetz 23rd edition; page 100
MPL: 0.50
19. Calcium dipicalinate is found in:
A. Aspergillus
B. Bacillus
Answer: B
Source: Jawetz 23rd edition; page 28
MPL: 0.50
C. Escherichia
D. Mycobacteria
20. Antimicrobial which exert its action through inhibition of cell wall synthesis.
A. Aminoglycoside
C. Penicillin
B. Tetracycline
D. Rifampicin
Answer: C
Source: Jawetz 23rd edition; page 175
MPL: 1.00
21. The diagnosis of a primary infection requires that a 4-fold increase in titer of this antibody be demonstrated
A. IgG
C. IgD
B. IgM
D. IgA
Answer: A
Source: Jawetz 23rd edition; page 128
MPL: 1.00
22. The presence of granules in discharge from draining sinus tracts is indicative of
A. Chromomycosis
C. Lobomycosis
B. Mycetoma
D. Sporotrichosis
Answer: B
Source: Jawetz 23rd edition; pages 635-636
MPL: 0.25
23. Cave explorers who develop pneumonia following such explorations are likely to have acquired
A. Blastomycosis
C. Histoplasmosis
MICROBIOLOGY
Page 45 of 137
B. Candidiasis
Answer: C
Source: Jawetz 23rd edition; pages 640-642
MPL: 0.50
D. Cryptococcosis
24. A diver in a deep stagnant water develops polypoid masses in his nostrils. He is likely to have acquired
A. Chromomycosis
C. Lobomycosis
B. Rhinosporidiosis
D. Sporotrichosis
Answer: B
Source: Jawetz 23rd edition; page 632
MPL: 0.50
25. A farmer developed verrucous and warty lesions on his legs. Upon examination of sample of skin crusting
by KOH preparation, dark colored sclerotic bodies were seen. The most likely diagnosis is
A. Chromomycosis B. mycetoma
C. lobomycosis D. sporotrichosis
Answer: A
Source: Jawetz 23rd edition; page 634
MPL: 0.50
26. A gardener developed multiple subcutaneous nodules on his forearm. A biopsy material did not show any
fungus on direct microscopic examination. However, in culture, white, pasty and moist colonies were seen.
Wet mount using lactophenol cotton blue of the culture showed the presence of very fine hyphae and teardrop-shaped conidia.
A. The probable organism will form yeast cells when grown in brain heart infusion blood agar at 37 C
B. The culture above showed the tissue form of the fungus
C. The infection was most likely acquired from diving in deep stagnant water
D. The causative fungus easily disseminates to other tissues
Answer: A
Source: Jawetz 23rd edition; page 633
MPL: 0.50
27. The presence of multiple budding cells in chain in a biopsy specimen obtained from subcutaneous nodules
in the legs is suggestive of
A. Paracoccidioidomycosis
C. lobomycosis
B. Blastomycosis
D. sporotrichosis
Answer: C
Source: Jawetz 23rd edition; page 636
MPL: 0.50
28. The tissue form of Histoplasma capsulatum is seen as
A. Intracellular yeast in macrophages C. multiple budding cells
B. Spherules
D. encapsulated yeast
Answer: A
Source: Jawetz 23rd edition; page 640
MPL: 0.50
29. The infection caused by this fungus is initiated by inhalation of arthroconidia, which are found in semi-arid
regions
A. C. neoformans
C. B. dermatitidis
B. C. immitis
D. H. capsulatum
Answer: B
Source: Jawetz 23rd edition; page 638
MPL: 0.50
30. In histoplasmosis, the causative organism identified based on the presence of
A. tuberculate macroconidia in culture at room temperature
B. encapsulated yeast by India ink preparation
MICROBIOLOGY
Page 46 of 137
C. mucoid colonies on Sabouraud’s agar
D. single budding yeast cells in infected tissue
Answer: A
Source: Jawetz 23rd edition; page 640
MPL: 0.50
31. Which of the following is true about the members of the family Enterobacteriaceae?
A. Salmonella serotype typhi is transmitted by human carriers as well as by animals
B. Shigella dysenteriae toxin kills cells by activating adenyl cylcase
C. Escherichia coli usually produces intestinal disease only when it carries a plasmid or a phage that
code for virulence factors
D. Proteus mirabilis is an important cause of gastroenteritis
Answer: C
Source: Jawetz 23rd edition; page 252
MPL: 0.50
32. True about opportunistic members of the family Enterobacteriaceae
A. They are nonpathogenic in the intestinal tract
B. Infections are produced in and out of the intestines
C. They are the most abundant flora of the colon
D. They are not known to cause septicemia
Answer: A
Source: Jawetz 23rd edition; page 248
MPL: 0.50
33. Which of the following is true about Rickettsiae
A. All are transmitted by arthropods
B. All infections are characterized by skin rashes
C. All are obligate intracellular parasites
D. All survive the environment by producing endospores
Answer: C
Source: Jawetz 23rd edition; page 349
MPL: 0.50
34. A 16-year old salesgirl present to the physician with the chief complaints of headache, vomiting, neck
stiffness, high-grade fever, photophobia and petechial rashes. P.E. reveals an ill-appearing child unable to
flex his neck without eliciting pain and diffuse petechial rashes in his extremities. Kernig and Brudzinski
signs are positive. CSF analysis reveals increased neutrophils, increased protein and low glucose. What is
the most probable causative agent
A. Listeria monocytogenes
C. Neisseria meningitidis
B. Escherichia coli
D. Reovirus
Answer: C
Source: Jawetz 23rd edition; page 302
MPL: 0.50
35. Which is the best method of diagnosis of typhoid fever?
A. Widal test
C. blood culture
B. Typhi dot
D. stool culture
Answer: C
Source: Jawetz 23rd edition; page 258
MPL: 0.50
36. Which is the most common cause of non-gonococcal urethritis?
A. C. trachomatis
C. U. urealyticum
B. M. pneumoniae
D. S. aureus
Answer: A
Source: Jawetz 23rd edition; page 361
MICROBIOLOGY
Page 47 of 137
MPL: 0.50
37. Resistance to tuberculosis is mediated by
A. Cell-mediated immunity
B. Humoral immunity
Answer: A
Source: Jawetz 23rd edition; page 322
MPL: 0.50
C. interferon
D. NK cells
38. Which of the following is LEAST likely about Escherichia coli?
A. Pathogenicity in the GIT is associated with the presence of a plasmid
B. Strains producing K1 antigen are associated with neonatal meningitis
C. Enterotoxigenic strains produce diarrhea by invasion of the intestinal mucosa
D. Serotype O157:H7 is associated with hemolytic uremic syndrome
Answer: C
Source: Jawetz 23rd edition; page 253
MPL: 0.50
39. Which of the following characterize Salmonella typhi?
A. Infects man and animals
B. Non-invasive
C. The organism is a non-lactose fermenting, nonmotile bacillus
D. Chronic carriers harbor the organism in the gallbladder
Answer: D
Source: Jawetz 23rd edition; page 259
MPL: 0.50
40. One of the following produces a toxin that inhibits protein synthesis by ADP-ribosylation of EF-2
A. S. dysenteriae
C. C. diphtheriae
B. S. pyogenes
D. M. tuberculosis
Answer: C
Source: Jawetz 23rd edition; page 214
MPL: 0.50
41. Which of the following produces a mononucleosis-like disease
A. Orientia tsutsugamushi
C. Ehrlichia chafeensis
B. Rickettsia prowazeki
D. Coxiella burnetii
Answer: C
Source: Jawetz 23rd edition; page 354
MPL: 0.50
42. A 63-year old man has been hospitalized for the previous 21 days for the management of newly diagnosed
leukemia. During the period of hospitalization, the patient developed a urinary tract infection with Escherichia
coli and was treated for 14 days with broad-spectrum antibiotics. On the 21st hospital day, the patient
developed fever and shaking chills. Within 24 hours, the patient became hypotensive, and ecthymic skin
lesions appeared. Despite aggressive therapy with antibiotics, the patient expired. Multiple blood cultures
were positive for gram negative bacilli that showed negative results in most biochemical tests and failed to
ferment sugars. Which of the following is the MOST likely causative agent?
A. E. coli
B. K. pneumoniae C. L. pneumophila
D. P. aeruginosa
Answer: D
Source: Jawetz 23rd edition; page 264
MPL: 0.50
43. A 20-year-old woman presents with sudden onset of fever of 40 C, severe headache and generalized
petechial rash. Physical examination reveals nuchal rigidity. Meningococcemia is suspected and is
confirmed by a culture CSF. She has a history of several serious infections with the causative organism.
Which one of the following is the MOST likely predisposing factor?
A. She is HIV antibody-positive
B. She is deficient in CD8-positive cells
C. She is deficient in one of the late-acting complement components
D. She is deficient in antigen-presenting cells
Answer: C
MICROBIOLOGY
Page 48 of 137
Source: Jawetz 23rd edition; page 137
MPL: 0.50
44. This test will differentiate staphylococcus from streptococcus
A. coagulase
C. oxidase
B. catalase
D. sugar fermentation
Answer: B
Source: Jawetz 23rd edition; page 223
MPL: 0.50
45. Laboratory procedure of choice for primary syphilis.
A. darkfield microscopy
C. MHA TP
B. RPR
D. VDRL
Answer: A
Source: Jawetz 23rd edition; page 333
MPL: 0.50
46. Infective particle of chlamydia
A. Elementary body
B. Reticulate body
Answer: A
Source: Jawetz 23rd edition; page 357
MPL: 0.50_
C. endospore
D. fission body
47. The toxin of V. cholerae is antigenically similar to
A. shiga toxin
B. LT of E. coli
Answer: B
Source: Jawetz 23rd edition; page 270
MPL: 0.50
C. ST of E. coli
D. enterotoxin of shigella
48. Responsible for the virulence of M. tuberculosis
A. exotoxin
C. mycolic acid
B. trehalose 6,6 dimyculate
D. endotoxin
Answer: B
Source: Jawetz 23rd edition; page 321
MPL: 0.50
49. A 5-year-old girl was brought by her mother in the E.R. due to high grade fever. The patient also complained
of dysphagia and sore throat. On physical exam, the throat was erythematous throat with white, leathery
membrane on both tonsillar pillars. Culture and sensitivity was done to the patient. Corynebacterium dipthteriae
was isolated.
This organism produces dipthteria toxin. This toxin
C. is a neurotoxin
C. induce cAMP production
D. inhibits protein synthesis
D. heat stable
Answer: B
Source: Jawetz 23rd edition; page 214
MPL: 0.50
50. Purpose/s of giving Penicillin in cases of tetanus infection is/are:
A. inhibit the growth of C. tetani
B. neutralize unbound toxin
C. relax the muscle
D. to sterilize the necrotic tissue
Answer: A
Source: Jawetz 23rd edition; page 307
MPL: 0.50
MICROBIOLOGY
Page 49 of 137
51. A high titer of this in the serum indicates acute or recent infection
A. IgG
B. IgM
C. IgD
Answer: B
Source: Jawetz 23rd edition; page 129
MPL: 1.00
D. IgA
52. A patient had a viral infection that started to manifest 7 days ago. Laboratory test showed that the patient’s
antibodies against this virus have a high ratio of IgM to IgG. What is your conclusion?
A. It’s likely that the patient has been exposed to the virus for the first time
B. It’s likely that the patient has an autoimmune disease
C. The patient is predisposed to type 1 hypersensitivity reactions
D. The patient developed an anamnestic response to the virus
Answer: A
Source: Jawetz 23rd edition; page 135
MPL: 1.00
53. A patient is suspected of having diphtheria. The bacterium was isolated but you want to test if it produces
an exotoxin. An agar plate, in which a filter paper containing specific amtitoxin is embedded, was streaked
with the bacterial isolate perpendicular to the filter paper. A precipitin line was formed between the antitoxin
and the isolate. Which technique was used?
A. double immunodiffusion
C. single radial immunodiffusion
B. immunoelectrophoresis
D. immunoblot
Answer: A
Source: Jawetz 23rd edition; page 142
MPL: 0.50
54. You want to determine whether your patient is infected with a deadly virus, HIV. Which is the most sensitive
technique that you will request in the laboratory?
A. complement fixation
C. countercurrent immunoelectrophoresis
B. ELISA
D. indirect immunoflorescence
Answer: B
Source: Jawetz 23rd edition; page 142
MPL: 1.00
55. The biologic properties of Ig is a function of
A. Fc fragment
B. Fab fragment
Answer: A
Source: Jawetz 23rd edition; page 127
MPL: 0.50
56. High levels in fetus with intrauterine infection:
A. IgG
B. IgM
Answer: B
Source: Jawetz 23rd edition; page 129
MPL: 1.00
57. Predominant antibody in tears
A. IgG
B. IgM
Answer: C
Source: Jawetz 23rd edition; page 129
MPL: 1.00
58. Predominant antibody in secondary immune response
A. IgG
B. IgM
Answer: A
Source: Jawetz 23rd edition; page 135
C. Hinge region
D. Disulfide bonds
C. IgE
D. IgD
C. IgA
D. IgE
C. IgA
D. IgE
MICROBIOLOGY
Page 50 of 137
MPL: 1.00
59. The coating of bacteria by antibody leading to engulfment by phagocytes is known as
A. ADCC
C. anamnestic response
B. opsonization
D. hybridization
Answer: B
Source: Jawetz 23rd edition; page 121
MPL: 0.50
60. This function of the T cells plays a central role in both humoral and CMI.
A. effector function
C. regulatory function
B. anaphylaxis
D. atopy
Answer: C
Source: Jawetz 23rd edition; page 139
MPL: 0.50
61. Which is true about the complement?
A. All of the complement components are produced by the liver
B. Always in an active state under normal conditions.
C. Activated via a cascade mechanism.
D. Alternative pathway is part of the Adaptive Immunity
Answer: C
Source: Jawetz 23rd edition; page 136
MPL: 0.50
62. Dengue hemorrahgic fever is an example of:
A. Anaphylactic
B. Cytotoxic
Answer: C
Source: Jawetz 23rd edition; page 526
MPL: 0.50_
C. Immune complex
D. Delayed
63. What method will you use to detect antigen directly in tissue?
A. ELISA
C. Immunoflourescence
B. Immunoelectrophoresis
D. Hemagglutination
Answer: C
Source: Jawetz 23rd edition; page 142
MPL: 0.50
64. Protective antibody in viral infections localized in the respiratory tract
A. IgG
B. IgM
C. IgA
Answer: C
Source: Jawetz 23rd edition; page 129
MPL: 1.00
65. Which is the most potent anaphylatoxin:
A. C5a
B. C5b
Answer: A
Source: Jawetz 23rd edition; page 137
MPL: 0.50
C. C4a
D. IgE
D. C3a
66. The Fc region of an immunoglobulin is involved in
A. binding with antigen
C. interaction with class I MHC of T cells
B. binding with complement
D. all of the above
Answer: B
Source: Jawetz 23rd edition; page 128
MPL: 0.50
67. A 3-year-old boy is having recurrent infections particularly by encapsulated bacteria. You suspect that he is
immunodeficient particularly a low level of IgA. The most appropriate test to determine this is
A. single radial immunodiffusion
C. Immunoflourescnce
B. ELISA
D. complement fixation test
MICROBIOLOGY
Page 51 of 137
Answer: A
Source: Jawetz 23rd edition; page 142
MPL: 0.50
68. The most common cause of bronchiolitis and pneumonitis in infants
A. adenovirus
C. RSV
B. parainfluenza virus
D. measles virus
Answer: C
Source: Jawetz 23rd edition; page 558
MPL: 1.00
69. Which is pathognomonic of Rabies?
A. viral culture
B. negri bodies
Answer: B
Source: Jawetz 23rd edition; page 578
MPL: 1.00
C. flourescent antibody test
D. none
70. A patient who partially recover from paralytic Poliomyelitis experiences a new onset of muscular weakness,
pain and muscular atrophy, 25-35 years after the acute illness. Which is condition is described?
A. Aseptic Poliomyelitis
C. Paralytic Polio
B. Post-poliomyelitis Syndrome
D. Non-paralytic Polio
Answer: B
Source: Jawetz 23rd edition; page 492
MPL: 0.50
71. The most antigenically unstable influenza type
A. type A
B. type B
Answer: A
Source: Jawetz 23rd edition; page 537
MPL: 0.50
C. type C
D. all
72. Which denotes infectivity and HBV replication
A. Hbe Ag
B. HBs Ah
Answer: A
Source: Jawetz 23rd edition; page 467
MPL: 0.50
73. Which is NOT a member of family Herpesviridae
a. herpes simplex virus
b. cytomegalovirus
Answer: D
Source: Jawetz 23rd edition; page 429
MPL: 1.00
C. Anti HBe
D. Anti HBs
C. chickenpox virus
D. variola virus
74. Which is true about Hepadnaviridae?
A. has RNA genome
B. carries reverse transcriptase
C. has infectious genome
D. dependent on delta hapatitis virus for replication
Answer: D
Source: Jawetz 23rd edition; page 466
MPL: 0.50
75. A neutralization test was done in a patient who is suspected to be suffering from a viral infection
characterized by fever, malaise and respiratory symptoms. Using anti-echovirus antibody, no CPE was
observed. Which statement is true regarding this infection?
A. The patient is infected with enterovirus
B. The patient is infected with echovirus
C. The patient is unlikely to produce antibodies against enterovirus
D. The result is inconclusive, other tests must be done
MICROBIOLOGY
Page 52 of 137
Answer: B
Source: Jawetz 23rd edition; page 499
MPL: 0.50
76. Viruses are cultivated in the following EXCEPT
A. cell culture
B. chick embryo
Answer: D
Source: Jawetz 23rd edition; page 379
MPL: 1.00
C. animals
D. enriched agar medium
77. Which of the following identifies a virus using a labeled specific antibody?
A. Immunoflourescence
C. electron microscopy
B. PCR
D. virus isolation
Answer: A
Source: Jawetz 23rd edition; page 729
MPL: 1.00
78. Hemagglutination inhibition test is NOT applicable in which virus
A. influenza virus
C. RSV
B. mumps virus
D. parainfluenza virus
Answer: C
Source: Jawetz 23rd edition; page 559
MPL: 0.50
79. A 2-month pregnant mother suspected of having acquired rubella because of exposure to a friend who has
the illness, was tested for antibodies using HAI test. The result showed that the mother was positive for IgG
but not IgM. Which is the best statement that describes this case?
A. The mother has been infected with the virus in the past before pregnancy
B. The mother is incubating snd would manifest the infection later
C. The fetus will most likely be infected transplacentally
D. The mother has no immunity
Answer: A
Source: Jawetz 23rd edition; page 567
MPL: 1.00
80. A hemagglutination inhibition test was done on the serum of a 6-month old boy, who is suffering from
severe respiratory distress. The results showed that hemagglutination occurred when parainfluenza virus
type 1 Ag was reacted with the serum; no hemagglutination with parainfluenza virus type 2;
hemagglutination with RSV. Which is the infecting agent?
A. Parainfluenza type 1
C. RSV
B. Parainfluenza type 2
D. the result is inconclusive
Answer: B
Source: Jawetz 23rd edition; page 729
MPL: 0.50
81. Genetic reassortment occurs when segmented genomes of duck strains of influenza A virus are enclosed
together with those of human strains in a capsid. Which of the followng is the most likely to occur?
A. point mutation
B. new strain of virus will appear
C. the human population is protected because of previous infection with human strain
D. antigenic drift
Answer: B
Source: Jawetz 23rd edition; page 545
MPL: 0.50
82. Which is associated with congenital abnormalities?
A. mumps
B. german measles
C. rubeola
D. all of the above
MICROBIOLOGY
Page 53 of 137
Answer: B
Source: Jawetz 23rd edition; page 567
MPL: 1.00
83. Which is true about poliovirus infection?
A. CNS involvement is NOT common
B. Most infections have translent viremia
C. The virus multiplies in the muscles
D. There is no effective preventive measure
Answer: B
Source: Jawetz 23rd edition; page 491
MPL: 0.50
84. Herpangina, which is characterized by the formation of vesicular lesions in the pharynx is most commonly
associated with
A. poliovirus
C. coxsackie A virus
B. hepatitis A virus
D. coxsackie B virus
Answer: C
Source: Jawetz 23rd edition; page 494
MPL: 0.50
85. An astray dog bit your neighbor. The wound was bleeding and was cleaned with soap and water. Your
advise will be
A. kill the dog and submit the head for examination for Negri bodies
B. give rabies immunoglobulin and vaccine
C. observe the dog
D. observe the patient
Answer: B
Source: Jawetz 23rd edition; page 581
MPL: 0.50
86. Latency occurs in the trigeminal ganglion
A. HSV 1
B. HSV 2
Answer: A
Source: Jawetz 23rd edition; page 400
MPL: 0.50
87. Latency occurs in the mononuclear cells and kidneys.
A. HSV 1
B. HSV 3
Answer: C
Source: Jawetz 23rd edition; page 442
MPL: 0.50
C. HSV 6
D. HSV 8
C. CMV
D. Variola
88. The difference between Dengue fever and Dengue Hemorrhagic fever is that
A. presence of bleeding in DHF
C. PPT is prolonged in DHF
B. hematocrit is elevated in DHF
D. absence of AbV in the former formation
Answer: B
Source: Jawetz 23rd edition; page 526
MPL: 0.33
89. A 4-year-old girl is brought to the physician’s office because she developed red cheeks that appear as if
someone has slapped her, and a lacy ash on her upper extremities and trunks. What is the MOST probable causative
agent?
A. Adenovirus
C. Marburg virus
B. Parvovirus
D. Rotavirus
Answer: B
Source: Jawetz 23rd edition; page 418
MICROBIOLOGY
Page 54 of 137
MPL: 0.50
90. Defective virus that cause infection only in presence of HBV
A. HAV
B. HBV
C. HCV
Answer: D
Source: Jawetz 23rd edition; page 470
MPL: 1.00
D. HDV
91. Very important serologic marker for HBV during the window phase
A. Anti HBs
C. Anti HBc
B. HBs Ag
D. HBe Ag
Answer: C
Source: Jawetz 23rd edition; page 467
MPL: 1.00
92. A modified cellular protein that is capable of auto-catalytically make several copies of itself
A. viroid
C. prion
B. virusoid
D. satelitte RNA
Answer: C
Source: Jawetz 23rd edition; page 584
MPL: 0.33
93. Responsible for attachment of influenza virus on specific host cell receptor
A. H glycoprotein
C. matrix protein
B. N glycoprotein
D. capsid protein
Answer: A
Source: Jawetz 23rd edition; page 542
MPL: 0.50
94. Measles is most infectious during tha appearance of
A. Koplik spots
B. Fever, cough and coryza and conjunctivitis
C. Macupapular rash
D. Photophobia
Answer: B
Source: Jawetz 23rd edition; page 565
MPL: 0.50
95. Oseltamivir is an antiviral drug-used against
A. parainfluenza
C. measles
B. influenza
D. RSV
Answer: B
Source: Jawetz 23rd edition; page 547
MPL: 0.50
96. Which is associated with extrahepatic manifestations
A. HAV
B. HCV
C. HBV
Answer: C
Source: Jawetz 23rd edition; page 466
MPL: 0.50
D. HEV
97. A woman appears in a hospital ER with vesicular lesions on her vulva. These lesions clinically resemble
those caused by herpes simplex virus. From which of the following clinical specimens would the virus most likely
to be isolated?
A. urine
C. Throat secretions
B. Blood
D. Vesicular fluid
Answer: D
Source: Jawetz 23rd edition; pages 433-438
MICROBIOLOGY
Page 55 of 137
MPL: 0.50
98. A 5-year old child presents fever for 8 days, lympadenopathy, splenomegaly and numerous reactive or
atypical lymphocytes on peripheral blood smear. The monospot test is negative. A likely cause of the clinical
picture is infection with
A. respiratory syncytial virus
C. Adenovirus
B. Cytomegalovirus
D. herpes virus
Answer: B
Source: Jawetz 23rd edition; pages 442-446
MPL: 0.50
99. A 30-year-old school teacher presents with cough of 1 mont duration, fever, night sweats and weight loss. PE
is normal. How many sputum smears are recommended by the council on Tuberculosis to establish a diagnosis
of TB?
A. Four
C. Two
B. Three
D. One
Answer: B
Source: Jawetz 23rd edition; pages 323-324
MPL: 0.50
100. A woman develops fever, rash and polyarthralgia during menstruation. Gram stain of the synovial fluid of
her right knee reveals gram-negative diplococci. Which of the following organisms is MOST likely to be
responsible?
A. Haemophilus influenzae
C. Neisseria gonorrhoeae
B. Staphylococcus epidermidis
D. Pseudomonas aeruginosa
Answer: C
Source: Jawetz 23rd edition; pages 295-301
MPL: 0.50
(0.5)
1. The habitat is the large intestine.
A] Entamoeba gingivalis
C] Giardia lamblia
*B] Entamoeba histolytica
D] Naegleria fowleri
(0.5)
2. The stool is the specimen for the diagnosis of the infection cause by
A] Acanthamoeba polyphaga
C] Naegleria fowleri
*B] Balantidium coli
D] A & B
(0.5)
3. The infective stage of Entamoeba histolytica to man has
A] pseudopodia
*C] bull’s eye karyosome
B] ingested red blood cells
D] A & B
(0.5)
4. The pathogenic stage of Entamoeba histolytica to man has
A] pseudopodia
C] cigar-shaped chromatoidal body
B] ingested RBC
*D] A & B
(1)
5. The stool of the patient infected with what parasite is described as steatorrheic or gruelly?
A] Balantidium coli
C] Entamoeba histolytica
B] Chilomastix mesnili
*D] Giardia lamblia
(0.5)
6. The most commonly involved area in Amoebiasis is the
A] brain
C] liver
*B] cecum
D] recto-sigmoid region
(0.5)
7. Typically flask-shaped lesion of Amoebiasis is usually seen in
*A] Amoebic colitis
C] Amoebic liver abscess
B] Amoebiasis cutis
D] Pulmonary amoebiasis
(0.5)
8. Frank dysentery in Amoebiasis is frequently present with the involvement of the
A] brain
C] liver
B] cecum
*D] recto-sigmoid region
(0.5)
9. Anchovy sauce-like materials is associated with
A] Amoebic colitis
*C] Amoebic liver abscess
B] Amoebiasis cutis
D] Pulmonary amoebiasis
(0.5)
10. There is no increase in the number of nuclei of
*A] Balantidium coli
C] Entamoeba histolytica
B] Entamoeba coli
D] Giardia lamblia
(0.5)
11. Associated with uveitis or ulceration of the cornea.
*A] Acanthamoeba culbertsoni
C] Entamoeba histolytica
B] Balantidium coli
D] Naegleria fowleri
(0.33) 12. Tissue invasion is relatively slow and tends to stimulate granuloma formation.
MICROBIOLOGY
Page 56 of 137
(0.5)
(0.5)
(1)
*A] Acanthamoeba castellanii
C] Entamoeba histolytica
B] Balantidium coli
D] Naegleria fowleri
13. Produces a big lesion with a wide opening and a rounded base in the large intestine.
*A] Balantidium coli
C] Giardia lamblia
B] Entamoeba histolytica
D] A & B
14. There are indurations of the irregular margins of the spreading ulcer.
A] Amoebic granuloma
C] Amoeboma
*B] Amoebiasis cutis
D] Amoebic colitis
15. Associated with Entamoeba gingivalis.
A] Trichomonas hominis
C] Trichomonas vaginalis
*B] Trichomonas tenax
D] B & C
2
MULTIPLE CHOICE QUESTIONS: CHOOSE THE BEST ANSWER.
(1)
(0.5)
(1)
(0.5)
(1)
(0.5)
(0.5)
(1)
(1)
(1)
(0.5)
(0.5)
(1)
(0.5)
(0.5)
16. The specimen for the diagnosis of Trichomonas vaginalis infection in female.
A] Prostatic secretions
C] Vaginal discharges
B] Urine
*D] B & C
17. The usual infective stage of Malaria to man is the
A] gametocytes
*C] sporozoites
B] schizonts
D] merozoites
18. Mosquito vector of Malaria in the Philippines is
A] Aedes aegypti
C] Culex quinquefasciatus
*B] Anopheles flavirostris
D] Mansonia annulata
19. There is secondary exo-erythrocytic phase in
*A] Benign tertian malaria
C] Quartan malaria
B] Malignant tertian malaria
D] Sub-tertian malaria
20. Produces the more severe type of Malaria.
*A] Plasmodium falciparum
C] Plasmodium ovale
B] Plasmodium malariae
D] Plasmodium vivax
21. Usually produces recrudescence.
A] Plasmodium falciparum
C] Plasmodium vivax
B] Plasmodium malariae
*D] A & B
22. The duration of the Erythrocytic-Schizogonic Cycle is 72 hours.
A] Benign tertian malaria
*C] Quartan malaria
B] Malignant tertian malaria
D] A & B
23. Sub-tertian Malaria is cause by.
*A] Plasmodium falciparum
C] Plasmodium ovale
B] Plasmodium malariae
D] Plasmodium vivax
24. The most common Malaria in the Philippines is
*A] Plasmodium falciparum
C] Plasmodium ovale
B] Plasmodium malariae
D] Plasmodium vivax
25. Produces Ziemann’s dot.
A] Plasmodium falciparum
C] Plasmodium ovale
*B] Plasmodium malariae
D] Plasmodium vivax
26. Stage/s of Malaria usually found in man.
A] Gametes
C] Trophozoites
B] Schizonts
*D] B & C
27. Malaria with the highest degree of parasitemia in man.
*A] Plasmodium falciparum
C] Plasmodium vivax
B] Plasmodium malariae
D] A & B
28. The gametocytes of the organism are described as banana or crescent-shaped.
*A] Plasmodium falciparum
C] Plasmodium ovale
B] Plasmodium malariae
D] Plasmodium vivax
29. Some strains are resistance to Chloroquine or Amodiaquine.
*A] Plasmodium falciparum
C] Plasmodium vivax
B] Plasmodium malariae
D] A & B
30. Opportunistic Protozoa in the stomach and intestine.
*A] Cryptosporidium parvum
C] Toxoplasma gondii
MICROBIOLOGY
Page 57 of 137
(0.5)
B] Pneumocystis carinii
31. Produces congenital infection in man.
A] Cryptosporidium parvum
B] Pneumocystis carinii
D] A & C
*C] Toxoplasma gondii
D] A & C
3
MULTIPLE CHOICE QUESTIONS: CHOOSE THE BEST ANSWER.
(0.5)
(1)
(0.33)
(0.5)
(0.33)
(0.33)
(0.33)
(0.5)
(0.5)
(0.5)
(0.5)
(1)
(0.5)
(0.5)
(0.33)
(0.25)
32. Produces massive diarrhea in patient with low resistance.
*A] Cryptosporidium parvum
C] Toxoplasma gondii
B] Pneumocystis carinii
D] A & C
33. Associated with hypogammaglobulinemia and respiratory distress.
A] Cryptosporidium parvum
C] Toxoplasma gondii
*B] Pneumocystis carinii
D] A & C
34. The encysted larva of the organism is the infective stage to man.
A] Capillaria philippinensis
C] Trichuris trichiura
*B] Trichinella spiralis
D] B & C
35. Produces rectal prolapse in heavy infection especially among children.
A] Capillaria philippinensis
*C] Trichuris trichiura
B] Trichinella spiralis
D] B & C
36. Produces malabsorption and cachexia in man.
*A] Capillaria philippinensis
C] Trichuris trichiura
B] Trichinella spiralis
D] A & C
37. The habitat is the small intestine.
A] Capillaria philippinensis
C] Trichuris trichiura
B] Trichinella spiralis
*D] A & B
38. The female parasites may be larviparous.
A] Capillaria philippinensis
C] Trichuris trichiura
B] Trichinella spiralis
*D] A & B
39. The infective stage to man is the embryonated egg.
A] Capillaria philippinensis
*C] Trichuris trichiura
B] Trichinella spiralis
D] A & C
40. The parasite/s that is/are associated with fish.
*A] Capillaria philippinensis
C] Trichuris trichiura
B] Trichinella spiralis
D] A & C
41. There is a dead-end cycle in man.
A] Capillaria philippinensis
C] Trichuris trichiura
*B] Trichinella spiralis
D] A & C
42. The stool is the specimen for the diagnosis of the infection cause by
A] Capillaria philippinensis
C] Trichuris trichiura
B] Trichinella spiralis
*D] A & C
43. Bachman intradermal test & Xenodiagnosis are use for the diagnosis of infection cause by
A] Capillaria philippinensis
C] Trichuris trichiura
*B] Trichinella spiralis
D] A & B
44. The habitat is the small intestine.
A] Ascaris lumbricoides
C] Trichuris trichiura
B] Necator americanus
*D] A & B
45. Geophagia or Pica may be seen in heavy infection with
A] Ascaris lumbricoides
*C] Necator americanus
B] Enterobius vermicularis
D] Strongyloides stercoralis
46. The most common cause of Creeping Eruption or Cutaneous Larva Migran.
*A] Ancylostoma braziliense
C] Strongyloides stercoralis
B] Enterobius vermicularis
D] Toxocara canis
47. Most frequent finding in the infection cause by the parasite is a persistent hypereosinophilia.
A] Ancylostoma braziliense
C] Strongyloides stercoralis
B] Enterobius vermicularis
*D] Toxocara canis
MICROBIOLOGY
Page 58 of 137
4
MULTIPLE CHOICE QUESTIONS: CHOOSE THE BEST ANSWER.
(0.33)
(0.33)
(0.5)
(0.5)
(0.33)
(0.33)
(0.33)
(1)
(0.33)
(1)
(0.5)
(1)
(1)
(0.33)
(0.5)
(0.5)
48. The most common cause of Visceral Larva Migran.
A] Ancylostoma braziliense
C] Strongyloides stercoralis
B] Enterobius vermicularis
*D] Toxocara canis
49. Produces autoinfection to man.
A] Ancylostoma duodenale
C] Necator americanus
B] Ascaris lumbricoides
*D] Strongyloides stercoralis
50. What parasite/s has a blood-lung phase in the life cycle?
A] Ascaris lumbricoides
C] Strongyloides stercoralis
B] Enterobius vermicularis
*D] A &C
51. The adults and migrating larvae are the pathogenic stages of
A] Ascaris lumbricoides
C] Strongyloides stercoralis
B] Enterobius vermicularis
*D] A &C
52. What parasite/s may produce Loeffler’s syndrome to man?
*A] Ascaris lumbricoides
C] Gnathostoma spinigerum
B] Enterobius vermicularis
D] Trichuris trichiura
53. Stage/s of the parasite that may produce Loeffler’s syndrome to man.
A] Adult
*C] Larva
B] Egg
D] A & C
54. The usual manner of transmission by the parasite is by skin penetration.
A] Ascaris lumbricoides
C] Strongyloides stercoralis
B] Necator americanus
*D] B & C
55. What parasite produces Iron deficiency anemia to man?
A] Ascaris lumbricoides
C] Strongyloides stercoralis
*B] Necator americanus
D] Trichuris trichiura
56. Produces Ground itch to man.
A] Ascaris lumbricoides
*C] Necator americanus
B] Enterobius vermicularis
D] Trichuris trichiura
57. Associated with Nocturnal Pruritus Ani.
A] Ascaris lumbricoides
C] Necator americanus
*B] Enterobius vermicularis
D] Trichuris trichiura
58. The manner of transmission of the parasite is by ingestion of the embryonated egg.
A] Ascaris lumbricoides
C] Trichuris trichiura
B] Strongyloides stercoralis
*D] A & C
59. Graham Scotch tape technique is used for the diagnosis of the infection cause by
A] Ascaris lumbricoides
C] Necator americanus
*B] Enterobius vermicularis
D] Trichuris trichiura
60. What parasite has an egg that is plano-convex and is readily infective after discharged?
A] Ancylostoma duodenale
*C] Enterobius vermicularis
B] Ascaris lumbricoides
D] Trichuris trichiura
61. The parasite has cephalic alae.
A] Ancylostoma caninum
C] Necator americanus
*B] Enterobius vermicularis
D] Toxocara canis
62. Produces group infection.
*A] Enterobius vermicularis
C] Toxocara cati
B] Toxocara canis
D] B & C
63. What parasite sucks and ingests blood?
*A] Ancylostoma duodenale
C] Enterobius vermicularis
B] Ascaris lumbricoides
D] Trichuris trichiura
5
MULTIPLE CHOICE QUESTIONS: CHOOSE THE BEST ANSWER.
MICROBIOLOGY
Page 59 of 137
(0.25)
(0.25)
(0.5)
(0.5)
(0.5)
(1)
(0.25)
(1)
(0.33)
(0.33)
(0.5)
(0.5)
(0.5)
(1)
(1)
(1)
64. What parasite shows Tramway sign in the X-ray?
*A] Ascaris lumbricoides
C] Necator americanus
B] Enterobius vermicularis
D] Trichuris trichiura
65. What parasite whose migrating larvae break the pulmonary capillaries of man?
A] Ancylostoma braziliense
C] Enterobius vermicularis
*B] Ascaris lumbricoides
D] Trichuris trichiura
66. Associated with mosquito.
A] Brugia malayi
C] Wuchereria bancrofti
B] Loa loa
*D] A & C
67. Knott’s method is use to demonstrate the microfilariae of
A] Brugia malayi
C] Wuchereria bancrofti
B] Onchocerca volvulus
*D] A & C
68. The usual infective stage of Trematodes to man is the
A] cercariae
*C] metacercariae
B] egg
D] miracidium
69. The infective stage of the parasite is found in crabs or crayfishes.
A] Clonorchis sinensis
C] Fasciola hepatica
B] Echinostoma ilocanum
*D] Paragonimus westermani
70. What parasite is associated with neoplasm of the biliary duct or cancer of the liver?
*A] Clonorchis sinensis
C] Opisthorchis felineus
B] Fasciola hepatica
D] Paragonimus westermani
71. What parasite produces infection that simulates tuberculosis?
A] Clonorchis sinensis
C] Opisthorchis felineus
B] Fasciola hepatica
*D] Paragonimus westermani
72. The infective stage of what parasite is encysted in aquatic vegetations?
A] Clonorchis sinensis
C] Echinostoma ilocanum
*B] Fasciolopsis buski
D] Paragonimus westermani
73. Snails are both the first and second intermediate hosts of
*A] Echinostoma ilocanum
C] Fasciola hepatica
B] Fasciola hepatica
D] Paragonimus westermani
74. The eggs of Paragonimus westermani may be demonstrated from the
A] sputum
C] urine
B] stool
*D] A & B
75. What stage of the Trematodes swims in the water?
*A] Cercariae
C] Metacercariae
B] Coracidium
D] Sporocyst
76. What is the infective stage of Schistosoma to man?
*A] Cercariae
C] Metacercariae
B] Embryonated egg
D] Miracidium
77. What is the usual manner of transmission of Schistosoma to man?
A] Arthropod vector
*C] Skin penetration of the cercariae
B] Ingestion of the embryonated egg
D] B & C
78. Molluscan host of Schistosoma in the Philippines is
A] Australorbis mystax
C] Coxiella burnetii
B] Biomphalaria alexandrina
*D] Oncomelania quadrasi
79. What Schistosoma is present in the Philippines?
A] Schistosoma haematobium
C] Schistosoma mansoni
*B] Schistosoma japonicum
D] Schistosoma mekongi
6
MULTIPLE CHOICE QUESTIONS: CHOOSE THE BEST ANSWER.
(0.33)
(0.33)
80. Laboratory procedure that makes use of the egg in the test is
A] CFT
*C] COPT
B] CHR
D] Casoni test
81. What Schistosoma produces more severe infection to man?
A] Schistosoma haematobium
C] Schistosoma mansoni
*B] Schistosoma japonicum
D] Schistosoma mekongi
MICROBIOLOGY
Page 60 of 137
(0.33)
(0.33)
(0.33)
(0.33)
(0.33)
(1)
(0.25)
(0.25)
(0.5)
(0.5)
(0.5)
(0.33)
(0.5)
(1)
82. What Schistosoma is more resistant to treatment?
A] Schistosoma haematobium
C] Schistosoma mansoni
*B] Schistosoma japonicum
D] Schistosoma mekongi
83. Pipe-stem fibrosis of the liver in Schistosomiasis is due to what stage of the parasite?
A] Adult
*C] Egg
B] Cercaria
D] Metacercaria
84. Allergic manifestations in Schistosomiasis are due to what stage of the parasite?
*A] Adult
C] Egg
B] Cercaria
D] Metacercaria
85. Obstructive uropathy is associated with
*A] Schistosoma haematobium
C] Schistosoma mansoni
B] Schistosoma japonicum
D] Schistosoma mekongi
86. What Schistosoma frequently cause Gynecological Schistosomiasis.
*A] Schistosoma haematobium
C] Schistosoma mansoni
B] Schistosoma japonicum
D] Schistosoma mekongi
87. The region of growth in Cestodes is the
A] Scolex
C] Proglottid
*B] Neck
D] Strobila
88. The infective stage of Diphyllobothrium latum to man to produce adult infection is the
A] cysticercoid larva
C] procercoid larva
B] cysticercus larva
*D] sparganum larva
89. The most common cause of Sparganosis in Asia is
A] Diphyllobothrium latum
C] Echinococcus granulosus
B] Dipylidium caninum
*D] Spirometra mansoni
90. Bothriocephalus anemia is associated with
A] iron deficiency anemia
*C] pernicious anemia
B] microcytic hypochromic cells
D] A & B
91. Diphyllobothrium latum is associated with
A] cat
*C] fish
B] dog
D] pig
92. Produces Ocular cysticercosis to man.
A] Dipylidium caninum
*C] Taenia solium
B] Taenia saginata
D] B & C
93. Produces autoinfection in man.
A] Dipylidium caninum
C] Taenia saginata
B] Hymenolepis diminuta
*D] Taenia solium
94. Lateral uterine branches in gravid proglottid are counted for diagnosis of infection cause by
A] Diphyllobothrium latum
C] Hymenolepis nana
B] Dipylidium caninum
*D] Taenia saginata
95. Taenia saginata is associated with
A] cat
C] dog
*B] cattle
D] pig
7
MULTIPLE CHOICE QUESTIONS: CHOOSE THE BEST ANSWER.
(0.33)
(1)
(0.25)
(0.25)
96. Taeniasis solium is produced after ingestion of the
A] cysticercoid larvae
*C] cysticercus cellulosae
B] cysticercus bovis
D] Taenia egg
97. What parasite is associated with measly pork?
A] Diphyllobothrium latum
C] Taenia saginata
B] Dipylidium caninum
*D] Taenia solium
98. What Cestodes does NOT need an intermediate host?
A] Dipylidium caninum
C] Hymenolepis diminuta
B] Echinococcus granulosus
*D] Hymenolepis nana
99. The most commonly involved organ in Hydatid Disease is the
*A] liver
C] kidney
MICROBIOLOGY
Page 61 of 137
(0.33)
(0.5)
(0.5)
(0.5)
(0.5)
(1)
(0.5)
(0.5)
(0.5)
(0.5)
(0.5)
(0.5)
(0.33)
(0.5)
(0.5)
(1)
B] lungs
D] brain
100. Ultrasound and MRI usually help in the diagnosis of the infection cause by
A] Dipylidium caninum
C] Hymenolepis diminuta
*B] Echinococcus granulosus
D] Hymenolepis nana
1. The habitat is the large intestine.
A] Entamoeba gingivalis
C] Giardia lamblia
*B] Entamoeba histolytica
D] Naegleria fowleri
2. The stool is the specimen for the diagnosis of the infection cause by
A] Acanthamoeba polyphaga
C] Naegleria fowleri
*B] Balantidium coli
D] A & B
3. The infective stage of Entamoeba histolytica to man has
A] pseudopodia
*C] bull’s eye karyosome
B] ingested red blood cells
D] A & B
4. The pathogenic stage of Entamoeba histolytica to man has
A] pseudopodia
C] cigar-shaped chromatoidal body
B] ingested RBC
*D] A & B
5. The stool of the patient infected with what parasite is described as steatorrheic or gruelly?
A] Balantidium coli
C] Entamoeba histolytica
B] Chilomastix mesnili
*D] Giardia lamblia
6. The most commonly involved area in Amoebiasis is the
A] brain
C] liver
*B] cecum
D] recto-sigmoid region
7. Typically flask-shaped lesion of Amoebiasis is usually seen in
*A] Amoebic colitis
C] Amoebic liver abscess
B] Amoebiasis cutis
D] Pulmonary amoebiasis
8. Frank dysentery in Amoebiasis is frequently present with the involvement of the
A] brain
C] liver
B] cecum
*D] recto-sigmoid region
9. Anchovy sauce-like materials is associated with
A] Amoebic colitis
*C] Amoebic liver abscess
B] Amoebiasis cutis
D] Pulmonary amoebiasis
10. There is no increase in the number of nuclei of
*A] Balantidium coli
C] Entamoeba histolytica
B] Entamoeba coli
D] Giardia lamblia
11. Associated with uveitis or ulceration of the cornea.
*A] Acanthamoeba culbertsoni
C] Entamoeba histolytica
B] Balantidium coli
D] Naegleria fowleri
12. Tissue invasion is relatively slow and tends to stimulate granuloma formation.
*A] Acanthamoeba castellanii
C] Entamoeba histolytica
B] Balantidium coli
D] Naegleria fowleri
13. Produces a big lesion with a wide opening and a rounded base in the large intestine.
*A] Balantidium coli
C] Giardia lamblia
B] Entamoeba histolytica
D] A & B
14. There are indurations of the irregular margins of the spreading ulcer.
A] Amoebic granuloma
C] Amoeboma
*B] Amoebiasis cutis
D] Amoebic colitis
15. Associated with Entamoeba gingivalis.
A] Trichomonas hominis
C] Trichomonas vaginalis
*B] Trichomonas tenax
D] B & C
2
MULTIPLE CHOICE QUESTIONS: CHOOSE THE BEST ANSWER.
(1)
(0.5)
16. The specimen for the diagnosis of Trichomonas vaginalis infection in female.
A] Prostatic secretions
C] Vaginal discharges
B] Urine
*D] B & C
17. The usual infective stage of Malaria to man is the
MICROBIOLOGY
Page 62 of 137
(1)
(0.5)
(1)
(0.5)
(0.5)
(1)
(1)
(1)
(0.5)
(0.5)
(1)
(0.5)
(0.5)
(0.5)
A] gametocytes
*C] sporozoites
B] schizonts
D] merozoites
18. Mosquito vector of Malaria in the Philippines is
A] Aedes aegypti
C] Culex quinquefasciatus
*B] Anopheles flavirostris
D] Mansonia annulata
19. There is secondary exo-erythrocytic phase in
*A] Benign tertian malaria
C] Quartan malaria
B] Malignant tertian malaria
D] Sub-tertian malaria
20. Produces the more severe type of Malaria.
*A] Plasmodium falciparum
C] Plasmodium ovale
B] Plasmodium malariae
D] Plasmodium vivax
21. Usually produces recrudescence.
A] Plasmodium falciparum
C] Plasmodium vivax
B] Plasmodium malariae
*D] A & B
22. The duration of the Erythrocytic-Schizogonic Cycle is 72 hours.
A] Benign tertian malaria
*C] Quartan malaria
B] Malignant tertian malaria
D] A & B
23. Sub-tertian Malaria is cause by.
*A] Plasmodium falciparum
C] Plasmodium ovale
B] Plasmodium malariae
D] Plasmodium vivax
24. The most common Malaria in the Philippines is
*A] Plasmodium falciparum
C] Plasmodium ovale
B] Plasmodium malariae
D] Plasmodium vivax
25. Produces Ziemann’s dot.
A] Plasmodium falciparum
C] Plasmodium ovale
*B] Plasmodium malariae
D] Plasmodium vivax
26. Stage/s of Malaria usually found in man.
A] Gametes
C] Trophozoites
B] Schizonts
*D] B & C
27. Malaria with the highest degree of parasitemia in man.
*A] Plasmodium falciparum
C] Plasmodium vivax
B] Plasmodium malariae
D] A & B
28. The gametocytes of the organism are described as banana or crescent-shaped.
*A] Plasmodium falciparum
C] Plasmodium ovale
B] Plasmodium malariae
D] Plasmodium vivax
29. Some strains are resistance to Chloroquine or Amodiaquine.
*A] Plasmodium falciparum
C] Plasmodium vivax
B] Plasmodium malariae
D] A & B
30. Opportunistic Protozoa in the stomach and intestine.
*A] Cryptosporidium parvum
C] Toxoplasma gondii
B] Pneumocystis carinii
D] A & C
31. Produces congenital infection in man.
A] Cryptosporidium parvum
*C] Toxoplasma gondii
B] Pneumocystis carinii
D] A & C
3
MULTIPLE CHOICE QUESTIONS: CHOOSE THE BEST ANSWER.
(0.5)
(1)
(0.33)
(0.5)
32. Produces massive diarrhea in patient with low resistance.
*A] Cryptosporidium parvum
C] Toxoplasma gondii
B] Pneumocystis carinii
D] A & C
33. Associated with hypogammaglobulinemia and respiratory distress.
A] Cryptosporidium parvum
C] Toxoplasma gondii
*B] Pneumocystis carinii
D] A & C
34. The encysted larva of the organism is the infective stage to man.
A] Capillaria philippinensis
C] Trichuris trichiura
*B] Trichinella spiralis
D] B & C
35. Produces rectal prolapse in heavy infection especially among children.
MICROBIOLOGY
Page 63 of 137
(0.33)
(0.33)
(0.33)
(0.5)
(0.5)
(0.5)
(0.5)
(1)
(0.5)
(0.5)
(0.33)
(0.25)
A] Capillaria philippinensis
*C] Trichuris trichiura
B] Trichinella spiralis
D] B & C
36. Produces malabsorption and cachexia in man.
*A] Capillaria philippinensis
C] Trichuris trichiura
B] Trichinella spiralis
D] A & C
37. The habitat is the small intestine.
A] Capillaria philippinensis
C] Trichuris trichiura
B] Trichinella spiralis
*D] A & B
38. The female parasites may be larviparous.
A] Capillaria philippinensis
C] Trichuris trichiura
B] Trichinella spiralis
*D] A & B
39. The infective stage to man is the embryonated egg.
A] Capillaria philippinensis
*C] Trichuris trichiura
B] Trichinella spiralis
D] A & C
40. The parasite/s that is/are associated with fish.
*A] Capillaria philippinensis
C] Trichuris trichiura
B] Trichinella spiralis
D] A & C
41. There is a dead-end cycle in man.
A] Capillaria philippinensis
C] Trichuris trichiura
*B] Trichinella spiralis
D] A & C
42. The stool is the specimen for the diagnosis of the infection cause by
A] Capillaria philippinensis
C] Trichuris trichiura
B] Trichinella spiralis
*D] A & C
43. Bachman intradermal test & Xenodiagnosis are use for the diagnosis of infection cause by
A] Capillaria philippinensis
C] Trichuris trichiura
*B] Trichinella spiralis
D] A & B
44. The habitat is the small intestine.
A] Ascaris lumbricoides
C] Trichuris trichiura
B] Necator americanus
*D] A & B
45. Geophagia or Pica may be seen in heavy infection with
A] Ascaris lumbricoides
*C] Necator americanus
B] Enterobius vermicularis
D] Strongyloides stercoralis
46. The most common cause of Creeping Eruption or Cutaneous Larva Migran.
*A] Ancylostoma braziliense
C] Strongyloides stercoralis
B] Enterobius vermicularis
D] Toxocara canis
47. Most frequent finding in the infection cause by the parasite is a persistent hypereosinophilia.
A] Ancylostoma braziliense
C] Strongyloides stercoralis
B] Enterobius vermicularis
*D] Toxocara canis
4
MULTIPLE CHOICE QUESTIONS: CHOOSE THE BEST ANSWER.
(0.33)
(0.33)
(0.5)
(0.5)
(0.33)
(0.33)
48. The most common cause of Visceral Larva Migran.
A] Ancylostoma braziliense
C] Strongyloides stercoralis
B] Enterobius vermicularis
*D] Toxocara canis
49. Produces autoinfection to man.
A] Ancylostoma duodenale
C] Necator americanus
B] Ascaris lumbricoides
*D] Strongyloides stercoralis
50. What parasite/s has a blood-lung phase in the life cycle?
A] Ascaris lumbricoides
C] Strongyloides stercoralis
B] Enterobius vermicularis
*D] A &C
51. The adults and migrating larvae are the pathogenic stages of
A] Ascaris lumbricoides
C] Strongyloides stercoralis
B] Enterobius vermicularis
*D] A &C
52. What parasite/s may produce Loeffler’s syndrome to man?
*A] Ascaris lumbricoides
C] Gnathostoma spinigerum
B] Enterobius vermicularis
D] Trichuris trichiura
53. Stage/s of the parasite that may produce Loeffler’s syndrome to man.
MICROBIOLOGY
Page 64 of 137
(0.33)
(1)
(0.33)
(1)
(0.5)
(1)
(1)
(0.33)
(0.5)
(0.5)
A] Adult
*C] Larva
B] Egg
D] A & C
54. The usual manner of transmission by the parasite is by skin penetration.
A] Ascaris lumbricoides
C] Strongyloides stercoralis
B] Necator americanus
*D] B & C
55. What parasite produces Iron deficiency anemia to man?
A] Ascaris lumbricoides
C] Strongyloides stercoralis
*B] Necator americanus
D] Trichuris trichiura
56. Produces Ground itch to man.
A] Ascaris lumbricoides
*C] Necator americanus
B] Enterobius vermicularis
D] Trichuris trichiura
57. Associated with Nocturnal Pruritus Ani.
A] Ascaris lumbricoides
C] Necator americanus
*B] Enterobius vermicularis
D] Trichuris trichiura
58. The manner of transmission of the parasite is by ingestion of the embryonated egg.
A] Ascaris lumbricoides
C] Trichuris trichiura
B] Strongyloides stercoralis
*D] A & C
59. Graham Scotch tape technique is used for the diagnosis of the infection cause by
A] Ascaris lumbricoides
C] Necator americanus
*B] Enterobius vermicularis
D] Trichuris trichiura
60. What parasite has an egg that is plano-convex and is readily infective after discharged?
A] Ancylostoma duodenale
*C] Enterobius vermicularis
B] Ascaris lumbricoides
D] Trichuris trichiura
61. The parasite has cephalic alae.
A] Ancylostoma caninum
C] Necator americanus
*B] Enterobius vermicularis
D] Toxocara canis
62. Produces group infection.
*A] Enterobius vermicularis
C] Toxocara cati
B] Toxocara canis
D] B & C
63. What parasite sucks and ingests blood?
*A] Ancylostoma duodenale
C] Enterobius vermicularis
B] Ascaris lumbricoides
D] Trichuris trichiura
5
MULTIPLE CHOICE QUESTIONS: CHOOSE THE BEST ANSWER.
(0.25)
(0.25)
(0.5)
(0.5)
(0.5)
(1)
(0.25)
(1)
64. What parasite shows Tramway sign in the X-ray?
*A] Ascaris lumbricoides
C] Necator americanus
B] Enterobius vermicularis
D] Trichuris trichiura
65. What parasite whose migrating larvae break the pulmonary capillaries of man?
A] Ancylostoma braziliense
C] Enterobius vermicularis
*B] Ascaris lumbricoides
D] Trichuris trichiura
66. Associated with mosquito.
A] Brugia malayi
C] Wuchereria bancrofti
B] Loa loa
*D] A & C
67. Knott’s method is use to demonstrate the microfilariae of
A] Brugia malayi
C] Wuchereria bancrofti
B] Onchocerca volvulus
*D] A & C
68. The usual infective stage of Trematodes to man is the
A] cercariae
*C] metacercariae
B] egg
D] miracidium
69. The infective stage of the parasite is found in crabs or crayfishes.
A] Clonorchis sinensis
C] Fasciola hepatica
B] Echinostoma ilocanum
*D] Paragonimus westermani
70. What parasite is associated with neoplasm of the biliary duct or cancer of the liver?
*A] Clonorchis sinensis
C] Opisthorchis felineus
B] Fasciola hepatica
D] Paragonimus westermani
71. What parasite produces infection that simulates tuberculosis?
MICROBIOLOGY
Page 65 of 137
(0.33)
(0.33)
(0.5)
(0.5)
(0.5)
(1)
(1)
(1)
A] Clonorchis sinensis
C] Opisthorchis felineus
B] Fasciola hepatica
*D] Paragonimus westermani
72. The infective stage of what parasite is encysted in aquatic vegetations?
A] Clonorchis sinensis
C] Echinostoma ilocanum
*B] Fasciolopsis buski
D] Paragonimus westermani
73. Snails are both the first and second intermediate hosts of
*A] Echinostoma ilocanum
C] Fasciola hepatica
B] Fasciola hepatica
D] Paragonimus westermani
74. The eggs of Paragonimus westermani may be demonstrated from the
A] sputum
C] urine
B] stool
*D] A & B
75. What stage of the Trematodes swims in the water?
*A] Cercariae
C] Metacercariae
B] Coracidium
D] Sporocyst
76. What is the infective stage of Schistosoma to man?
*A] Cercariae
C] Metacercariae
B] Embryonated egg
D] Miracidium
77. What is the usual manner of transmission of Schistosoma to man?
A] Arthropod vector
*C] Skin penetration of the cercariae
B] Ingestion of the embryonated egg
D] B & C
78. Molluscan host of Schistosoma in the Philippines is
A] Australorbis mystax
C] Coxiella burnetii
B] Biomphalaria alexandrina
*D] Oncomelania quadrasi
79. What Schistosoma is present in the Philippines?
A] Schistosoma haematobium
C] Schistosoma mansoni
*B] Schistosoma japonicum
D] Schistosoma mekongi
6
MULTIPLE CHOICE QUESTIONS: CHOOSE THE BEST ANSWER.
(0.33)
(0.33)
(0.33)
(0.33)
(0.33)
(0.33)
(0.33)
(1)
(0.25)
(0.25)
80. Laboratory procedure that makes use of the egg in the test is
A] CFT
*C] COPT
B] CHR
D] Casoni test
81. What Schistosoma produces more severe infection to man?
A] Schistosoma haematobium
C] Schistosoma mansoni
*B] Schistosoma japonicum
D] Schistosoma mekongi
82. What Schistosoma is more resistant to treatment?
A] Schistosoma haematobium
C] Schistosoma mansoni
*B] Schistosoma japonicum
D] Schistosoma mekongi
83. Pipe-stem fibrosis of the liver in Schistosomiasis is due to what stage of the parasite?
A] Adult
*C] Egg
B] Cercaria
D] Metacercaria
84. Allergic manifestations in Schistosomiasis are due to what stage of the parasite?
*A] Adult
C] Egg
B] Cercaria
D] Metacercaria
85. Obstructive uropathy is associated with
*A] Schistosoma haematobium
C] Schistosoma mansoni
B] Schistosoma japonicum
D] Schistosoma mekongi
86. What Schistosoma frequently cause Gynecological Schistosomiasis.
*A] Schistosoma haematobium
C] Schistosoma mansoni
B] Schistosoma japonicum
D] Schistosoma mekongi
87. The region of growth in Cestodes is the
A] Scolex
C] Proglottid
*B] Neck
D] Strobila
88. The infective stage of Diphyllobothrium latum to man to produce adult infection is the
A] cysticercoid larva
C] procercoid larva
B] cysticercus larva
*D] sparganum larva
89. The most common cause of Sparganosis in Asia is
MICROBIOLOGY
Page 66 of 137
(0.5)
(0.5)
(0.5)
(0.33)
(0.5)
(1)
A] Diphyllobothrium latum
C] Echinococcus granulosus
B] Dipylidium caninum
*D] Spirometra mansoni
90. Bothriocephalus anemia is associated with
A] iron deficiency anemia
*C] pernicious anemia
B] microcytic hypochromic cells
D] A & B
91. Diphyllobothrium latum is associated with
A] cat
*C] fish
B] dog
D] pig
92. Produces Ocular cysticercosis to man.
A] Dipylidium caninum
*C] Taenia solium
B] Taenia saginata
D] B & C
93. Produces autoinfection in man.
A] Dipylidium caninum
C] Taenia saginata
B] Hymenolepis diminuta
*D] Taenia solium
94. Lateral uterine branches in gravid proglottid are counted for diagnosis of infection cause by
A] Diphyllobothrium latum
C] Hymenolepis nana
B] Dipylidium caninum
*D] Taenia saginata
95. Taenia saginata is associated with
A] cat
C] dog
*B] cattle
D] pig
7
MULTIPLE CHOICE QUESTIONS: CHOOSE THE BEST ANSWER.
(0.33)
(1)
(0.25)
(0.25)
(0.33)
1.
MPL
2.
MPL
3.
MPL
96. Taeniasis solium is produced after ingestion of the
A] cysticercoid larvae
*C] cysticercus cellulosae
B] cysticercus bovis
D] Taenia egg
97. What parasite is associated with measly pork?
A] Diphyllobothrium latum
C] Taenia saginata
B] Dipylidium caninum
*D] Taenia solium
98. What Cestodes does NOT need an intermediate host?
A] Dipylidium caninum
C] Hymenolepis diminuta
B] Echinococcus granulosus
*D] Hymenolepis nana
99. The most commonly involved organ in Hydatid Disease is the
*A] liver
C] kidney
B] lungs
D] brain
100. Ultrasound and MRI usually help in the diagnosis of the infection cause by
A] Dipylidium caninum
C] Hymenolepis diminuta
*B] Echinococcus granulosus
D] Hymenolepis nana
Bacteria which require organic form of carbon for growth and requires light for their source of energy are
classified as: ( Zinsser Microbiology, 19th ed., p. 54)
A. Photoorganotrophs
C. Chemoautotrophs
B. Photolautotrophs
D. Chemolithotrophs
0.25
Doubling time of Mycobacterium tuberculosis is… (Essentials of Medical Microbiology, 5 th ed., 430)
A. 1 – 2 hours
C. 10 – 12 hours
B. 5 – 10 hours
D. 15 – 20 hours
0.25
The CNM group of bacteria include the following EXCEPT: (Zinsser Microbiology, 19 th ed., p. 81)
A. Mycobacterium
C. Clostridium
B. Nocardia
D. Corynebacterium
0.5
MICROBIOLOGY
Page 67 of 137
4.
MPL
5.
MPL
6.
MPL
7.
MPL
8.
MPL
9.
MPL
10.
MPL
11.
MPL
12.
MPL
13.
The bioactive center of the lipopolysaccharide of gram-negative bacteria reside in this region: (Zinsser
Microbiology , 19th ed, p. 84)
A. I
C. III
B. II
D. IV
0.25
Serologic specificity of the LPS reside in this region: (Zinsser Microbiology, 19th ed, p. 84
A. I
C. III
B. II
D. IV
0.25
Development of spores by Bacillus anthracis occurs during this phase in its growth cycle:
A. Lag phase
C. Stationary phase
B. Log phase
D. Decline phase
0.5
This is a process of exchange of genetic information wherein a bacterial virus serves as a vector for gene
transmission: : (Zinsser Microbiology, 19th ed p. 136)
A. Transformation
C. Transfection
B. Transduction
D. Conjugation
0.5
This process is the major mechanism transfer of drug resistance which can occur between unrelated
genera: (Essentials of Medical Microbiology, 5 th ed., 297)
A. Transformation
C. Transfection
B. Transduction
D. Conjugation
0.5
The phosphoribosylribitol phosphate capsule plays the critical role in the pathogenesis of invasive disease
caused by this organism: (Zinsser Microbiology, 19th ed p. 465)
A. Streptococcus pneumoniae
C. Klebsiella pneumoniae
B. Haemophilus influenzae
D. Mycoplasma pneumoniae
0.33
Martin, 2 years old, was admitted for difficulty of breathing and actibve measles. Chest X-ray revealed the
presence of pneumonia with pleural effusion. Diagnostic tap revealed purulent material which was cultured
in BAP. Colonies showed β-hemolytic colonies and Gram staining showed gram-positive cocci in clusters.
Catalase test was positive. What is the most likely causative agent? (Zinsser Microbioology, 19th ed p. 402
and 412)
A. Streptococcus pneumoniae
C. Haemophilus influenzae
B. Klebsiella pneumoniae
D. Staphylococcus aureus
1.00
Yinyin, 24 years old, was admitted because of diarrhea after eating oysters. The organism isolated is a
gram-negative bacillus which grew best at pH of 7.6 to 9.0. It is oxidase positive. What is the most likely
agent? (Zinsser Microbiology, 19th ed p.570)
A. Shigella dysenteriae
C. Escherichia coli
B. Vibrio parahemolyticus
D. Salmonella typhi
1.00
Sophia, 45 years old, consulted because of multiple nodular lesions on her face earlobes and wrists. Skin
slit smear of the nodule on the earlobe showed numerous acid fast bacilli (+++++). What is the most likely
classification of this patient? (Zinsser Microbiology, 19th ed p.518)
A. Lepromatous leprosy
C. Borderline leprosy
B. Indeterminate leprosy
D. Tuberculoid leprosy
1.00
Diana, 2 weeks old, was admitted because of seizure. CSF examination showed pleocytosis, increased
protein and decreased sugar. The colonies isolated in BAP were β-hemolytic. (Zinsser Microbiology, 19th
ed. p 426 – 427)
A. Streptococcus pneumoniae
C. Streptococcus agalactiae
MICROBIOLOGY
Page 68 of 137
MPL
14.
MPL
14.
MPL
15.
MPL
B. Staphylococcyus aureus
D. Staphylococcus epidermidis
0.5
Fred, 39 years old, consulted because of chronic cough of one month. Sputum culture was requested. The
colonies were first observed after two weeks of incubation. The colonies were described as rough, dry and
buff colored. What is the most likely causative agent? (Zinsser Microbiology, 19 th ed, p. 499
A. Mycobacterium tuberculosis
C. Mycoplasma pneumoniae
B. Streptococcus pneumoniae
D. Haemophilus influenzae
0.5
Jodie, 10 years old, was admitted with admitting diagnosis of bacterial endocarditis. Blood culture in BAP
grew colonies with partial hemolysis. Gram staining of the growth revealed gram-positive cocci. Optochin
and bile solubility tests were negative. What is the most likely causative agent? (Jawetz Medical
Microbiology, 23th ed, p. 236)
A. Staphylococcus aureus
C. Viridans streptococcus
B. Streptococcus pneumoniae
D. Staphylococcus epidermidis
1.00
Charlie, 21-year old male, complaint of dysuria. Urine culture was done on MacConkey showing colorless
colonies. Biochemical test revealed K/A with H2S on TSI, urease and phenylalanine deaminase tests were
positive. What is the most likely causative agent? (Jawetz Medical Microbiology, 23th ed, p. 249)
A. Yersinia enterocolitica
C. Escherichia coli
B. Pseudomoans aeruginosa
D. Proteus mirabilis
1.00
16.
Cholo, 27 years old engineer, was diagnoed with rickettsial infection. His serum sample was for Weil-Felix test
which turned out to be positive for antibodies to Proteus OK-19 and OX-2. The serum sample was also
inoculated intraperitoneally to a male guinea pig producing scrotal swelling. What is the most likely causative
agent? (Essentials of Medical Microbiology, 5th ed., 463)
A. Rickettsia typhi
C. Rickettsia tsutsugamushi
B. Rickettsia prowazeki
D. Rickettsia ricketsii
MPL
0.25
16.
MPL
17.
MPL
18.
MPL
19.
MPL
20.
MPL
This organism is obligately intracellular, susceptible to sulfonamide and produces inclusion bodies that
contain glycogen in tissue culture: (Jawetz Medical Microbiology, 23 th ed, p. 359)
A. Chlamydia trachomatis
C. Ureaplasma urealyticum
B. Mycoplasma hominis
D. Coxiella burnetti
1.00
This gram-negative, non-motile, non-lactose fermenting bacillus produces toxin which is neurotoxic,
cytotoxic and enterotoxic: (Zinsser Microbiology, 19 th ed, p. 557-558)
A. Salmonella typhi
C. Escherichia coli
B. Campylobacter jenuni
D. Shigella dysenteriae
0.5
The following characteristics differentiate the el tor biotype of Vibrio cholera from the classic biotype
EXCEPT: (Zinsser Microbiology, 19th ed, p. 567)
A. Positive VP test
C. Agglutination of chicken erythrocytes
B. Positive oxidase test
D. Resistance to polymyxin B
0.25
This gram-negative bacillus grows well in moist environment, produces blue phenazine pigment, obtain
energy from carbohydrates using the oxidative rather than the fermentative pathway: (Zinsser Microbiology,
19th ed, p. 577)
A. Klebsiella pneumoniae
C. Pseudomonas aeruginosa
B. Proteus vulgaris
D. Escherichia coli
1.00
This gram-negative, facultatively intracellular bacillus is marked by bipolar staining and produces V and W
antigens which correlate with its ability to proliferate rapidly and cause overwhelming infection: (Zinsser
Microbiology, 19th ed p. 585)
A. Yersinia pestis
C. Calymmatobacterium granulomatis
B. Bacillus anthracis
D. Pasteurella multocida
0.25
MICROBIOLOGY
Page 69 of 137
21.
MPL
22.
MPL
23.
MPL
24.
MPL
25.
MPL
26.
MPL
27.
MPL
The toxin of this gram-positive obligate anaerobic organism acts on the cholinergic nerve endings to block
the release of acetylcholine: (Zinsser Microbiology, 19th ed p. 650)
A. Clostridium botulinum
C. Corynebacterium diphtheriae
B. Clostridium tetani
D. Bacillus anthracis
0.5
Ryza, 6 years old, female, was diagnosed with meningitis. CSF specimen was cultured in CAP at 5-10%
CO2. The colonies that grew were gram stained which revealed gram-negative cocci. What is the most likely
causative agent of the meningitis? (Zinsser Microbiology, 19th ed p. 444 and 447)
A. Haemophilus influenzae
C. Neisseria meningitidis
B. Streptococcus pneumoniae
D. Streptococcus agalactiae
0.5
Jhona, 5 years old, was brought for consultation because of sore throat. PE showed enlarged, exudative
tonsillitis with large palpable, tender, cervical lymph nodes. A smear from the tonsils was gram stained
showing gram-positive, clubbed-shaped bacilli which lie parallel or form angles with each other. What is the
most likely causative agent? ( Zinsser Microbiology, 19th ed p. 487 and 492)
A. Clostridium tetani
C. Corynebacterium diphtheriae
B. Bacillus anthracis
D. Coxiella burnetti
1.00
Cathy, 28 years old, consulted because of dysuria. Urine was cultured on BAP. The growth was Gram
stained showing gram-positive cocci in clusters. What is the most likely organism? ( Zinsser Microbiology,
19th ed p. 401 and 414)
A. Staphylococcus saprophyticus C. Proteus mirabilis
B. Pseudomonas aeruginosa
D. Streptococcus agalactiae
0.5
Marie, 3 years old, was admitted with a diagnosis of acute glomerulonephritis. Review of history revealed
that she had impetigo on the buttocks. What M-protein type of Streptococcus pyogenes is the most likely
cause? (Zinsser Microbiology, 19th ed p. 424)
A. M4
C. M49
B. M12
D. M57
0.25
This organism grows in 6.5% NaCl, 40% bile, and can hydrolyze esculin and is resistant to penicillin:
(Zinsser Microbiology, 19th ed, p. 427-428)
A. Streptococcus bovis
C. Enterococcus fecalis
B. Streptococcus pneumoniae
D. Staphylococcus epidermidis
0.25
Sputum culture of a patient diagnosed with pneumonia showed partial hemolysis on BAP. Biochemical tests
showed the colonies which are bile-soluble. Subculture of growth in disc containing ethylhydroxycupreine
HCl showed inhibition of growth. What is the most likely causative agent? (Jawetz Medical Microbiology,
23th ed, p. 237).
A. Streptococcus pneumoniae
C. Streptococcus viridans
B. Klebsiella pneumonaie
D. Haemophilus influenzae
0.5
30.
What is the most useful and rapid method for the identification of Streptococcus pneumoniae?
Microbiology, 19th ed, p. 434)
A. Bacitracin test
C. CAMP test
B. Neufeld Quellung test
D. VP test
MPL
0.5
(Zinsser
MICROBIOLOGY
Page 70 of 137
31.
MPL
The capsule contributes to the invasive property of the following organisms EXCEPT one:
Microbiology, 19th ed, p. 447, 464, 549)
A. Neisseria menigitidis
C. Hemophilus influenzae
B. Klebsiella pneumoniae
D. Corynebacterium diphtheriae
1.00
(Zinsser
32.
The presence of this antigen is one of the factors that can explain the virulence of Escherichia coli in neonatal
disease: (Zinsser Microbiology, 19th ed, p. 545)
A. Vi antigen
C. O antigen
B. H antigen
D. K1 antigen
MPL
0.33
33.
MPL
34.
MPL
Edgar, 9 years old boy, was admitted because of fever of 5 days duration with accompanying headache,
abdominal pain, and vomiting. Enteric fever was entertained. How are you going to confirm the diagnosis?
(Zinsser Microbiology, 19th ed, p. 563)
A. Blood culture
C. Polymerase chain reaction
B. Stool culture
D. Serology
1.00
What is considered the most sensitive method to the diagnosis of whooping cough? Jawetz Medical
Microbiology, 23th ed, p. 283.
A. Direct fluorescent antibody test C. Culture isolation
B. Polymerase chain reaction
D. Serology
0.5
35.
The exotoxin of Pseudomonas aeruginosa and Corynebacterium diphtheriae have the same mechanism of
action: (Zinsser Microbiology, 19th ed, p. 578)
A. Inhibition of protein synthesis by blocking EF-2
B. Activation of intracellular adenyl cyclase
C. Inhibition of the release of excitatory neurotransmitter
D. Activation of extracellular guanyl cyclase
MPL
0.5
36. This highly specific laboratory test for syphilis is easy to perform, inexpensive and
sensitive: (Zinsser Microbiology, 19th ed, p. 664)
A. IgG-FTA-ABS
C. MHA-TP
B. TPI
D. VDRL
MPL
1.00
37.
MPL
38.
MPL
39.
MPL
40.
MPL
Chester, 17 years old, was brought for consultation for productive cough of three weeks duration. On PE, she
was afebrile, not in CR distress, with coarse and fine rales appreciated on both lung fields. This is most
probably caused by this gram-negative, branched, filamentous organism that can induce the development of
hemagglutinins that will agglutinate human type-O erythrocytes when incubated at 0oC to 4oC, but not at
37oC: (Zinsser Microbiology, 19th ed, p. 735)
A. Streptococcus pneumoniae
C. Chlamydia pneumoniae
B. Klebsiella pneumoniae
D. Mycoplasma pneumoniae
0.25
Laryngeal papilloma is commonly associated with this human papillomavirus serotypes: (Zinsser Microbiology,
19th ed, p. 976)
A. 6 and 11
C. 45 and 52
B. 16 and 18
D. 62 and 65
0.5
What is the most common cause of rhinitis is all age group?
A. Adenovirus
C. Parainfluenza virus
B. Rhinovirus
D. Respiratory syncytial virus
1.00
Among these viruses, which one has the following characteristics: ssRNA,
enveloped virus with nonsegmented, positive-sense genome? (Zinsser Microbiology, 19th ed, p. 761-762)
A. Respiratory syncytial virus
C. Coronavirus
B. Measles virus
D. Influenza virus
0.25
MICROBIOLOGY
Page 71 of 137
41.
MPL
42.
MPL
43.
MPL
44.
MPL
45.
MPL
46.
MPL
47.
MPL
48.
MPL
49.
MPL
50.
MPL
51.
MPL
The human immunodeficiency virus is described with the following characteristics: (Zinsser Microbiology,
19th ed, p. 761-762)
A. ssRNA, enveloped virus with segmented, positive-sense genome
B. ssRNA, enveloped virus with non-segmented, positive-sense genome
C. ssRNA, enveloped virus with segmented, negative-sense genome
D. ssRNA, enveloped virus with non-segmented, negative-sense genome
0.25
The Rhabdoviruses are described with the following characteristics: (Zinsser Microbiology, 19 th ed, p. 761-762)
A. ssRNA, enveloped virus with non-segmented, positive-sense genome
B. ssRNA, naked virus with non-segmented, positive-sense genome
C. ssRNA, enveloped virus with non-segmented, negative-sense genome
D. ssRNA, naked virus with non-segmented, negative-sense genome
0.25
The picornaviruses are described with the following characteristics: Zinsser Microbiology, 19 th ed, p. 761-762)
A. ssRNA, enveloped viruses with non-segmented, positive-sense genome
B. ssRNA, naked viruses with non-segmented, positive-sense genome
C. ssRNA, enveloped viruses with non-segmented, positive-sense genome
D. ssRNA, naked viruses with non-segmented, positive-sense genome
0.25
The hepatitis C virus is described as follows: (Zinsser Microbiology, 19 th ed, p. 761-762 and 1040)
A. ssRNA, enveloped viruses with non-segmented, positive-sense genome
B. ssRNA, naked viruses with non-segmented, positive-sense genome
C. ssRNA, enveloped viruses with non-segmented, positive-sense genome
D. ssRNA, naked viruses with non-segmented, positive-sense genome
0.25
Which of these markers of hepatitis B infection correlates with the absence of HBcAg and DNA polymerase,
less infectivity and a better prognosis for chronic hepatitis. (Zinsser Microbiology, 19 th ed, p. 1043)
A. Anti-HBc
C. Anti-HBe
B. Anti-HBd
D. Anti-HBs
0.33
Which of these markers of HIV indicates active viral replication?
A. p24
C. gp41
B. p55
D. gp120
0.5
Which of the following is the only filamentous mammalian virus? Zinsser, 19 th ed, p. 761-762 and 1031
A. Ebola
C. Japanese B encephalitis
B. Dengue
D. Yellow fever
0.25
This virus family is described with the following characteristics: double-stranded nucleic acid, naked with
segmented genome: (Zinsser Microbiology, 19th ed, p. 761-762)
A. Picornaviridae
C. Coronaviridae
B. Reoviridae
D. Rhabdoviridae
1.00
This virus shows tropism for the B lymphocytes and certain epithelial cells. It produces nonproductive
infection and transformation of B cells: (Essentials of Medical Microbiology by Volk et al, 5th ed., p. 553-554
)
A. Human herpes 1 and 2
C. Human herpes virus 4
B. Human herpes virus 3
D. Human herpes virus 5
0.25
This virus undergoes antigenic shift producing major changes in the H and N glycoproteins: (Zinsser
Microbiology, 19th ed, p. 994)
A. Influenza A virus
C. Parainfluenza virus
B. Respiratory syncytial virus
D. Calicivirus
0.5
This fungus occurs as yeast in tissues of infected patients and when cultured at room temperature occurs as
mold form: (Zinsser Microbiology, 19th ed, p.1072)
A. Trichophyton mentagrophytes
C. Sporothrix schenckii
B. Cladosporium werneckii
D. Epidermophyton floccosum
0.5
MICROBIOLOGY
Page 72 of 137
52.
MPL
53.
MPL
54.
MPL
55.
MPL
56.
MPL
57.
MPL
58.
MPL
57.
MPL
58.
MPL
61.
MPL
Mark, 24-year old male, consulted because of hypopigmented macules on his chest, upper back, shoulders
and arms. Scrapings of the lesions were taken for microscopic examination. Short, unbranched hyphae and
numerous spherical cells were seen. What is the most likely causative agent? Zinsser Microbiology, 19 th ed,
p. 1131-1132
A. Phialophora verrucosa
C. Aspergillus fumigatus
B. Malassezia furfur
D. Trichosporon beigellii
1.00
Rapid presumptive test for Candida albicans is… Zinsser Microbiology, 19 th ed, p. 1142
A. Grape-like clustering of microconidia
C. Multiple budding yeast cell
B. Germ tube production
D. Sclerotic bodies
1.00
What is the most common cause of athlete’s foot? (Zinsser Microbiology, 19 th ed, p. 1130)
A. Trichophyton mentagrophytes
C. Microsporum canis
B. Epidermophyton floccosum
D. Microsporum gypseum
.25
Multiple budding yeast cell is characteristic of this fungus: Zinsser Microbiology, 19 th ed, p. 1109)
A. Coccidioides immitis
C. Sporothrix schenckii
B. Histoplasma capsulatum
D. Paracoccidioides braziliensis
1.00
Charlie, 57 years old male, was admitted because of meningitis. India ink staining of the CSF showed large,
encapsulated yeast cells. What is the most likely causative agent? Zinsser Microbiology, 19th ed, p. 1144)
A. Cryptococcus neoformans
C. Coccidioides immitis
B. Candida albicans
D. Histoplasma capsulatum
1.00
What is the diagnostic feature of chromoblastomycosis? Zinsser Microbiology, 19 th ed, p. 1118
A. Spherule
C. Multiple buds
B. Sclerotic bodies
D. Rosetting of microconidia
0.5
What is the infectious particle of Coccidioides immitis? Zinsser Microbiology, 19 th ed, p. 1092)
A. Arthroconidium
C. Spherule
B. Chlamydospore
D. Macroconidia
1.00
The defining feature of this fungal infection is the presence of local swelling and interconnecting sinuses
containing sulfur granules: Jawetz Medical Microbiology, 23 rd ed, p. 633.
A. Lobo’s disease
C. Darling’s disease
B. Mycetoma
D. Sporothrichosis
0.25
Mang Tomas, 56 year old gardener, came in with multiple subcutaneous nodules and abscesses along the
lymphatics: Jawetz Medical Microbiology, 23 rd ed, p. 636.
A. Sporothrix schenckii
C. Trichophyton tonsurans
B. Epidermophyton floccosum
D. Pseudoallescheria boydii
1.00
One of the following is a type IV hypersensitivity reaction: Zinsser Microbiology, 19 th ed, p. 323
A. Bronchial asthma
B. Contact dermatitis
C. ABO incompatibility
D. Post-streptococcal glomerulonephritis
1.00
62.
The following are mediators of non-specific cellular immunity EXCEPT
A. T and B cells
C. Macrophages
B. Polymorphonuclear leukocytes
D. NK cells
MPL
0.5
63.
Specificity of the immunoglobulin molecule is determined by these components:
A. Variable and constant regions of the light chains
MICROBIOLOGY
Page 73 of 137
MPL
64.
MPL
65.
MPL
B.
C.
D.
-
Variable regions of both the light and the heavy chains
Constant regions of both the light and heavy chains
Variable and constant regions of the heavy chains
1.00
The cellular mediators of immediate response to cytosolic pathogens are the:
A. PMNs
C. NK cells
B. T and B cells
D. CRP
0.25
The predominant antibody found in breastmilk is the: ( Zinsser Microbiology, 19 th ed, p. 224)
A. IgA
C. IgG
B. IgE
D. IgM
-
1.00
66.
This antibody molecule has neutralizing, opsonizing and complement activating activity:
A. IgA
C. IgG
B. IgE
D. IgM
MPL
0.25
67.
The level of this antibody approximates that of the adult at birth:
A. IgA
C. IgG
B. IgE
D. IgM
MPL
1.00
68. These cells have dual functions in immunity: antibody secretion and antigen presentation:
A. B cells
C. Macrophages
B. NK cells
D. Dendritic cells
MPL
0.25
69.
MPL
This cytokine secreted by TH1 cells enhances the microbicidal power of the macophages:
A. TNF-α
C. IL-2
B. IFN-γ
D. GM-CSF
0.25
70.
The primary mediators of specific humoral immunity:
A. Antibodies
B. Alternate complement
MPL
1.00
71.
MPL
72.
MPL
73.
MPL
74.
C. Acute phase proteins
D. Lysozyme
What is the only nematode that requires intermediate host to complete its life cycle? (Clinical Parasitology by
Beaver et al, 9th ed., 246)
A. Capillaria philippinensis
C. Trichuris trichiura
B. Enterobius vermicualris
D. Trichinella spiralis
0.5
The infective larva of this parasite causes erythematous, serpiginous, intracutaneous tunnels which is
accompanied by intense itchiness: (Clinical Parasitology by Beaver et al, 9 th ed., 281)
A. Ancylostoma braziliense
C. Toxocara canis
B. Strongyloides stercoralis
D. Trichuris trichiura
0.25
The adult stage of this parasite is described to have attenuated anterior portion and a fleshy posterior
portion: (Clinical Parasitology by Beaver et al, 9th ed., 240)
A. Ancylostoma brazilinese
C. Necator americanus
B. Strongyloides stercoralis
D. Trichuris trichiura
1.00
All these parasites have a lung phase during their developmental cycle EXCEPT: (Clinical Parasitology by
Beaver et al, 9th ed., 259, 283, 310)
A. Ascaris lumbricoides
C. Strongyloides stercoralis
MICROBIOLOGY
Page 74 of 137
MPL
75.
MPL
76.
MPL
77.
MPL
B. Enterobius vermicularis
1.00
D.. Necator americanus
Embryonated ovum is the infective stage of this parasite: (Clinical Parasitology by Beaver et al, 9 th ed., 303)
A. Trichinella spiralis
C. Capillaria philippinensis
B. Enterobius vermicularis
D. Strongyloides stercoralis
0.5
This adult nematode is larviparous which means it produces larva rather than ova: (Clinical Parasitology by
Beaver et al, 9th ed., 232)
A. Capillaria philippinensis
C. Enterobius vermicualris
B. Trichinella spiralis
D. Trichuris trichiura
0.5
The most effective preventive measure against this parasite is thru thorough freezing or cooking of food
potentially carrying the parasite: (Clinical Parasitology by Beaver et al, 9 th ed., 240)
A. Trichuris trichiura
C. Enterobius vermicularis
B. Trichinella spiralis
D. Strongyloides stercoralis
1.00
78.
When embryonated egg of this parasite is ingested, its second-stage larva encysts in tissues such as the liver
and lungs and cause hypereosinophilia: (Clinical Parasitology by Beaver et al, 9th ed., 326-327)
A. Wuchereria bancrofti
C. Toxocara canis
B. Trichinella spiralis
D. Trichuris trichiura
MPL
1.00
79.
MPL
80.
MPL
81.
MPL
82.
MPL
83.
MPL
A stained peripheral blood film revealed the presence of this microfilaria with two discreet nuclei at the tip of
the tail: (Clinical Parasitology by Beaver et al, 9th ed., 365)
A. Brugia malayi
C. Onchocerc volvulus
B. Loa loa
D. Wuchereria bancrofti
0.5
The ova of this parasite are NOT commonly recovered in the stool: (Clinical Parasitology by Beaver et al, 9th
ed., 303)
A. Ascaris lumbricoides
C. Enterobius vermicularis
B. Trichuris trichiura
D. Hookworms
1.00
Routine examination of a stool sample was submitted for facalysis. The result revealed the presence of
spherical cyst-like material with two nuclei and one chromatoidal body with rounded ends. What is the most
probable parasite seen? (Clinical Parasitology by Beaver et al, 9 th ed., 103)
A. Balantidium coli
C. Giardia lamblia
B. Entamoeba coli
D. Entamoeba histolytica
0.5
This parasite is transmitted by ingestion of the sporulated oocyst: (Basic ClinicalParasitology by Brown et al,
6th ed., p.52)
A. Naegleria fowleri
C. Cryptosporidium parvum
B. Trichomonas vaginalis
D. Balantidium coli
0.25
Parasites which are rounded anteriorly and pointed posteriorly with a pair of nuclei containing a central
karyosome were seen in watery stool of a patient with severe diarrhea. What is the most likely causative
agent? (Clinical Parasitology by Beaver et al, 9th ed., 44)
A. Trichomonas vaginalis
C. Giardia lamblia
B. Chilomastix mesnilii
D. Endolimax nana
0.5
MICROBIOLOGY
Page 75 of 137
84.
MPL
85.
MPL
86.
MPL
87.
MPL
Nelson, 32-year old engineer. Was admitted because of on and off high grade fever with accompanying
chills of two weeks duration. Peripheral blood smear showed ring form trophozoites and crescent-shaped
gametocytes. What is the most probable causative agent? (Clinical Parasitology by Beaver et al, 9 th ed.,
188)
A. Plasmodium vivax
C. Plasmodium ovale
B. Plasmodium malariae
D. Plasmodium falciparum
1.00
The infective stage of Trypanosoma cruzi is... (Clinical Parasitology by Beaver et al, 9 th ed., 90)
A. Amastigote
C. Epimastigote
B. Promastigote
D. Trypomastigote
0.25
Kalaazar is caused by this parasite: (Clinical Parasitology by Beaver et al, 9 th ed., 71)
A. Trypanosoma cruzi
C. Leishmania donovani
B. Trypanosoma gambiense
D. Leishmania tropica
0.5
CJ, 1 year old male, had dysentery described as bloody and mucoid. What are the most probable causative
agents? (Clinical Parasitology by Beaver et al, 9th ed., 115, 216)
A. Entamoeba coli and Endolimax nana
B. Entamoeba histolytica and Balantidium coli
C. Cryptosporidium parvum and Isospora belli
D. Giardia lamblia and Trichomonas hominis
1.00
The infective stage of Plasmodium falciparum to the intermediate host is the… (Clinical Parasitology by
Beaver et al, 9th ed., 176)
A. Trophozoites
C. Merozoites
B. Sporozoites
D. Gametocytes
MPL - 0.5
88.
89.
MPL
This is the only trematode with a non-operculated ovum: (Clinical Parasitology by Beaver et al, 9 th ed., 418,
452, 459, 464)
A. Schistosoma japonicum
C. Clonorchis sinensis
B. Fasciola hepatica
D. Paragonimus westermani
1.00
90.
The metacercaria is the infective stage of the following parasites EXCEPT: (Clinical Parasitology by Beaver et
al, 9th ed., 416-417, 458, 473, 481)
A. Opistorchis felineus
C. Schistosoma japonicum
B. Metagonimus yokogawai
D. Echinostoma ilocanum
MPL
0.25
91.
MPL
Mang Pandoy, 58 years old male, had a history of four weeks cough with blood-tinged expectorate. Sputum
examination was done which revealed the presence of operculated ova. What is the most likely causative
agent? (Clinical Parasitology by Beaver et al, 9 th ed., 464)
A. Metagonimus yokogawai
C. Paragonimus westermani
B. Clonorchis sinensis
D. Echinostoma ilocanum,
1.00
92. This cestode requires two intermediate hosts for the completion of its developmental cycle: (Clinical Parasitology
by Beaver et al, 9th ed., 496)
A. Taenia saginata
C. Echinococcus granulosus
B. Diphyllobothrium latum
D. Dipylidium caninum
MPL
0.25
93
MPL
Sparganum larva is the infective stage of this parasite: (Clinical Parasitology by Beaver et al, 9 th ed., 496)
A. Diphyllobotrium latum
C. Dipylidium caninum
B. Taenia solium
D. Hymenolepis nana
1.00
MICROBIOLOGY
Page 76 of 137
94
MPL
95
MPL
96.
MPL
97.
MPL
98.
MPL
99.
MPL
These parasites are transmitted by eating improperly cooked pork EXCEPT: (Clinical Parasitology by
Beaver et al, 9th ed., 167, 240, 519, 533)
A. Taenia solium
C. Trichinella spiralis
B. Echinococcus granulosus
D. Toxoplasma gondii
0.5
This parasite utilizes the snail as its first and second intermediate hosts to complete its developmental cycle:
(Clinical Parasitology by Beaver et al, 9th ed., 458)
A. Metagonimus yokogawai
C. Paragonimus westermani
B. Clonorchis sinensis
D. Echinostoma ilocanum,
0.25
This parasite is described as ribbon-like, scolex with 4 cup-like suckers and rostellum with double row of
large and small hooks: (Clinical Parasitology by Beaver et al, 9 th ed., 513)
A. Taenia solium
C. Hymenolepis nana
B. Taenia saginata
D. Hymenolepis diminuta
0.5
This parasite is referred to as the dwarf tapeworm: (Basic Clinical Parasitology by Brown, 6 th ed, p. 191)
A. Hymenolepis nana
C. Echinococcus granulosus
B. Hymenolepid diminuta
D. Dipylidium caninum
1.00
Which of the following cestodes does NOT require an intermediate host to complete its life cycle?
Clinical Parasitology by Brown, 6th ed, p. 192)
A. Hymenolepis nana
C. Echinococcus granulosus
B. Hymenolepid diminuta
D. Dipylidium caninum
0.25
Proper cooking of food is the most important preventive measure against this condition:
Parasitology by Beaver et al, 9th ed., 247, 261, 425)
A. Schistosomiasis
C. Capillariasis
B. Cochin china diarrhea
D. Malaria
0.5
(Basic
(Clinical
100. Man becomes infected by this parasite by ingestion of cysticercoid larva: (Clinical Parasitology by Beaver et al,
9th ed., 508)
A. Taenia solium
C. Diphyllobothrium latum
B. Dipylidium caninum
D. Metagnonimus yokogawai
MPL
0.5
Bacteria which require organic form of carbon for growth and requires light for their source of energy are classified as:
( Zinsser Microbiology, 19th ed., p. 54)
A. Photoorganotrophs
C. Chemoautotrophs
B. Photolautotrophs
D. Chemolithotrophs
2.
Doubling time of Mycobacterium tuberculosis is… (Essentials of Medical Microbiology, 5 th ed., 430)
A. 1 - 2 hours
C. 10 - 12 hours
B. 5 - 10 hours
D. 15 - 20 hours
3.
The CNM group of bacteria include the following EXCEPT: (Zinsser Microbiology, 19th ed., p. 81)
A. Mycobacterium
C. Clostridium
B. Nocardia
D. Corynebacterium
4.
The bioactive center of the lipopolysaccharide of gram-negative bacteria reside in this region: (Zinsser
Microbiology , 19th ed, p. 84)
A. I
C. III
B. II
D. IV
5.
Serologic specificity of the LPS reside in this region: (Zinsser Microbiology, 19th ed, p. 84
A. I
C. III
MICROBIOLOGY
Page 77 of 137
B. II
D. IV
Development of spores by Bacillus anthracis occurs during this phase in its growth cycle:
A. Lag phase
C. Stationary phase
B. Log phase
D. Decline phase
This is a process of exchange of genetic information wherein a bacterial virus serves as a vector for gene
transmission: : (Zinsser Microbiology, 19th ed p. 136)
A. Transformation
C. Transfection
B. Transduction
D. Conjugation
8.
This process is the major mechanism transfer of drug resistance which can occur between unrelated
genera: (Essentials of Medical Microbiology, 5 th ed., 297)
A. Transformation
C. Transfection
B. Transduction
D. Conjugation
The phosphoribosylribitol phosphate capsule plays the critical role in the pathogenesis of invasive disease caused by
this organism: (Zinsser Microbiology, 19th ed p. 465)
A. Streptococcus pneumoniae
C. Klebsiella pneumoniae
B. Haemophilus influenzae
D. Mycoplasma pneumoniae
Martin, 2 years old, was admitted for difficulty of breathing and actibve measles. Chest X-ray revealed the presence
of pneumonia with pleural effusion. Diagnostic tap revealed purulent material which was cultured in BAP.
Colonies showed ß-hemolytic colonies and Gram staining showed gram-positive cocci in clusters. Catalase
test was positive. What is the most likely causative agent? (Zinsser Microbioology, 19th ed p. 402 and 412)
A. Streptococcus pneumoniae
C. Haemophilus influenzae
B. Klebsiella pneumoniae
D. Staphylococcus aureus
Yinyin, 24 years old, was admitted because of diarrhea after eating oysters. The organism isolated is a gram-negative
bacillus which grew best at pH of 7.6 to 9.0. It is oxidase positive. What is the most likely agent? (Zinsser
Microbiology, 19th ed p.570)
A. Shigella dysenteriae
C. Escherichia coli
B. Vibrio parahemolyticus
D. Salmonella typhi
Sophia, 45 years old, consulted because of multiple nodular lesions on her face earlobes and wrists. Skin slit smear
of the nodule on the earlobe showed numerous acid fast bacilli (+++++). What is the most likely classification
of this patient? (Zinsser Microbiology, 19th ed p.518)
A. Lepromatous leprosy
C. Borderline leprosy
B. Indeterminate leprosy
D. Tuberculoid leprosy
Diana, 2 weeks old, was admitted because of seizure. CSF examination showed pleocytosis, increased protein and
decreased sugar. The colonies isolated in BAP were ß-hemolytic. (Zinsser Microbiology, 19th ed. p 426 –
427)
A. Streptococcus pneumoniae
C. Streptococcus agalactiae
B. Staphylococcyus aureus
D. Staphylococcus epidermidis
14.
Fred, 39 years old, consulted because of chronic cough of one month. Sputum culture was requested. The
colonies were first observed after two weeks of incubation. The colonies were described as rough, dry and
buff colored. What is the most likely causative agent? (Zinsser Microbiology, 19 th ed, p. 499
A. Mycobacterium tuberculosis
C. Mycoplasma pneumoniae
B. Streptococcus pneumoniae
D. Haemophilus influenzae
Jodie, 10 years old, was admitted with admitting diagnosis of bacterial endocarditis. Blood culture in BAP grew
colonies with partial hemolysis. Gram staining of the growth revealed gram-positive cocci. Optochin and bile
solubility tests were negative. What is the most likely causative agent? (Jawetz Medical Microbiology, 23 th
ed, p. 236)
A. Staphylococcus aureus
C. Viridans streptococcus
B. Streptococcus pneumoniae
D. Staphylococcus epidermidis
MICROBIOLOGY
Page 78 of 137
Charlie, 21-year old male, complaint of dysuria. Urine culture was done on MacConkey showing colorless colonies.
Biochemical test revealed K/A with H2S on TSI, urease and phenylalanine deaminase tests were positive.
What is the most likely causative agent? (Jawetz Medical Microbiology, 23 th ed, p. 249)
A. Yersinia enterocolitica
C. Escherichia coli
B. Pseudomoans aeruginosa
D. Proteus mirabilis
16.
Cholo, 27 years old engineer, was diagnoed with rickettsial infection. His serum sample was for Weil-Felix test
which turned out to be positive for antibodies to Proteus OK-19 and OX-2. The serum sample was also
inoculated intraperitoneally to a male guinea pig producing scrotal swelling. What is the most likely causative
agent? (Essentials of Medical Microbiology, 5 th ed., 463)
A. Rickettsia typhi
C. Rickettsia tsutsugamushi
B. Rickettsia prowazeki
D. Rickettsia ricketsii
This organism is obligately intracellular, susceptible to sulfonamide and produces inclusion bodies that contain
glycogen in tissue culture: J(awetz Medical Microbiology, 23 th ed, p. 359)
A. Chlamydia trachomatis
C. Ureaplasma urealyticum
B. Mycoplasma hominis
D. Coxiella burnetti
This gram-negative, non-motile, non-lactose fermenting bacillus produces toxin which is neurotoxic, cytotoxic and
enterotoxic: (Zinsser Microbiology, 19th ed, p. 557-558)
A. Salmonella typhi
C. Escherichia coli
B. Campylobacter jenuni
D. Shigella dysenteriae
The following characteristics differentiate the el tor biotype of Vibrio cholera from the classic biotype EXCEPT:
(Zinsser Microbiology, 19th ed, p. 567)
A. Positive VP test
C. Agglutination of chicken erythrocytes
B. Positive oxidase test
D. Resistance to polymyxin B
This gram-negative bacillus grows well in moist environment, produces blue phenazine pigment, obtain energy from
carbohydrates using the oxidative rather than the fermentative pathway: (Zinsser Microbiology, 19th ed, p.
577)
A. Klebsiella pneumoniae
C. Pseudomonas aeruginosa
B. Proteus vulgaris
D. Escherichia coli
This gram-negative, facultatively intracellular bacillus is marked by bipolar staining and produces V and W antigens
which correlate with its ability to proliferate rapidly and cause overwhelming infection: (Zinsser Microbiology,
19th ed p. 585)
A. Yersinia pestis
C. Calymmatobacterium granulomatis
B. Bacillus anthracis
D. Pasteurella multocida
The toxin of this gram-positive obligate anaerobic organism acts on the cholinergic nerve endings to block the release
of acetylcholine: (Zinsser Microbiology, 19th ed p. 650)
A. Clostridium botulinum
C. Corynebacterium diphtheriae
B. Clostridium tetani
D. Bacillus anthracis
Ryza, 6 years old, female, was diagnosed with meningitis. CSF specimen was cultured in CAP at 5-10% CO2. The
colonies that grew were gram stained which revealed gram-negative cocci. What is the most liley causative
agent of the meningitis? (Zinsser Microbiology, 19th ed p. 444 and 447)
A. Haemophilus influenzae
C. Neisseria meningitidis
B. Streptococcus pneumoniae
D. Streptococcus agalactiae
Jhona, 5 years old, was brought for consultation because of sore throat. PE showed enlarged, exudative tonsillitis
with large palpable, tender, cervical lymph nodes. A smear from the tonsils was gram stained showing
MICROBIOLOGY
Page 79 of 137
gram-positive, clubbed-shaped bacilli which lie parallel or form angles with each other. What is the most
likely causative agent? ( Zinsser Microbiology, 19th ed p. 487 and 492)
A. Clostridium tetani
C. Corynebacterium diphtheriae
B. Bacillus anthracis
D. Coxiella burnetti
Cathy, 28 years old, consulted because of dysuria. Urine was cultured on BAP. The growth was Gram stained
showing gram-positive cocci in clusters. What is the most likely organism? ( Zinsser Microbiology, 19th ed p.
401 and 414)
A. Staphylococcus saprophyticus C. Proteus mirabilis
B. Pseudomonas aeruginosa
D. Streptococcus agalactiae
Marie, 3 years old, was admitted with a diagnosis of acute glomerulonephritis. Review of history revealed that she
had impetigo on the buttocks. What M-protein type of Streptococcus pyogenes is the most likely cause?
(Zinsser Microbiology, 19th ed p. 424)
A. M4
C. M49
B. M12
D. M57
This organism grows in 6.5% NaCl, 40% bile, and can hydrolyze esculin and is resistant to penicillin: (Zinsser
Microbiology, 19th ed, p. 427-428)
A. Streptococcus bovis
C. Enterococcus fecalis
B. Streptococcus pneumoniae
D. Staphylococcus epidermidis
Sputum culture of a patient diagnosed with pneumonia showed partial hemolysis on BAP. Biochemical tests showed
the colonies which are bile-soluble. Subculture of growth in disc containing ethylhydroxycupreine HCl
showed inhibition of growth. What is the most likely causative agent? (Jawetz Medical Microbiology, 23 th ed,
p. 237).
A. Streptococcus pneumoniae
C. Streptococcus viridans
B. Klebsiella pneumonaie
D. Haemophilus influenzae
30.
What is the most useful and rapid method for the identification of Streptococcus pneumoniae? (Zinsser
Microbiology, 19th ed, p. 434)
A. Bacitracin test
C. CAMP test
B. Neufeld Quellung test
D. VP test
31. The capsule contributes to the invasive property of the following organisms EXCEPT one: (Zinsser
Microbiology, 19th ed, p. 447, 464, 549)
A. Neisseria menigitidis
C. Hemophilus influenzae
B. Klebsiella pneumoniae
D. Corynebacterium diphtheriae
32.
The presence of this antigen is one of the factors that can explain the virulence of Escherichia coli in neonatal
disease: (Zinsser Microbiology, 19th ed, p. 545)
A. Vi antigen
C. O antigen
B. H antigen
D. K1 antigen
Edgar, 9 years old boy, was admitted because of fever of 5 days duration with accompanying headache, abdominal
pain, and vomiting. Enteric fever was entertained. How are you going to confirm the diagnosis? (Zinsser
Microbiology, 19th ed, p. 563)
A. Blood culture
C. Polymerase chain reaction
B. Stool culture
D. Serology
What is considered the most sensitive method to the diagnosis of whooping cough? Jawetz Medical Microbiology,
23th ed, p. 283.
A. Direct fluorescent antibody test C. Culture isolation
B. Polymerase chain reaction
D. Serology
The exotoxin of Pseudomonas aeruginosa and Corynebacterium diphtheriae have the same mechanism of action:
(Zinsser Microbiology, 19th ed, p. 578)
Inhibition of protein synthesis by blocking EF-2
Activation of intracellular adenyl cyclase
Inhibition of the release of excitatory neurotransmitter
Activation of extracellular guanyl cyclase
MICROBIOLOGY
Page 80 of 137
36.
37.
38.
39.
40.
This highly specific laboratory test for syphilis is easy to perform, inexpensive and
sensitive: (Zinsser Microbiology, 19th ed, p. 664)
A. IgG-FTA-ABS
C. MHA-TP
B. TPI
D. VDRL
Chester, 17 years old, was brought for consultation for productive cough of three weeks duration. On PE, she
was afebrile, not in CR distress, with coarse and fine rales appreciated on both lung fields. This is most
probably caused by this gram-negative, branched, filamentous organism that can induce the development of
hemagglutinins that will agglutinate human type-O erythrocytes when incubated at 0oC to 4oC, but not at
37oC: (Zinsser Microbiology, 19th ed, p. 735)
A. Streptococcus pneumoniae
C. Chlamydia pneumoniae
B. Klebsiella pneumoniae
D. Mycoplasma pneumoniae
Laryngeal papilloma is commonly associated with this human papillomavirus serotypes: (Zinsser Microbiology,
19th ed, p. 976)
A. 6 and 11
C. 45 and 52
B. 16 and 18
D. 62 and 65
What is the most common cause of rhinitis is all age group?
A. Adenovirus
C. Parainfluenza virus
B. Rhinovirus
D. Respiratory syncytial virus
Among these viruses, which one has the following characteristics: ssRNA, enveloped virus with nonsegmented, positive-sense genome? (Zinsser Microbiology, 19th ed, p. 761-762)
A. Respiratory syncytial virus
C. Coronavirus
B. Measles virus
D. Influenza virus
The human immunodeficiency virus is described with the following characteristics: (Zinsser Microbiology, 19 th ed, p.
761-762)
A. ssRNA, enveloped virus with segmented, positive-sense genome
B. ssRNA, enveloped virus with non-segmented, positive-sense genome
C. ssRNA, enveloped virus with segmented, negative-sense genome
D. ssRNA, enveloped virus with non-segmented, negative-sense genome
42.
The Rhabdoviruses are described with the following characteristics: (Zinsser Microbiology, 19 th ed, p. 761-762)
A. ssRNA, enveloped virus with non-segmented, positive-sense genome
B. ssRNA, naked virus with non-segmented, positive-sense genome
C. ssRNA, enveloped virus with non-segmented, negative-sense genome
D. ssRNA, naked virus with non-segmented, negative-sense genome
43.
The picornaviruses are described with the following characteristics: Zinsser Microbiology, 19 th ed, p. 761-762)
A. ssRNA, enveloped viruses with non-segmented, positive-sense genome
B. ssRNA, naked viruses with non-segmented, positive-sense genome
C. ssRNA, enveloped viruses with non-segmented, positive-sense genome
D. ssRNA, naked viruses with non-segmented, positive-sense genome
44. The hepatitis C virus is described as follows: (Zinsser Microbiology, 19 th ed, p. 761-762 and 1040)
A. ssRNA, enveloped viruses with non-segmented, positive-sense genome
B. ssRNA, naked viruses with non-segmented, positive-sense genome
C. ssRNA, enveloped viruses with non-segmented, positive-sense genome
D. ssRNA, naked viruses with non-segmented, positive-sense genome
45.
Which of these markers of hepatitis B infection correlates with the absence of HBcAg and DNA polymerase,
less infectivity and a better prognosis for chronic hepatitis. (Zinsser Microbiology, 19 th ed, p. 1043)
A. Anti-HBc
C. Anti-HBe
B. Anti-HBd
D. Anti-HBs
46.
Which of these markers of HIV indicates active viral replication?
A. p24
C. gp41
B. p55
D. gp120
47.
Which of the following is the only filamentous mammalian virus? Zinsser, 19 th ed, p. 761-762 and 1031
A. Ebola
C. Japanese B encephalitis
B. Dengue
D. Yellow fever
48.
This virus family is described with the following characteristics: double-stranded nucleic acid, naked with
segmented genome: (Zinsser Microbiology, 19th ed, p. 761-762)
A. Picornaviridae
C. Coronaviridae
B. Reoviridae
D. Rhabdoviridae
49.
This virus shows tropism for the B lymphocytes and certain epithelial cells. It produces nonproductive
infection and transformation of B cells: (Essentials of Medical Microbiology by Volk et al, 5th ed., p. 553-554
)
MICROBIOLOGY
Page 81 of 137
50.
51.
52.
53.
54.
55.
56.
A. Human herpes 1 and 2
C. Human herpes virus 4
B. Human herpes virus 3
D. Human herpes virus 5
This virus undergoes antigenic shift producing major changes in the H and N glycoproteins: (Zinsser
Microbiology, 19th ed, p. 994)
A. Influenza A virus
C. Parainfluenza virus
B. Respiratory syncytial virus
D. Calicivirus
This fungus occurs as yeast in tissues of infected patients and when cultured at room temperature occurs as
mold form: (Zinsser Microbiology, 19th ed, p.1072)
A. Trichophyton mentagrophytes
C. Sporothrix schenckii
B. Cladosporium werneckii
D. Epidermophyton floccosum
Mark, 24-year old male, consulted because of hypopigmented macules on his chest, upper back, shoulders
and arms. Scrapings of the lesions were taken for microscopic examination. Short, unbranched hyphae and
numerous spherical cells were seen. What is the most likely causative agent? Zinsser Microbiology, 19 th ed,
p. 1131-1132
A. Phialophora verrucosa
C. Aspergillus fumigatus
B. Malassezia furfur
D. Trichosporon beigellii
Rapid presumptive test for Candida albicans is… Zinsser Microbiology, 19 th ed, p. 1142
A. Grape-like clustering of microconidia
C. Multiple budding yeast cell
B. Germ tube production
D. Sclerotic bodies
What is the most common cause of athlete's foot? (Zinsser Microbiology, 19 th ed, p. 1130)
A. Trichophyton mentagrophytes
C. Microsporum canis
B. Epidermophyton floccosum
D. Microsporum gypseum
Multiple budding yeast cell is characteristic of this fungus: Zinsser Microbiology, 19 th ed, p. 1109)
A. Coccidioides immitis
C. Sporothrix schenckii
B. Histoplasma capsulatum
D. Paracoccidioides braziliensis
Charlie, 57 years old male, was admitted because of meningitis. India ink staining of the CSF showed large,
encapsulated yeast cells. What is the most likely causative agent? Zinsser Microbiology, 19 th ed, p. 1144)
A. Cryptococcus neoformans
C. Coccidioides immitis
B. Candida albicans
D. Histoplasma capsulatum
57.
What is the diagnostic feature of chromoblastomycosis? Zinsser Microbiology, 19 th ed, p. 1118
A. Spherule
C. Multiple buds
B. Sclerotic bodies
D. Rosetting of microconidia
58.
What is the infectious particle of Coccidioides immitis? Zinsser Microbiology, 19 th ed, p. 1092)
A. Arthroconidium
C. Spherule
B. Chlamydospore
D. Macroconidia
The defining feature of this fungal infection is the presence of local swelling and interconnecting sinuses containing
sulfur granules: Jawetz Medical Microbiology, 23 rd ed, p. 633.
A. Lobo's disease
C. Darling's disease
B. Mycetoma
D. Sporothrichosis
Mang Tomas, 56 year old gardener, came in with multiple subcutaneous nodules and abscesses along the
lymphatics: Jawetz Medical Microbiology, 23 rd ed, p. 636.
A. Sporothrix schenckii
C. Trichophyton tonsurans
B. Epidermophyton floccosum
D. Pseudoallescheria boydii
61.
One of the following is a type IV hypersensitivity reaction: Zinsser Microbiology, 19 th ed, p. 323
A. Bronchial asthma
B. Contact dermatitis
C. ABO incompatibility
D. Post-streptococcal glomerulonephritis
62.
The following are mediators of non-specific cellular immunity EXCEPT
A. T and B cells
C. Macrophages
B. Polymorphonuclear leukocytes
D. NK cells
MICROBIOLOGY
Page 82 of 137
63.
Specificity of the immunoglobulin molecule is determined by these components:
Variable and constant regions of the light chains
Variable regions of both the light and the heavy chains
Constant regions of both the light and heavy chains
Variable and constant regions of the heavy chains
64.
The cellular mediators of immediate response to cytosolic pathogens are the:
A. PMNs
C. NK cells
B. T and B cells
D. CRP
The predominant antibody found in breastmilk is the: ( Zinsser Microbiology, 19 th ed, p. 224)
A. IgA
C. IgG
B. IgE
D. IgM
65.
66.
This antibody molecule has neutralizing, opsonizing and complement activating activity:
A. IgA
C. IgG
B. IgE
D. IgM
67.
The level of this antibody approximates that of the adult at birth:
A. IgA
C. IgG
B. IgE
D. IgM
68. These cells have dual functions in immunity: antibody secretion and antigen presentation:
A. B cells
C. Macrophages
B. NK cells
D. Dendritic cells
This cytokine secreted by TH1 cells enhances the microbicidal power of the macophages:
A. TNF-a
C. IL-2
B. IFN-?
D. GM-CSF
70.
The primary mediators of specific humoral immunity:
A. Antibodies
B. Alternate complement
C. Acute phase proteins
D. Lysozyme
What is the only nematode that requires intermediate host to complete its life cycle? (Clinical Parasitology by Beaver
et al, 9th ed., 246)
A. Capillaria philippinensis
C. Trichuris trichiura
B. Enterobius vermicualris
D. Trichinella spiralis
The infective larva of this parasite causes erythematous, serpiginous, intracutaneous tunnels which is accompanied
by intense itchiness: (Clinical Parasitology by Beaver et al, 9th ed., 281)
A. Ancylostoma braziliense
C. Toxocara canis
B. Strongyloides stercoralis
D. Trichuris trichiura
The adult stage of this parasite is described to have attenuated anterior portion and a fleshy posterior portion:
(Clinical Parasitology by Beaver et al, 9th ed., 240)
A. Ancylostoma brazilinese
C. Necator americanus
B. Strongyloides stercoralis
D. Trichuris trichiura
All these parasites have a lung phase during their developmental cycle EXCEPT: (Clinical Parasitology by Beaver et
al, 9th ed., 259, 283, 310)
A. Ascaris lumbricoides
C. Strongyloides stercoralis
B. Enterobius vermicularis
D.. Necator americanus
Embryonated ovum is the infective stage of this parasite: (Clinical Parasitology by Beaver et al, 9 th ed., 303)
A. Trichinella spiralis
C. Capillaria philippinensis
B. Enterobius vermicularis
D. Strongyloides stercoralis
This adult nematode is larviparous which means it produces larva rather than ova: (Clinical Parasitology by Beaver et
al, 9th ed., 232)
A. Capillaria philippinensis
C. Enterobius vermicualris
MICROBIOLOGY
Page 83 of 137
B. Trichinella spiralis
D. Trichuris trichiura
77.
The most effective preventive measure against this parasite is thru thorough freezing or cooking of food
potentially carrying the parasite: (Clinical Parasitology by Beaver et al, 9 th ed., 240)
A. Trichuris trichiura
C. Enterobius vermicularis
B. Trichinella spiralis
D. Strongyloides stercoralis
78.
When embryonated egg of this parasite is ingested, its second-stage larva encysts in tissues such as the liver
and lungs and cause hypereosinophilia: (Clinical Parasitology by Beaver et al, 9 th ed., 326-327)
A. Wuchereria bancrofti
C. Toxocara canis
B. Trichinella spiralis
D. Trichuris trichiura
A stained peripheral blood film revealed the presence of this microfilaria with two discreet nuclei at the tip of the tail:
(Clinical Parasitology by Beaver et al, 9th ed., 365)
A. Brugia malayi
C. Onchocerc volvulus
B. Loa loa
D. Wuchereria bancrofti
The ova of this parasite are NOT commonly recovered in the stool: (Clinical Parasitology by Beaver et al, 9 th ed., 303)
A. Ascaris lumbricoides
C. Enterobius vermicularis
B. Trichuris trichiura
D. Hookworms
Routine examination of a stool sample was submitted for facalysis. The result revealed the presence of spherical
cyst-like material with two nuclei and one chromatoidal body with rounded ends. What is the most probable
parasite seen? (Clinical Parasitology by Beaver et al, 9 th ed., 103)
A. Balantidium coli
C. Giardia lamblia
B. Entamoeba coli
D. Entamoeba histolytica
This parasite is transmitted by ingestion of the sporulated oocyst: (Basic ClinicalParasitology by Brown et al, 6th ed.,
p.52)
A. Naegleria fowleri
C. Cryptosporidium parvum
B. Trichomonas vaginalis
D. Balantidium coli
Parasites which are rounded anteriorly and pointed posteriorly with a pair of nuclei containing a central karyosome
were seen in watery stool of a patient with severe diarrhea. What is the most likely causative agent?
(Clinical Parasitology by Beaver et al, 9th ed., 44)
A. Trichomonas vaginalis
C. Giardia lamblia
B. Chilomastix mesnilii
D. Endolimax nana
Nelson, 32-year old engineer. Was admitted because of on and off high grade fever with accompanying chills of two
weeks duration. Peripheral blood smear showed ring form trophozoites and crescent-shaped gametocytes.
What is the most probable causative agent? (Clinical Parasitology by Beaver et al, 9 th ed., 188)
A. Plasmodium vivax
C. Plasmodium ovale
B. Plasmodium malariae
D. Plasmodium falciparum
The infective stage of Trypanosoma cruzi is... (Clinical Parasitology by Beaver et al, 9th ed., 90)
A. Amastigote
C. Epimastigote
B. Promastigote
D. Trypomastigote
Kalaazar is caused by this parasite: (Clinical Parasitology by Beaver et al, 9 th ed., 71)
A. Trypanosoma cruzi
C. Leishmania donovani
B. Trypanosoma gambiense
D. Leishmania tropica
87.
CJ, 1 year old male, had dysentery described as bloody and mucoid. What are the most probable causative
agents? (Clinical Parasitology by Beaver et al, 9th ed., 115, 216)
A. Entamoeba coli and Endolimax nana
B. Entamoeba histolytica and Balantidium coli
C. Cryptosporidium parvum and Isospora belli
D. Giardia lamblia and Trichomonas hominis
MICROBIOLOGY
Page 84 of 137
88.
The infective stage of Plasmodium falciparum to the intermediate host is the… (Clinical Parasitology by
Beaver et al, 9th ed., 176)
A. Trophozoites
C. Merozoites
B. Sporozoites
D. Gametocytes
89.
This is the only trematode with a non-operculated ovum: (Clinical Parasitology by Beaver et al, 9 th ed., 418,
452, 459, 464)
A. Schistosoma japonicum
C. Clonorchis sinensis
B. Fasciola hepatica
D. Paragonimus westermani
90. The metacercaria is the infective stage of the following parasites EXCEPT: (Clinical Parasitology by Beaver et
al, 9th ed., 416-417, 458, 473, 481)
A. Opistorchis felineus
C. Schistosoma japonicum
B. Metagonimus yokogawai
D. Echinostoma ilocanum
91.
Mang Pandoy, 58 years old male, had a history of four weeks cough with blood-tinged expectorate. Sputum
examination was done which revealed the presence of operculated ova. What is the most likely causative
agent? (Clinical Parasitology by Beaver et al, 9 th ed., 464)
A. Metagonimus yokogawai
C. Paragonimus westermani
B. Clonorchis sinensis
D. Echinostoma ilocanum,
92. This cestode requires two intermediate hosts for the completion of its developmental cycle: (Clinical Parasitology
by Beaver et al, 9th ed., 496)
A. Taenia saginata
C. Echinococcus granulosus
B. Diphyllobothrium latum
D. Dipylidium caninum
93
Sparganum larva is the infective stage of this parasite: (Clinical Parasitology by Beaver et al, 9 th ed., 496)
A. Diphyllobotrium latum
C. Dipylidium caninum
B. Taenia solium
D. Hymenolepis nana
94
These parasites are transmitted by eating improperly cooked pork EXCEPT: (Clinical Parasitology by
Beaver et al, 9th ed., 167, 240, 519, 533)
A. Taenia solium
C. Trichinella spiralis
B. Echinococcus granulosus
D. Toxoplasma gondii
95
This parasite utilizes the snail as its first and second intermediate hosts to complete its developmental cycle:
(Clinical Parasitology by Beaver et al, 9th ed., 458)
A. Metagonimus yokogawai
C. Paragonimus westermani
B. Clonorchis sinensis
D. Echinostoma ilocanum,
96.
This parasite is described as ribbon-like, scolex with 4 cup-like suckers and rostellum with double row of
large and small hooks: (Clinical Parasitology by Beaver et al, 9 th ed., 513)
A. Taenia soilum
C. Hymenolepis nana
B. Taenia saginata
D. Hymenolepis diminuta
97.
This parasite is referred to as the dwarf tapeworm: (Basic Clinical Parasitology by Brown, 6 th ed, p. 191)
A. Hymenolepis nana
C. Echinococcus granulosus
B. Hymenolepid diminuta
D. Dipylidium caninum
98.
Which of the following cestodes does NOT require an intermediate host to complete its life cycle? (Basic
Clinical Parasitology by Brown, 6th ed, p. 192)
A. Hymenolepis nana
C. Echinococcus granulosus
B. Hymenolepid diminuta
D. Dipylidium caninum
Proper cooking of food is the most important preventive measure against this condition: (Clinical Parasitology by
Beaver et al, 9th ed., 247, 261, 425)
A. Schistosomiasis
C. Capillariasis
B. Cochin china diarrhea
D. Malaria
100. Man becomes infected by this parasite by ingestion of cysticercoid larva: (Clinical Parasitology by Beaver et al,
9th ed., 508)
MICROBIOLOGY
Page 85 of 137
A. Taenia solium
B. Dipylidium caninum
C. Diphyllobothrium latum
D. Metagnonimus yokogawai
1. A newborn presents with lethargy, purpura, chorioretinitis and pulmonary infiltrates. Which of the following could be
a cause of this syndrome?
(A) Amebiasis
(B) Candidiasis
(C) Cytomegalovirus
(D) Human papillomavirus
ANS: C
2. Which one of the following is a subcellular structure that is found only in Gram negative bacteria?
(A) endospores
(B) lipopolysaccharide
(C) mitochondria
(D) phosphatidylethanolamine
ANS: B (Micro)
3. Which one of the following statements best describes a function of the peptidoglycan layer in bacteria?
(A) The ability of bacteria to survive changes in the osmolarity of their environment is due to the physical properties of
the peptidoglycan layer.
(B) The peptidoglycan layer contains all the enzymes responsible for the synthesis of membrane phospholipids.
(C) The peptidoglycan layer protects bacteria from the effects of ultraviolet light.
(D) The peptidolgycan layer renders Gram negative bacteria resistant to detergents.
ANS: A (Micro)
4. Which one of the following structures is responsible for the passage of low molecular weight solutes through the
outer membrane of Gram negative bacteria?
(A) capsules
(B) endospores
(C) flagella
(D) porins
ANS: D (Micro)
5. Which one of the following is a component of the cytoplasmic membrane of Gram positive bacteria?
(A) Braun's lipoprotein (Murein lipoprotein)
(B) capsules
(C) lipopolysaccharide
(D) phosphatidylglycerol
ANS: D (Micro)
6. The inhibition of bacterial protein synthesis by the macrolide antibiotic erythromycin is directly due to which one
of the following mechanisms?
(A) Covalent modification of the ribosomal S12 protein
(B) Inhibition of the “charging” of transfer RNA (tRNA)
(C) Inhibition of the formation of a stable 70S initiation complex
(D) Inhibition of the peptidyl transfer reaction
(E) Inhibition of the translocation step
7.
E.
ANS: E
Your patient, suffering from a bacterial infection, receives a penicillin injection and almost immediately
experiences respiratory distress and loses consciousness. This reaction is MOST LIKELY mediated by
(A) complement.
(B) CD8+ cytolytic T cells.
(C) macrophages.
(D) mast cells.
ANS: D
8.
The complement component, C3b, plays an important role in the immune response by functioning to increase
(A) antibody recognition.
(B)
chemotaxis.
(C)
opsonization.
(D) TCR signaling.
MICROBIOLOGY
Page 86 of 137
ANS: C
9. Which one of these statements explains the origin of Rh antibodies in the blood of an
Rh-negative mother?
(A)
(B)
(C)
(D)
They are naturally occurring in the blood of all Rh-negative persons.
They are received through mis-matched blood transfusions.
They are synthesized by the mother in response to Rh-positive cells from her first child.
They are transferred from the Rh-positive father during intercourse.
ANS: C
10.
Which one of the following bacterial pathogens requires selective culture conditions for reliable isolation from
the stool, including media with antibiotics and incubation at 42 C in increased CO2?
A) Campylobacter jejuni
B) Enterohemorrhagic Escherichia coli
C) Salmonella sp.
D) Shigella sp.
ANS: A, Campylobacter jejuni
11. Which one of the following is considered facultatively intracellular for human macrophages?
A) Enteroinvasive Escherichia coli
B) Enterotoxigenic Escherichia coli
C) Salmonella typhi
D) Shigella sp.
ANS: C, Salmonella typhi
12..
A 20-year-old female presents with severe pelvic pain, fever, nausea and vomiting. She was on the second
day of her menstrual cycle. A purulent cervical discharge was detected upon physical examination. Oxidase
positive, Gram-negative diplococci were isolated on Thayer Martin agar from the endocervical swab. What is
the MOST LIKELY identity of this organism?
A)
B)
C)
D)
Bordetella pertussis
Chlamydia trachomatis
Haemophilus ducreyi
Neisseria gonorrhoeae
ANS: D, Neisseria gonorrhoeae
13.
A large outbreak of food poisoning occurred at a company picnic. Eighty people reported explosive vomiting
and diarrhea within 3 hours after eating. All of the victims recovered within 24 hours without medical
intervention. No known bacterial or viral pathogens were isolated from the stool of affected individuals. A
survey of the food eaten at the picnic suggested that potato salad was the common source of the poisoning.
The individual who prepared the salad had a sty on her left eye. Which one of the following is the MOST
LIKELY cause of this outbreak?
A)
B)
C)
D)
Clostridium perfringens
Salmonella typhimurium
Staphylococcus aureus
Staphylococcus epidermidis
ANS: C, Staphylococcus aureus
14.
Gram-positive cocci and polymorphonuclear leukocytes are observed on Gram stain of sputum from a 60year old alcoholic with pneumonia. Culture of the sputum reveals alpha-hemolytic, catalase-negative
bacteria. Which one of the following laboratory tests is the MOST APPROPRIATE next step in the definitive
identification of this organism?
MICROBIOLOGY
Page 87 of 137
E. A) bacitracin sensitivity test
B) beta-lactamase production test
C) ELEK test
D) bile solubility test
ANS: D, bile solubility test
15.
A 19-year old female is admitted to an outpatient clinic with suprapubic pain, dysuria and an urgency to
urinate frequently. She is otherwise healthy. A clean catch midstream urine specimen was collected and
sent to the laboratory. Numerous neutrophils are detected in the urine sample. The bacteriology laboratory
reports that less than 102 Gram-positive cocci and greater than 105 Gram-negative bacilli were recovered
per milliliter of urine. Which one of the following is the MOST LIKELY cause of her infection?
A) Escherichia coli
B) Klebsiella pneumoniae
C) Pseudomonas aeruginosa
D) Staphylococcus aureus
ANS: A, Escherichia coli
16.
A 35-year old male park ranger in rural New Mexico arrives to an emergency room with fever and signs and
symptoms of septic shock. An enlarged lymph node is discovered under his arm upon physical examination.
Gram-negative coccobacilli that demonstrate bi-polar staining were seen in both lymph node aspirate and
spinal fluid from the patient. Which one of the following is the MOST LIKELY etiologic agent of this patient’s
illness?
A) Brucella abortus
B) Borrelia burgdorferi
C) Chlamydia trachomatis LGV
D) Rickettsia rickettsii
E) Yersinia pestis
ANS: E, Yersinia pestis
17.
A 40-year old male hog farmer complains of intermittent fever, night sweats and headaches to his physician.
The patient is hospitalized and blood and bone marrow specimens are collected. The physician requests
that the laboratory incubate the cultures for at least 6 weeks. Nine days after admission, Gram negative
coccobacilli that require increased CO2 for growth are isolated from the blood and bone marrow cultures.
The patient is MOST LIKELY to be suffering from which one of the following diseases?
A) Brucellosis
B) Bubonic plague
E.
C) Legionnaire’s disease
D) Rocky Mountain spotted fever
ANS: A, Brucellosis
18.
Which one of the following diseases is MOST LIKELY to respond favorably to treatment with a -lactam
antibiotic?
A)
B)
C)
D)
Legionellosis
Mycoplasmosis
Syphilis
Trachoma
ANS: C, Syphilis
19.
A 20-year old woman comes to your clinic complaining of an irritating vaginal discharge. Upon examination,
you find that the cervix and vaginal walls appear normal. The discharge is thin and milky, pH 5.5, and has a
fishy odor when treated with potassium hydroxide. You are unable to detect pseudohyphae, buds, or
MICROBIOLOGY
Page 88 of 137
flagellates upon microscopic examination. A Gram stain reveals numerous curved Gram-negative rods,
epithelial cells, and clue cells, but relatively few Gram-positive rods or WBCs. Cervical and vaginal cultures
are negative for STDs. What is your diagnosis?
A) Bacterial vaginosis
B) Cervicitis
C) Trichomoniasis
D) Vaginal candidiasis
ANS: A, Bacterial vaginosis
20.
A 75-year old man in a nursing home is treated with a broad spectrum antibiotic for bed sores. Two days
later he develops bloody diarrhea. A stool specimen from the patient is positive in an ELISA test for a
necrotizing cytotoxin, and his treatment is switched to vancomycin. The agent MOST LIKELY to be
responsible for this clinical syndrome is
A)
B)
C)
D)
Bacteroides fragilis.
Campylobacter jejuni.
Clostridium difficile.
Escherichia coli O157:H7.
ANS: C, Clostridium difficile
21.
Which one of the following toxins is responsible for scalded skin syndrome?
A) Alpha hemolysin
B) Exfoliatin
C) Hyaluronidase
D) Staphylococcal enterotoxin
ANS: B, Exfoliatin
22.
A 50-year old cancer patient with severe neutropenia became septic two weeks after being admitted to the
hospital. An oxidase-positive, Gram-negative rod that did not ferment glucose was isolated from the patient’s
blood. Which one of the following organisms is the MOST LIKELY cause of the patient’s sepsis?
A) Enterobacter cloacae
B) Escherichia coli
C) Klebsiella pneumoniae
D) Pseudomonas aeruginosa
ANS: D, Pseudomonas aeruginosa
23.
Blood culture from a 12-year old with acute osteomyelitis yields Gram-positive cocci that are b hemolytic and
catalase positive. Which one of the following laboratory tests is the MOST APPROPRIATE next step in the
definitive identification of this organism?
A) Coagulase test
B) Optochin sensitivity test
C) Slide agglutination test for capsular antigen type
D) Test for capacity to grow in bile esculin
ANS: A, Coagulase test
24.
Activated macrophages are the effector cells in the expression of a protective, cell-mediated immune
response against
A)
B)
C)
D)
Corynebacterium diphtheriae.
Haemophilus influenzae type b.
Listeria monocytogenes.
Streptococcus pneumoniae.
ANS: C, Listeria monocytogenes
MICROBIOLOGY
Page 89 of 137
25.
A patient recovering from a crushing leg injury suffered during an a motorcycle accident develops a
temperature of 100o F, a rapid pulse, and extreme pain at the site of his the wound two days after the
accident. The skin overlying the patient’s sutured wound is brownish- white, shiny, and studded with
vesicles. Gram stain of the watery exudate from the wound reveals Gram-positive rods, Gram-negative
rods, and Gram-positive cocci. The Gram-positive rods are MOST LIKELY
A)
B)
C)
D)
Bacillus cereus.
Bacteroides fragilis.
Clostridium perfringens.
Corynebacterium ulcerans.
ANS: C, Clostridium perfringens
26.
A 13-year old and her 10-year old brother are seen at an emergency room with complaints of double vision,
difficulty swallowing, and progressive muscular weakness. A detailed history reveals that both children
attended a church picnic three days earlier but neither child has gastroenteritis. The MOST LIKELY
presumptive diagnosis is
A) botulism.
B) Chinese rice syndrome.
C) Guillan-Barré syndrome.
D) salmonellosis.
ANS: A, botulism
27.
Epidemics of bacterial meningitis in young, previously healthy adults are MOST LIKELY caused by
A) Escherichia coli K-1.
B) Haemophilus influenzae type b.
C) Listeria monocytogenes.
D) Neisseria meningitidis group A.
ANS: D, Neisseria meningitidis group A.
28.
A 55-year old woman develops endocarditis following oral surgery. Gram-positive, alpha-hemolytic,
catalase-negative cocci are isolated from a blood culture. The MOST LIKELY agent is
A)
B)
C)
D)
E)
Enterococcus faecalis (group D streptococci).
Staphylococcus aureus.
Staphylococcus epidermidis.
Streptococcus pyogenes (group A streptococci).
Viridans streptococci.
ANS: E, Viridans streptococci.
29.
A 2-month old infant is admitted to the hospital with fever, lymphocytosis, and bouts of violent coughing that
often end in vomiting. The infant’s mother reveals that the child has not as yet received any immunizations.
The attending physician swabs the infant’s nasopharynx and requests that the swab be plated on routine
media as well as a special medium (Bordet-Gengou) that contains blood and glycerol. No organisms are
found on blood or chocolate agar, but small Gram-negative rods are isolated on Bordet-Gengou agar. The
MOST LIKELY cause of the infant’s illness is
A)
B)
C)
D)
Bordetella pertussis.
Chlamydia pneumoniae.
Moraxella catarrhalis.
Mycoplasma pneumoniae.
ANS: A, Bordetella pertussis.
30.
Which one of the following viruses establishes a chronic persistent infection in humans?
A)
B)
Hepatitis A virus
Hepatitis C virus
MICROBIOLOGY
Page 90 of 137
C) Herpesvirus
D) Poliovirus
ANSWER: B, Hepatitis C virus
31.
Which one of the following parasitic infections is MOST LIKELY to produce life-threatening anemia in
children?
A)
B)
C)
D)
Amoebic dysentery
Malaria
Hookworm disease
Kala azar
ANSWER: B, Malaria
32.
Which one of the following viruses is transmitted via the gastrointestinal route?
A)
B)
C)
D)
Delta-associated agent
Hepatitis A
Hepatitis B
Hepatitis C
ANSWER: B, hepatitis A.
33. A 33-year-old pregnant woman delivered a baby boy via a normal vaginal delivery. Neither she nor the
father has a history of genital or oral herpes. Although she had chickenpox as a child, she was exposed to a
child with chickenpox about 10 days before delivery. There was no evidence of vesicular skin lesions on
either the mother or the child at the time of delivery. Three days after birth, the baby now develops vesicular
skin lesions on his back, accompanied by a fever. Which one of the following viruses is MOST LIKELY to
be the cause of the baby's infection?
A)
B)
C)
D)
Cytomegalovirus
Herpes simplex virus type 1
Herpes simplex virus type 2
Human herpesvirus 6
ANSWER: C, Herpes simplex virus type 2
34. Which one of the following viruses is MOST LIKELY to infect and cause rapid cell lysis in a fibroblast cell
line culture?
A)
B)
C)
D)
Hepatitis B virus
Herpes simplex virus
Human papilloma virus
Norwalk virus
Answer: B, Herpes simplex virus
35.
Six months after returning from travelling in North Africa and the Middle East, a graduate student
experiences two episodes of intense headache, shaking chills, and high fever, followed by sweating and
exhaustion. The episodes occurred 48 hours apart and each lasted about 6 hours. He appears at your
clinic the day after the second episode. Laboratory test results reveal low hematocrit (35%), slight
hypoglycemia, and an intraerythrocytic parasite on Giemsa-stained blood films. What is your diagnosis?
A)
B)
C)
D)
Vivax malaria
Falciparum malaria
Ovale malaria
Quartan malaria
ANSWER: A, Vivax malaria
36.
Examination by dark-field microscopy of scrapings from a genital ulcer of 19-year old female reveals mobile,
spiral-shaped organisms against a black background. The MOST LIKELY diagnosis is
MICROBIOLOGY
Page 91 of 137
A) chancroid.
B) Chlamydia trachomatis infection.
C) gonorrhea.
D) syphilis.
ANS: D, syphilis
37.
To accurately determine the cause of repeated bouts of otitis media that have begun to affect a toddler’s
hearing, inner ear fluid is obtained by careful puncture of her tympanic membrane with a sterile needle.
Gram stain of the aspirate reveals short Gram-negative rods and polymorphonuclear leukocytes. No
organisms are isolated on blood agar but colonies are observed on chocolate agar. The MOST LIKELY
cause of this infection is
A)
B)
C)
D)
Haemophilus influenzae non typable.
Haemophilus influenzae type b.
Klebsiella pneumoniae.
Streptococcus pneumoniae.
ANS: A, Haemophilus influenzae non typable.
38. Gram-positive cocci are observed on a Gram stain of purulent discharge from a skin lesion of a 7-year-old.
Culture of the exudate reveals beta-hemolytic, catalase-positive organisms. Which one of the following laboratory
tests is the MOST APPROPRIATE next step in the definitive identification of this organism?
A) Bacitracin sensitivity test
B) Bile solubility test
C) Coagulase test
D) Optochin sensitivity test
ANS: C
39. Which one of the following descriptions is correct about cholera gravis?
A) Associated with the formation of colonic ulcers
B) Death may occur due to dehydration, acidosis, and shock
C) Most common clinical manifestation of Vibrio cholerae infection
D) Occurs in response to plasmid-borne heat stable (ST) toxin
ANS: B
40. Which one of the following species or serovars of Chlamydia can be transmitted as a zoonotic infection?
A) Chlamydia pneumoniae
B) Chlamydia psittaci
C) Chlamydia trachomatis serovar C
D) Chlamydia trachomatis serovar D
ANS: B
41. Gram stain of synovial fluid from the knee of a 20-year-old woman with arthritis reveals Gram-negative cocci. To
isolate the MOST LIKELY responsible agent, the specimen should be plated on
A) Bordet-Gengou agar.
B) Chocolate agar.
C) Eosin Methylene Blue (EMB) agar.
D) MacConkey agar.
ANS: B
42. The formation of colonic ulcers and absesses during Shigella dysentery is due to
A) ability of bacteria to survive stomach acidity because of the production of urease.
B) bacterial survival within macrophages.
C) invasion of enterocytes and cell-to-cell spread of the bacteria.
D) production of a superantigen by the bacillus.
ANS: C
43. An 8-year-old boy living in a wooded area of Virginia suddenly develops fever, headache, and myalgia. On
physical exam, a rash is noted on his wrists and ankles. Within hours, the rash spreads to his trunk. Numerous
arthropod bites are observed on the child's extremities. No organisms are isolated from 3 sequential blood cultures.
MICROBIOLOGY
Page 92 of 137
The child is treated with tetracycline and recovers. Serological studies on acute and convalescent serum samples
from the child are MOST LIKELY to reveal that the etiologic agent was
A) Borrelia recurrentis.
B) Coxiella burnetti.
C) Franciscella tularensis.
D) Rickettsia rickettsia.
ANS: D
44. Which one of the following organisms is the MOST COMMON cause of bacterial diarrhea in the United States?
A) Campylobacter jejuni
B) enterotoxigenic Escherichia coli
C) Salmonella cholerasuis
D) Shigella sonnei
ANS: A
45. Dissemination of Salmonella typhi from the intestine to extraintestinal sites requires survival of this pathogen in
which one of the following host cells?
A) Colonic enterocytes
B) Gastric epithelial cells
C) M cells
D) Macrophages
ANS: D
46. Gram stain of cerebrospinal fluid from a 3-year-old child with fever, petechiae, stiff neck and positive Kernig and
Brudzinski's signs reveals PMNs and a few Gram-negative cocci. The MOST LIKELY agent is
A) Escherichia coli.
B) Haemophilus influenzae type b.
C) Neisseria meningitidis.
D) Streptococcus agalactiae (group B streptococci ).
ANS: C
47. Listeria monocytogenes infection of an adult is MOST LIKELY to be acquired by
A) ingestion of contaminated food.
B) inhalation of contaminated aerosals.
C) inoculation with a contaminated needle.
D) person-to-person contact.
.
ANS: A
48.
What is the mechanism of action of the antifungal drug fluconazole?
A)
B)
C)
D)
E)
49.
Binds to ergosterol resulting in membrane disruption
Inhibits ergosterol synthesis
Inhibits protein synthesis
Inhibits RNA synthesis
Prevents cell wall formation by inhibiting chitin synthesis
A 10-year-old with a dog complains of a “bump” on the back of his head. He says its redness and itching
have increased over the last three weeks. The lesion is several centimeters in diameter, with hair
involvement and scaling. Examination of several of his hairs in a KOH preparation reveals hyphae in and
around the hair shafts. What is your diagnosis?
A)
B)
C)
D)
Actinomycosis
Candidiasis
Pityriasis versicolor
Tinea capitis
ANSWER: D, Tinea capitis
50.
You are assigned to a hospital in Phoenix, Arizona. Several patients have been admitted in the past week
with cough, chest pain, fever, and pneumonitis that has not responded to antibacterial therapy. You
discover that most of the patients are new arrivals to Arizona who had been working outdoors during the
MICROBIOLOGY
Page 93 of 137
dusty wind storms that had occurred over the past few weeks. The causative organism is observed in a
KOH preparation of sputum. What is your presemptive diagnosis?
A)
B)
C)
D)
Blastomycosis
Coccidioidomycosis
Histoplasmosis
Psittacosis
ANSWER: B, Coccidioidomycosis
51. A 65-year-old male develops endocarditis following urinary catheterization. Gram-positive, catalase-negative
cocci are isolated from a blood culture. The MOST LIKELY agent is
A) Enterococcus faecalis (group D streptococci).
B) Staphylococcus aureus.
C) Staphylococcus epidermidis.
D) Streptococcus pyogenes (group A streptococci).
ANS: A
52. A 10-year-old child of an instructor at the U.S. Air Force Academy develops marked swelling in her right inguinal
lymph node with fever of 104.1EF. An aspirate from the painful, swollen node reveals Gram-negative rods. A direct
immunofluorescent assay on the same aspirate provides a definitive diagnosis. The child is treated with streptomycin,
and the aspirate is sent for culture. The culture yields white colonies on MacConkey agar grown at room temperature.
The MOST LIKELY agent is
A) Brucella canis.
B) Yersinia pestis.
C) Francisella tularensis.
D) Haemophilus ducreyi.
ANS: B
53. A 22-year-old develops an abcess in his peritoneum following rupture of his appendix. Gram stain of the exudate
from his foul-smelling abcess reveals numerous polymorphonuclear leukocytes, Gram-positive cocci, Gram-positive
rods, and Gram-negative rods. Aerobic culture of the exudate at 37o C on blood and MacConkey agar plates yields
only enterococci (group D streptococci). The Gram-negative rods are MOST LIKELY
A) Bacteroides fragilis.
B) Escherichia coli.
C) Pseudomonas aeruginosa.
D) Shigella dysenteriae.
ANS: A
54. Several people become ill within two hours after leaving a summer reunion picnic. The majority of people
complain of vomiting and diarrhea without fever, and recovered within 24 hours after the onset of symptoms. The
contaminated food is determined to be a bowl of coleslaw prepared by someone with a stye on her left eyelid. Which
one of the following is the MOST LIKELY source of the food poisoning?
A) Clostridium botulinum
B) Staphylococcus aureus
C) Salmonella enteriditis
D) Shigella sonnei
ANS: B
55. A 35-year-old female presents with fever, headache and a non-productive cough. The patient is prescribed
penicillin and sent home. Two weeks later, she returns with similar symptoms. A Gram stain and routine culture of the
patient's sputum reveal only normal mouth flora. Sputum is then cultured on specialized medium containing sterols,
and, after two days, tiny colonies were visible under a microscope. Which one of the following organisms does the
physician suspect is the cause of the patient's pneumonia?
A) Corynebacterium diphtheriae
B) Haemophilus influenzae type b
C) Legionella pneumophila
D) Mycoplasma pneumoniae
ANS: D
56. A 70-year-old male, who is a chain-smoker, complains of fever, diarrhea, cough and chest pain. Evidence of
pneumonia is seen by chest X-ray. Due to the inability of the patient to produce a sputum specimen, a transtrachial
MICROBIOLOGY
Page 94 of 137
aspirate is obtained. No respiratory bacterial pathogen is isolated from aspirated fluid cultured on blood agar plates.
However, poorly staining Gram-negative rods are isolated on buffered charcoal-yeast extract (BCYE) agar after three
days of incubation. The physician had already begun treatment with erythromycin due to the detection of bacilli in the
aspirated fluid using a fluorescently-labeled monoclonal antibody specific for the causative agent. The MOST LIKELY
pathogen responsible for this pneumonia is
A) Haemophilus influenzae.
B) Legionella pneumophila.
C) Mycoplasma pneumoniae.
D) Neisseria meningitides.
ANS: B, Legionella pneumophila
57. Immunopathologic delayed-type hypersensitivity responses to eggs in tissue, resulting in granulomas, obstruction
and fibrosis is characteristic of
A. acute trichinosis.
B. Chagas' disease.
C. . schistosomiasis.
D. onchocerciasis.
Answer: C
58. Which one of the following fungi exhibits thermal dimorphism?
A. Aspergillus fumigatus
B. Candida tropicalis.
C. Cryptococcus neoformans
D. Histoplasma capsulatum
Answer: D
59. An otherwise healthy 35-year-old female florist presented to her primary care physician complaining of a
persistent lesion on the third digit of her left hand. This had been present for several weeks and showed no sign of
resolving. Upon examination, the patient was found to have a mild fever (100 0F), the aforementioned lesion and two
nodules on her left forearm. A biopsy of the lesion was performed, and the pathology report indicated the presence of
numerous polymorphonuclear leukocytes and a rare elongated budding yeast. What is the MOST LIKELY etiologic
agent?
A. Blastomyces dermatitidis
B. Candida albicans
C. Histoplasma capsulatum
D. Sporothrix schenckii
Answer: D
60. Mode of transmission of Trichomonas vaginalis is:
A. Ingestion of infested beef
B. Skin contact with larvae
C. Mosquito bites
D. Sexual
E. Fecal-oral
61. Mode of transmission of Onchocerca volvulus is:
A. Exposure to infested water
B. Louse bites
C. Skin contact with larvae
D. Blackfly bites
E. Wound infection
62. Mode of transmission of Leishmania mexicana is:
A. Sexual
B. Mosquito bites
C. Fecal-oral
D. Tick bites
E. Sandfly bites
63. Mode of transmission of Enterobius vermicularis is:
A. Sexual
B. Blackfly bites
C. Louse bites
D. Fecal-oral
E. Deer fly bites
64. Oxygen requirements of Neisseria gonorrhoeae is:
A. Aerobic
B. Facultative anaerobe
MICROBIOLOGY
Page 95 of 137
C. Microaerophilic
D. Obligate anaerobe
65. Mode of transmission of Borrelia burgdorferi is:
A. Direct contact with animals
B. Conidia inhalation
C. Sexual
D. Tick bites
E. Transplacental
66. Therapy of Vibrio cholerae is:
A. TMP-SMX
B. ORS
C. Ceftriaxone
D. Erythromycin
E. Augmentin
67. Family of Measles virus is:
A. Papovaviridae
B. Rhabdoviridae
C. Orthomyxoviridae
D. Flaviviridae
E. Paramyxoviridae
68. Family of Hepatitis A virus is:
A. Papovaviridae
B. Hepadnaviridae
C. Parvoviridae
D. Picornaviridae
E. Bunyaviridae
69. Which of the following statements refers to Toxoplasma gondii:
A. Sandfly gut: promastigote -> amastigote
B. Three major serogroups. After-infection immunity only for the serougroup of the strain involved.
C. Primary infection is usually asymptomatic. Congenital infection results in cytomegalic inclusion disease.
D. ELISA helps distinguish from dispar - non-pathogenic.
E. Cyst(bradyzoites) -> feline gut: schizogony/gametogony -> oocysts -> tachozoites -> cysts
70. Mode of transmission of Leishmania donovani is:
A. Sandfly bites
B. Sexual
C. Exposure to infested water
D. Spore ingestion
E. Fecal-oral
71. Phenotypic characteristic of Schistosoma hematobium is:
A. Eggs have a small lateral spine
B. Eggs have a terminal spine
C. Non-enveloped
D. Four suckers and circle of hooks on scolex
E. Lactose fermenter
72. Therapy of Taenia solium is:
A. mebendazole
B. TMP-SMX
C. Piperacillin
D. Suramin
E. niclosamide
73. Phenotypic characteristic of Streptococcus pyogenes is:
A. Alpha-hemolytic
B. Oxidase "+"
C. H2S "-"
D. dsDNA
E. Beta-hemolytic
74. Mode of transmission of Rickettsia prowazekii is:
A. Droplet nuclei
B. Direct contact with animals
C. Louse bites
D. Wound infection
E. Ingestion of infested canine feces
MICROBIOLOGY
Page 96 of 137
75. Non-toxin virulence factor of Escherichia coli is:
A. Opacity associated protein (opa) - promotes adhesion to mucosal surfaces
B. Fimbria - adhesion
C. Fibronectin receptor
D. HSP-60 - possible "mimicry" inducing autoimmune response
E. Urease - raises environmental pH
76. Phenotypic characteristic of Gonococcus is:
A. Fastidious
B. Mycolic acid in cell wall
C. Catalase "-"
D. Non-lactose fermenter
E. Alpha-hemolytic
77. Morbidity of Blastomyces dermatitidis is:
A. Urinary tract infections
B. Whooping cough
C. Disseminated granulomas
D. Bronchopneumonia
E. Systemic infection
78. Staphylococcal food poisoning is the result of
(A) a heat labile enterotoxin.
(B) ingestion of a pre-formed enterotoxin produced by Staphylococcus aureus during bacterial multiplication in
contaminated food.
(C) ingestion and multiplication of coagulase-negative Staphylococcus sp.
(D) multiplication of Staphylococcus aureus in the small intestine.
(E) Staphylococcus aureus alpha toxin.
ANS: B
79. A 19-year-old soldier develops fever, carditis, and migratory polyarthritis. While taking her history, the patient
remembers that she had a "sore throat" three weeks previously. A throat culture of the patient yields normal flora
only. The laboratory result MOST CONSISTENT with these findings is that the patient has high titers of antibody to:
(A) meningococcal lipooligosaccharide.
(B) pneumococcal IgA protease.
(C) pneumococcal polysaccharide.
(D) staphylococcal protein A.
(E) streptolysin O.
ANS: E
80. A 65-year-old man with a colonic tumor undergoes a bowel resection. Three days later, the patient develops a
fever, and the drainage from the surgical site is foul smelling. The skin near the site of the drainage is markedly
erythematous. Gram stain of the drainage reveals Gram-positive cocci and Gram-negative rods. Aerobic culture of
the drainage yields streptococci. The Gram-negative rod seen on Gram stain was most likely:
(A) Bacteroides fragilis
(B) Clostridium perfringens
(C) Escherichia coli
(D) Pseudomonas aeruginosa
(E) Shigella flexneri
ANS: A
81. A 50-year-old rancher scrapes his hand with clippers while shearing sheep. Two days later he notices an ulcer
surrounded by vesicles at the site of the wound. Eight days after the injury, the wound develops a black necrotic
center (eschar) surrounded by edematous fluid. The rancher experiences mild systemic symptoms. The most likely
cause of the infection is:
(A) Bacillus anthracis.
(B) Clostridium tetani.
(C) Neisseria meningitidis.
(D) Streptococcus pyogenes (group A streptococci).
(E) Staphylococcus aureus.
ANS: A
82. A 19-year-old college sophomore dies 24 hours after the development of fever, petechiae, purpura, and
disseminated intravascular coagulation. Culture of cerebrospinal fluid and blood obtained from the patient on
admission to the hospital MOST LIKELY yielded:
(A) Gram-negative, lactose fermenting rods.
(B) Gram-negative, oxidase positive diplococci.
(C) Gram-positive, catalase negative cocci.
(D) Gram-positive, catalase positive cocci.
MICROBIOLOGY
Page 97 of 137
(E) Gram-positive, catalase positive rods.
ANS: B
83. The blood culture of a patient with a presumptive diagnosis of endocarditis yields non-hemolytic colonies of
Gram-positive cocci. The isolate is catalase negative, can grow in 6.5% NaCl, and is resistant to penicillin. The isolate
is MOST LIKELY:
(A) Enterococci.
(B) Streptococcus agalactiae (group B streptococci).
(C) Streptococcus bovis.
(D) Streptococcus mutans (Viridans streptoccoci).
(E) Streptococcus pneumoniae.
ANS: A
84. A 25-year-old, previously healthy woman develops fever within 24 hours of delivery of an infant born 6 weeks
prematurely. Blood culture of the mother reveals Gram-positive cocci that are catalase negative, beta-hemolytic on
blood agar, and resistant to bacitracin. The MOST LIKELY cause of the bacteremia is:
(A) Enterococci.
(B) Staphylococcus aureus.
(C) Streptococcus agalactiae (group B streptococci).
(D) Streptococcus pneumoniae.
(E) Streptococcus pyogenes (group A streptococci).
ANS: C
85. A 24-year-old college student comes to the emergency room with low blood pressure. He reports that he had a
recent cold, with fever and sore throat. On physical exam, he has cervical lymphadenopathy, and an enlarged tender
spleen. Which virus infection is the MOST CONSISTENT with his clinical picture?
A) Adenovirus type 12
B) Epstein-Barr virus
C) Herpes simplex virus type 1
D) Rhinovirus
E) Varicella-zoster virus
Answer: B
86. An HIV-infected homosexual man presents with Kaposi's sarcoma. Which viral genome is MOST LIKELY to be
present in his tumor biopsy?
A. Herpes simplex virus tupe 1
B. Human cytomegalvirus
C. Human herpesvirus type 6
D. Human herpesvirus type 8
E. Varicella-zoster virus
Answer: D
87. Which one of the following viral infections is not effectively prevented by vaccination?
A. Hepatitis B virus
B. Poliovirus
C. Rhinovirus
D. Rubella virus
E. Smallpox virus
Answer: C
88. Prions cause spongiform encephalopathies including the human disease
A. AIDS dementia complex.
B. Creutzfeldt-Jacob disease.
C. scrapie.
D. subacute sclerosing panencephalopathy (SSPE).
E. tropical spastic paraparesis.
Answer: B
89. Congenital infection by which one of the following viruses can result in birth defects?
A. Adenovirus
B. Cytomegalovirus
C. Hepatitis B virus
D. Human immunodeficiency virus
E. Human papilloma virus
Answer: B
MICROBIOLOGY
Page 98 of 137
90. Viral carditis of adolescents and adults is frequently caused by
A. Coxsackieviruses.
B. Orthomyxoviruses.
C. Pseudorabiesvirus.
D. Reoviruses.
E. Rhinoviruses.
Answer: A
91. Several newborns housed in a neonatal intensive care unit have developed nosocomial pneumonia. Most are
wheezing and some have required care such as mechanical ventilation, increased doses of oxygen, antibiotic
therapy, or monitoring for apnea. A simple, rapid diagnostic test reveals a common viral antigen in nasal wash
specimens from each of the patients. The virus MOST LIKELY to be responsible is
A. Coronavirus
B. Echovirus B2
C. Respiratory syncytial virus
D. Rubeola virus (measles)
E. Type A influenza virus
Answer: C
92. A baby is born with microphthalmia, cataracts, and a holosystolic/holodiastolic heart murmer. Infection by what
virus is the MOST LIKELY cause?
A. Cytomegalovirus
B. Enterovirus
C. Human immunodeficiency virus
D. Herpes simplex virus type 2
E. Rubella virus
Answer: E
93. A 7 year-old newly adopted Russian child who has been in the United States 10 days presents with fever,
malaise, sore throat, and a grey membrane on the tonsils, uvula, and soft palate. The child's immunization history is
unknown. The physician obtains a swab of the child's throat which provokes bleeding of the membrane. The swab is
sent to the laboratory for routine throat culture. Two days later the laboratory reports only the presence of normal flora
in the specimen. The MOST LIKELY organism responsible for this child's illness is:
(A) Bordetella pertussis.
(B) Corynebacterium diphtheriae.
(C) Haemophilus influenzae type b.
(D) Listeria monocytogenes.
(E) Streptococcus pyogenes (group A streptococci).
Answer: B
94. Soft cheese imported from Mexico was implicated as the vehicle in an outbreak of meningitis and bacteremia that
occurred among attendees of a large company picnic. Twenty of the 40 affected individuals were pregnant women.
One of the victims had AIDS. Gram positive, non-spore forming rods were isolated from the cheese that had been
stored in the cold. The MOST LIKELY organism responsible for this outbreak is:
(A) Bacillus cereus
(B) Clostridium difficile
(C) Clostridium perfringens
(D) Listeria monocytogenes
(E) Staphylococcus aureus
Answer: D
95. A 25 year old patient presents at a sexually transmitted disease (STD) clinic with a painful genital ulcer. On
examination, the ulcer does not appear to be typical of herpes simplex virus. Dark field microscopy of exudate from
the ulcer is negative for spiral-shaped organisms. However, a small, gram negative rod is isolated from culture of the
exudate on chocolate agar. The organism MOST LIKELY to be responsible for the lesion is:
(A) Chlamydia trachomatis.
(B) Haemophilus ducreyi.
(C) Neisseria gonorrhoae.
(D) Treponema pallidum.
(E) Ureoplasma urealyticum.
Answer: B
96. Which ONE of the following diseases is transmitted by lice?
(A) Epidemic typhus
(B) Ehrlichiosis
(C) Psitticosis
(D) Scrub typhus
(E) Typhoid fever
MICROBIOLOGY
Page 99 of 137
Answer: A
97. A young army recruit presents at a sexually transmitted disease (STD) clinic clinic with acute urethritis. A Gramstained preparation of his urethral exudate reveals neutrophils with intracellular Gram-negative diplococci. The patient
is treated with ceftriaxone and sent home. He is requested to return in one week so that a urethral culture can be
obtained to test for proof of antibiotic cure. Which ONE of the following culture media should be used for the follow-up
culture?
(A) Blood agar
(B) Bordet Gengou agar
(C) MacConkey agar
(D) Serum tellurite agar
(E) Thayer-Martin agar
Answer: E
98. The growth in the vagina of which ONE of the following bacteria helps maintain a protective acid pH?
(A) Bacteroides
(B) Gardnerella
(C) Lactobacillus
(D) Mobiluncus
(E) Peptostreptococcus
Answer: C
99. A 25 year-old sexually active woman complains of vaginal itching, odor, and discharge. The vaginal walls are red
and slightly swollen, but the cervix is normal. A sample of her vaginal discharge is tested with 10% KOH, which
produces a strong fishy odor. Upon microscopic examination of the discharge, you find clue cells but no pear-shaped
flagellates or pseudohyphae. A Gram stain reveals numerous Gram-negative curved rods. What is your diagnosis?
(A) Bacterial vaginosis
(B) Pelvic inflammatory disease (PID)
(C) Syphilis
(D) Trichomoniasis
(E) Yeast infection
Answer: A
100. A 25 year-old male develops an ulcerative lesion of the hand and several fluctuant nodules along the lymphatics
draining that site. The patient is seen in the dermatology clinic at the Naval Hospital. Case history reveals that he is a
weekend gardener who cultivates roses as a hobby. What is the MOST LIKELY fungal disease?
(A) Chromoblastomycosis
(B) Mycetoma
(C) Phaeohyphomycosis
(D) Sporotrichosis
(E) Zygomycosis
Answer: D
OUR LADY OF FATIMA UNIVERSITY-FATIMA MEDICAL SCIENCE FOUNDATION, INC.
FATIMA COLLEGE OF MEDICINE
DEPARTMENT OF MICROBIOLOGY
BASIC MICROBIOLOGY
1.
2.
3.
4.
5.
Which of the following statements is INCORRECT regarding the capsules of bacteria?
A. Most gram positive have capsules and gram negative rarely do
B. Most bacterial capsules are polysaccharides
C. Bacterial capsules can be purified and used as vaccine
D. Bacterial capsules are immunogenic and as a result some bacteria have many serologic types
LPS are correctly described as EXCEPT:
A. found in all gram negative bacteria of all specie
B. able to produce febrile response
C. contain lipid A responsible its antigen specificity
D. it is in the outer membrane
Phase in bacterial growth cycle wherein there is active metabolism but cells do not divide
A. lag phase
B. log phase
D. stationary phase
D. death
Organism that requires complex substances like amino acid as source of carbon
A. lithotrophs
B. autotrophs
C. heterotrophs D. A & C only
Any change in the base sequence of DNA
A. Inversion B. recombination
C. mutation
D. gene replication
MICROBIOLOGY
Page 100 of 137
6.
Mating between F' & F- cells would result to
A. acquisition of maleness by the recipient cell
B. no acquisition of maleness by the recipient cell
C. acquisition of femaleness by the recipient cell
D. acquisition of maleness by the donor’s cell
7.
Invasion of body by pathogens
A. contamination
B. disease
C. pathogenecity
D. infection
8.
The ffg. are associated with the autoclave,except,
A. moist heat
B. live steam
C.121˚C for 15-20 min D.15lb. pressure/sq. in.
9.
The target or binding site of β lactam agents
A. Penicilloic acid B. Thiazolidone ring
C. β lactam bond
D.PBP
10. It inhibits the cross-linking of peptidoglycan
A. autolytic enzymes
B. carboxypeptidase C. topoisomerase
D. transpeptidase
F.
IMMUNOLOGY
11. Deposition of immune complex in tissue
A. Type I
B. Type II
C. Type III
D. Type IV
12. In type I hypersensitivity, the following are true:
13.
14.
15.
16.
17.
18.
19.
20.
A. production of allergen is induced by antibody
C. Both
B. most cells released mediators after initial exposure
D. neither
Antigen-binding site on T cells
A. CD4-molecule
B. TCR
C. IgD
D. CD3 molecule
MHC class II are found on the surface of
A. B cells B. macrophages C. dendritic cells D. all of the above
Binding of TCR CD3 complex with peptide signals:
A. B cell activation B. T cell activation
C. complement activation D. none
Most of the autoimmune diseases are:
A. Type I
B. Type II
C. TypeIII
D. TypeIV
With phagocytic defects:
A. Chronic granulomatous disease
C. Di George syndrome
B. Job’s syndrome
D. A and B only
Most useful test for Chronic Granulomatous Disease:
A. CBC
B. Blood culture
C. NBT
D. PTH determination
Immune deficiency that is associated with hypocalcemia:
A. Di George
B. Bruton’s
C. SCID
D. Job’s
Defective in Chediak Higashi Syndrome
A. CD18
B. Microtubules C. Chemotactic factor
D. All
G. BACTERIOLOGY
21. Specie of Rickettsia that causes severe infection.
A. RMSF
B. Ehrlichiosis
C. Scrub typhus D. Typhus fever
22. Cat Scratch Disease
23.
24.
25.
26.
27.
A. Cardiobacterium hominis
C. Legionella species
B. Erysipelothrix rhisopathiae
D. Bartonella henselae
Virulent types of N. gonorrhea
A. T1
B. T3
C. T4
Most common clinical disease caused by gonococcus seen in males.
A. urethritis
B. prostatitis
C. cervicitis
Found in freshwater ponds.
A. V. vulnificus
B. V. cholerae C. V. parahemolyticus
Cell surface antigen in Salmonella, EXCEPT:
A. O
B. Vi
C. Both
Most contagious stage of Pertussis.
A. catarrhal stage
C. Convalescent stage
B. paroxysmal stage
D. none
D. all of the above
D. A & C only
D. none
D. Neither
MICROBIOLOGY
Page 101 of 137
28. True of Pseudomonas aeruginosa, EXCEPT:
29.
30.
31.
32.
33.
34.
35.
36.
37.
38.
39.
40.
41.
42.
43.
44.
45.
46.
47.
A. hospital acquired
C. can produce pigment
B. opportunistic pathogen
D. susceptible to most antibiotics
The most important cause of gastroenteritis involving seafoods is:
A. V. cholerae
B. V. parahemolyticus
C. P. aeruginosa D. B. pertussis
Gram (-) bacilli, “gull wing” appearance, microaerophilic
A. B. pertussis
B. H. influenzae C. Campylobacter
D. T. pallidium
Important cause of neonatal meningitis.
A. E. coli
C. L. monocytogenes
B. S. agalactiae
D. All of the above
In gonorrhea, the gram stain is very reliable for laboratory diagnosis in which group of patients?
A. asymptomatic women
C. Asymptomatic men
B. men with purulent discharge
D. All of the above
What is responsible for the spectrum of diseases seen in SSSS?
A. exofoliatin
C. sphingomyelinase C
B. leukocidin
D. TSST-I
In cases of active PTB, the lesions on chest x-ray are located over
A. base of the lungs B. middle lobe
C. apex
D. whole lung
This lethal toxin of C. perfringens is a lecithinase which cause severe hemolysis
A. epsilon
B. beta
C. kappa
D. iota
E. alpha
Botulinum toxicity can cause the following EXCEPT
A. respiratory failure
C. cranial nerve dysfunction
B. uncontrolled muscle contraction D. vomiting
Characteristic/s of M. tuberculosis
A. facultative aerobe
C. grows well in lowered CO2 tension
B. hydrophobic
D. all of the above
Blocks the release of neurotransmitters for inhibitory synapses
A. tetanospasmin B, tetanolysin
C. botulinum toxin
D. all of the above
True of B. fragilis
A. growth stimulated by bile
C. implicated with pathogenesis of acne
B. sporeforming anaerobe
D. all of the above
Dissemination of the infection in Syphilis is seen in what stage?
A. primary stage
C. tertiary stage
B. secondary stage
D. B & C only
Diagnostic test for Relapsing fever EXCEPT:
A. microscopy
B. Wright stain C. serology
D. culture
Rickettsial infection which can be transmitted by human to human contact
A. R. prowazeki
B. R. Quintana
C. R. typhi
D. R. akarii
Rickettsia require living cells for its growth, except,
A. R. typhi
B. E. canis
C. R. Quintana
D. C. burnetti
Which of the following statement/s is/are true?
A. All Hemophilus influenzae are pathogenic.
B. Some strains of Yersenia pestis cause disease through inhalation.
C. Both Brucella and Francisella can be transmitted by direct contact.
D. All of the above.
The organism which survives stomach acid by creating an ammonia cloud around it.
A. H. aegypticus
B. bronchoseptica
C. C. jejuni
D. H. pylori
Gram (-) microaerophilic curved bacteria.
A. Campylobacter
B. Helicobacter C. Both D. neither
True about tuberculosis.
A. Disease primarily due to impaired cell-mediated immunity.
B. May be acquired from contaminated unpasteurized milk.
C. The organism may dormant in macrophages without producing disease.
D. All of the above.
CASE: A 7 y/o boy complains of fever and sore throat.
48. Which of the following can cause this illness?
A. S. pyogenes
B. adenovirus
C. EBV
D. all of the above
MICROBIOLOGY
Page 102 of 137
49. Which of the following are possible Sequelae of S. pyogenes throat infection?
A. rheumatic fever B. acute appendicitis
C. both D. neither
CASE: A 1 y/o male had seizure, vomiting and cough. Meningitis was entertained. LP showed high protein, low
sugar, WBC = 1,000 with predominance of segmenters.
50. Which among the following is the gold standard for detecting the causative agent?
A. culture and sensitivity
H.
B. ELISA
C. Gram staining D. All of the above
MYCOLOGY
51. When cultured at room temperature, Sporothrix schenckii is seen as:
52.
53.
54.
55.
A. tear drop-shaped conidia in flowerette arrangement C. Cigar-shaped bodies
B. tuberculate macroconidia
D. sporangium
Lipophilic organism
A. M. furfur
B. E. werneckii C. T. beigelii
D. none of the above
Diabetic patients are prone to
A. mucormycosis
B. Aspergillosis C. Candidiasis
D. All of the above
Budding yeast cell in pilots wheel appearance
A. C. immitis
B.S. schenckii
C. P. brasiliensis
D. C. carionii
The following are true of Systemic Mycoses
A. all are dimorphic B.Geographically restricted
C. Transmitted by inhalation
D.all
VIROLOGY
CASE: A 23 y/o female had fever, nausea, vomiting and anorexia for 4 days followed by jaundice. Laboratory results
showed SGPT 800, anti-HAV IgM (+), anti-HBC IgG negative, anti-HbsAg(+).
56. The patient has:
57.
58.
59.
60.
A. acute hepatitis A
C. carrier state for hepatitis B
B. acute hepatitis B
D. acute hepatitis C
She most likely acquired the infection thru:
A. sexual contact
C. blood transfusion
B. ingestion of fecally contaminated food or water D. kissing
She most likely had prior:
A. Hepatitis A infection
C. Hepatitis A immunization
B. Hepatitis B infection
D. Hepatitis B immunization
Which of the following agents of Hepatitis is an incomplete virus?
A. Hepatitis A
B. Hepatitis B
C. Hepatitis C
D. Hepatitis D
These agents. Though not hepatotrophic, can also cause hepatitis:
A. Einstein Barr virus
B. Cytomegalovirus
C. Salmonella typhi
D. All of the above
61. Progressive multifocal leukoencephalopathy is associated with
A. BK virus B. JC virus
C. HSV
D. Adenovirus
62. True of Poxviruses
A. acquire its envelope from the guarnieri bodies
C. have multiple serotypes
B. buds out from the Guarnieri bodies
D. all of the above
63. Papillomavirus serotypes associated with CA of the cervix & penis
A. 1,2,3,4
B. 6 & 11
C. 16 & 18
D. 6 & 19
64. True of HHV6:
A. closely related to gamma herpesviruses based on molecular analysis
B. based on biologic criteria similar to beta herpes viruses
C. infects lymphocytes
D. all of the above
65. Correct statement/s regarding shingles
A. common among elderly & immunocompromised patient
B. radicular pain
C. unilateral involvement
D. all of the above
MICROBIOLOGY
Page 103 of 137
66. True statement/s regarding viruses.
67.
68.
69.
70.
A. they are cells with a true nucleus & nuclear membrane.
B. composed of only one nucleic acid.
C. can be grown in enriched media.
D. All of the above
Which of the following viruses have haploid nucleic acid?
A. Adenoviruses
B. Picornaviruses C. Retroviruses D. All of the above
Confers instability to the virus.
A. viral genome
B. Viral envelope
C. viral capsid
D. core protein
Correct statement/s regarding defective virus.
A. cannot replicate without the help of another virus.
B. composed of nucleic acid but lacks a protein coat
C. cannot elicit an immune or inflammatory response.
D. All of the above.
Which of the following statement/s is/are correct?
A. interaction of the host cell receptor & viral nucleic acid is the major determinant of species and organ
specificity.
B. viral envelope is composed of lipid derived from the host cell membrane & protein that is virusspecific.
C. outcome of infection is dependent on the viral nucleic acid.
D. All of the above.
The following are the most effective deterrents to progressive viral infection EXCEPT:
A. interferon
C. lymphocytes
B. multinucleated giant cells
D. specific antibody globulin
(Answer: B / Reference: Microbiology p.388 / MPL 60)
Which of the following is true regarding endotoxin?
A. become toxoids with formaldehyde
C. heat labile
B. causes fever
D. very specific in their action
(Answer: B / Reference: Microbiology p.391 / MPL 60)
The most important protective antibody:
A. IgA
B. IgE
(Answer: C / Reference: Microbiology p.224 / MPL 90)
C. IgG
D. IgM
The immunoglobulin that serve as B cell antigen receptor in mature B cells:
A. IgD
C. IgM
B. IgG
D. A and C
(Answer: C / Reference: Microbiology p.233 / MPL 60 )
The antibody produced by the fetus in utero during fetal infection:
A. IgA
B. IgE
(Answer: D / Reference: Microbiology p.206 / MPL 90 )
C. IgG
D. IgM
There are immunogens that require specific recognition by the B cells and T cells. The T cell subset that
regulate the activation and differentiation of B cells in the immune response:
A. Helper T cells
C. Cytotoxic T cells
B. Delayed- type hypersensitivity T cells
D. Suppressor T cells
(Answer: A / Reference: Microbiology p.220 / MPL 60)
Following activation of the complement system, when the membrane attack is effective the end result is:
A. clumping of cells
C. destruction of cell
B. formation of soluble complexes
D. no effect on the cell
(Answer: C / Reference: Microbiology p.292 / MPL 90)
The following events take place inside the phagocytic cells EXCEPT:
A. destruction of bacteria by production of superoxide radicals
destruction of bacteria by oxygen independent mechanisms like lysozymes
MICROBIOLOGY
Page 104 of 137
C. may inhibit the replication of intracellular bacteria, fungi and protozoa
unable to digest dead bacteria
(Answer: D / Reference: Microbiology p.351 / MPL 60 )
The CD8+ T cells recognize antigens in association with MHC Class:
A. I
C. III
B. II
D. IV
(Answer: A / Reference: Microbiology p.303 / MPL 60)
The MHC class 1 genes expressed the antigens on the surface of most tissues like:
A. lymphocytes
C. red blood cells
B. brain
D. A and B
(Answer: A / Reference: Microbiology p.268 / MPL 60)
The similarity between attenuated and inactivated vaccines:
A. produce high level of antibodies
C. given parenterally
replicate in parts of the where immunity is needed
D. safe to administer
(Answer: A / Reference: Microbiology p.365 / MPL 33)
The Immunopathologic process or hypersensitivity reactions that involve the complement system:
A. Immediate hypersensitivity
C. Immune complex reactions
B. Humorally mediated cytotoxic reactions
D. B and C
(Answer: D / Reference: Microbiology p.319 / MPL 60)
Which statement is not true of Type 4 or Delayed type Hypersensitivity reaction?
A. initiated by antigen specific T cells
B. exemplified by tuberculin test
C. histologically shows edema and numerous granulocytes
D. reaction can be transferred in vivo with sensitized T cells
(Answer: C / Reference: Microbiology p.322 / MPL 60)
Some oncofetal antigens are cell-associated but sometimes can be shed like:
A. carcinoembryonic antigen
C. alpha fetoprotein
B. human chronic gonadotropin
D. all of the above
(Answer: A / Reference: Microbiology p.373 / MPL 33)
Failure of the maternal immune system to reject the fetus is attributed to:
A. lack of antigenicity of the fetus
B. humoral modification of maternal immune response
the uterus is a privileged site
fetus is an allograft
(Answer: B / Reference: Microbiology p.381 / MPL 33)
16. The stomach under normal condition is sterile, however, bacteria may colonize in the stomach when the
gastric pH is greater than:
A. 2
C. 4
B. 3
D. 5
(Answer: D / Reference: Microbiology p.396 / MPL 33 )
17. The criterion used for identification of pathogenic staphylococcus is the production of:
A. hemolysin
C. coagulase
B. catalase
D. pigment
(Answer: C / Reference: Microbiology p.402 / MPL 90)
18. The presumptive test used to differentiate Streptococcus pyogenes from other beta hemolytic streptococcus:
A. catalase test
C. bacitracin sensitivity test
B. optochin sensitivity test
D. oxidase test
(Answer: C / Reference: Microbiology p.419 / MPL 90)
19. The most common complication of pneumococcal pneumonia:
A. pleural effusion
C. meningitis
B. empyema
D. pericarditis
MICROBIOLOGY
Page 105 of 137
(Answer: A / Reference: Microbiology p.438 / MPL 60 )
20. Streptococcus agalactiae is normal flora of various parts of the body. Infection of newborns with this bacteria
is attributed to the mother harboring the organisms in the:
A. pharynx
C. gastrointestinal tract
B. vagina
D. skin
(Answer: B / Reference: Microbiology p.427 / MPL 60)
21. Neisseria gonorrhoeae can be differentiated from Neisseria meningitides by:
A. oxidase test
C. growth in the candle jar
B. fermentation of sugars
D. growth on chocolate agar
(Answer: B / Reference: Microbiology. p.445 / MPL 90)
22. The most common of the classic sexually transmitted disease:
A. syphilis
C. HIV
B. gonorrhea
(Answer: B / Reference: Microbiology p.454 / MPL 60)
D. Herpes
23. Carriers of Neisseria meningitides harbor the organisms in the:
A. throat
C. eyes
B. nasopharynx
D. lungs
(Answer: B / Reference: Microbiology p.449 / MPL 90)
24. The most common cause of acute bacterial meningitis in infancy and early childhood:
A. Hemophilus influenzae type B
C. Neisseria meningitides
B. Streptococcus pneumoniae
D. Escherichia coli
(Answer: A / Reference: Microbiology p.462 / MPL 60)
25. The systemic manifestations of pertussis is attributed to:
A. invasiveness of the organisms
C. circulating toxin
B. outer membrane proteins of the organism D. hemagglutinins
(Answer: C / Reference: Microbiology . p.477 / MPL 60)
26. Reported outbreaks of diarrhea, hemorrhagic colitis and HUS has been attributed to
coli strain linked to this outbreak:
A. EPEC
C. VTEC
B. EIEC
D. ETEC
(answer: C / Reference: Microbiology page: 548 / MPL 90)
Escherichia coli. The E.
27. The Escherichia coli strain identified to be the major cause(s) of traveler's diarrhea.
A. EPEC
C. EAEC
B. ETEC
D. B and C
(answer: D / Reference: Microbiology page: 548 / MPL 60 )
28. The ability of Klebsiella pneumoniae to resist phagocytosis and the killing power of the normal serum is due to
its:
A. endotoxin
C. capsule
B. flagella
D. cell wall
(answer: C / Reference: Microbiology page: 549 / MPL 90)
29. Urinary tract infection caused by urease producing organisms are characterized by an alkaline pH. This
cause precipitation of calcium and magnesium salts in the urine leading to production of:
A. urinary calculi
C. bacterial infection
B. hematuria
D. frequent scanty urination
(Answer: A/ Reference: Microbiology page: 552 / MPL 60)
30. The ability of Salmonella to attach to host receptor cells and survive intracellularly is attributed to its:
A. Vi antigen
C. H antigen
B. O antigen
D. A and B
MICROBIOLOGY
Page 106 of 137
(Answer: D / Reference: Microbiology page: 560 / MPL 60 )
31. The best time to perform blood culture in a patient with typhoid fever is during the:
A. first week of illness
C. third week of illness
B. second week of illness
D. fourth week of illness
(Answer: A / Reference: Microbiology page: 563 / MPL 90 )
32. The source of Salmonella typhi:
A. man
C. eggs
B. turtle
D. all of the above
(Answer: A / Reference: Microbiology page: 562 / MPL 90)
33. In the acute stage of shigellosis, the organisms are recovered from the:
A. blood
C. stool
B. urine
D. A and B
(Answer: C / Reference: Microbiology page: 537 / MPL 90)
34. One of the following is not true of Pseudomonas aeruginosa:
A. a common cause of nosocomial infection
B. susceptible to chemical disinfectants
C. resistant to first generation cephalosporin
D. survive with a minimum nutrient with adequate moisture
(Answer: B / Reference: Microbiology page: 577 / MPL 60)
35. To successfully culture Vibrio cholerae from stool of patients, the specimen should be cultured immediately or
placed in a suitable transport medium because a delay in processing of the specimen might lead to.
A. acidic pH of stool to which V. cholerae is sensitive to
B. dehydration of clinical specimens leading to death of the microorganisms
C. multiplication of coliforms which will outgrow the pathogen
D. A and B
(Answer: D / Reference: Microbiology page: 569 / MPL 60)
36. The following are true of Vibrio parahemolyticus:
A. infection is due to ingestion of raw or poorly cooked seafood
B. the short generation time allows the infective dose of 10 6 organisms to be easily attained in food
C. gastroenteritis is due to potent endotoxin it releases
D. A and B
(Answer: D / Reference: Microbiology page: 571 / MPL 60 )
37. Laboratory identification of H. pylori is based on:
A. tissue sections showing gram negative filamentous rod
B. positive urease test
C. demonstration of the organism in silver stain tissue section
D. B and C
(Answer: D / Reference: Microbiology page: 679 / MPL: 60 )
38. The most common vehicle of transmission of human listeriosis in both adults and neonates:
A. sick person
C. food
B. sick animal
D. water
(Answer: C / Reference: Microbiology page: 483 / MPL 60)
39. The systemic manifestations of diphtheria is essentially due to:
A. endotoxins
C. invasiveness of the organisms
B. exotoxins
D. metachromatic granules
(Answer: B / Reference: Microbiology page: 488 / MPL 90)
40. The simple staining procedure used in the laboratory to demonstrate the metachromatic granules of
Corynebacterium diphtheriae:
A. safranin staining
C. carbol fuchsin staining
B. crystal violet staining
D. methylene blue staining
(Answer: D / Reference: Microbiology page: 488 / MPL 60 )
MICROBIOLOGY
Page 107 of 137
41. The difference in the body response to primary and secondary tuberculosis is due to:
A. resistance
C. virulence of the organisms
B. delayed hypersensitivity
D. A and B only
(Answer: D/ Reference: Microbiology page: 506 / MPL 60)
42. The biochemical test used to identify the culture of Mycobacterium tuberculosis:
A. oxidase test
C. catalase test
B. niacin test
D. urease test
(Answer: B/ Reference: Microbiology page: 511 / MPL 90)
43. The usual source of secondary or re-infection tuberculosis:
A. endogenous
C. exogenous
B. organisms from BCG vaccine that became virulent D. none of the above
(Answer: A / Reference: Microbiology page: 506 / MPL 90)
44. The mode of transmission of Mycobacteria associated with non tuberculous infections:
A. man to man
C. inhalation of contaminated soil
B. reservoir animal to man
D. ingestion of contaminated food
(Answer: C / Reference: Microbiology page: 512 / MPL 60)
45. The MOTT which cause one of the common complication of AIDS:
A. M. kansasii
C. M. marinum
B. M. avium-intracellulare
D. M. fortuitum
(Answer: B / Reference: Microbiology page: 515 / MPL 90)
46. Mycobacterium leprae has been successfully grown in:
A. tissue culture
C. Lowenstein Jensen medium
B. footpads of mice
D. Petragnani medium
(Answer: B / Reference: Microbiology page: 516 / MPL 90 )
47. Diagnosis of leprosy is made by demonstration of acid fast bacilli in smears of the skin, nasal scrapings and
ear lobes. The bacilli are difficult and often impossible to detect in:
A. tuberculoid leprosy
C. borderline leprosy
B. borderline lepromatous leprosy
D. lepromatous leprosy
(Answer: A / Reference: Microbiology page: 518 / MPL 90 )
48. Nocardia asteroides in sputum smears appears as:
A. filamentous and branching
C. partially acid fast
B. gram positive
D. all of the above
(Answer: D / Reference: Microbiology page: 534 / MPL 90)
49. Bacteroides fragilis is known to be:
A. highly susceptible to penicillin
B. normal flora of the vagina and cervix
C. associated primarily with intrabdominal infection or septicemia
D. B and C
(Answer: C / Reference: Microbiology . p.631 / MPL 60)
50. The specimen that can be processed for anaerobic culture:
A. throat swab
C. blood
B. stool
D. sputum
(answer: C / Reference: Microbiology page: 629 / MPL 60 )
51. The most frequent anaerobic organism isolated from clinical specimens:
A. Bacteroides fragilis
C. Prevotella
B. Porphyromonas
D. Fusobacterium
(Answer: A / Reference: Microbiology page: 631 / MPL 90)
52. The following are true of Fusobacterium nucleatum EXCEPT:
A. most common species isolated from clinical specimens
B. organisms appear as thin bacilli with pointed ends microscopically
C. frequent flora of the urogenital tract
MICROBIOLOGY
Page 108 of 137
D. important agent in oral infection
(Answer: C / Reference: Microbiology page: 632 / MPL 60 )
53. The anaerobic organism which is not a pigment producer:
A. Porphyromonas
C. Prevotella
B. Fusobacterium
D. Bacteroides
(Answer: D / Reference: Microbiology page: 621 / MPL 60 )
54. The disease that is not attributed to Clostridium perfringens:
A. wound infection
C. food poisoning
B. pseudomembranous enterocolitis
D. clostridial myonecrosis
(Answer: B / Reference: Microbiology p.637 / MPL 60)
55. The signs and symptoms of tetanus is attributed to its:
A. endotoxin
C. exotoxin
B. production of hyaluronidase
D. invasiveness
(Answer: C / Reference: Microbiology p.645 / MPL 60)
56. The statements that are true with reference to Bacillus cereus infection:
A. clindamycin is the drug of choice
C. manifest as food poisoning
B. enterotoxin is a determinant of pathogenicity
D. all of the above
(Answer: D / Reference: Microbiology p.618 / MPL 60 )
57. Leptospirosis is acquired through:
A. ingestion of contaminated food
C. skin penetration of organisms
B. inhalation of microorganism
D. sexual contact
(Answer: C / Reference: Microbiology p.672 / MPL 90 )
58. Mycoplasma spp. is resistant to:
A. erythromycin
C. minocycline
B. penicillin
D. doxycycline
(Answer: B / Reference: Microbiology page: 736 / MPL 90 )
59. A syphilitic person with grnulomatous lesion in the mouth is:
A. in the second stage of the disease
B. positive for VDRL
(Answer: D / Reference: Microbiology p.662 / MPL 60)
C. highly infectious
D. FTA-ABS positive
60. The most common complication of Varicella is:
A. bacterial superinfection of the skin
C. cerebellar ataxia
B. postherpetic neuralgia
D. pneumonia
(Answer: C / Reference: Microbiology page: 960 / MPL 60 )
61. Which of the following is the common feature of herpesvirus?
A. ability to establish latency in the body after primary infection
B. ability to produce rashes in infected individuals
C. ability to cause cancer in humans
D. ability to affect nerve cells in humans
(Answer: A / Reference: Microbiology page: 955 / MPL 60)
62. Which of the following is true regarding adenovirus infection?
A. it may cause hemorrhagic cystitis
B. it is the most common viral respiratory pathogen in children
C. it is rarely associated with the common cold syndrome
D. it causes pneumonia that is less severe than that caused by parainlfuenza
(Answer: C / Reference: Microbiology page: 969 / MPL 60 )
63. A 25-year old patient had his blood tested for Hepatitis B yielding the following results:
HBs Ag (+)
HBe Ag (-)
Anti-HBc (+)
Anti-HBs (-)
Anti-HBe (+)
MICROBIOLOGY
Page 109 of 137
What is the most likely interpretation of the above results?
A. Early recovery phase from acute Hepatitis B
B. Has received Hepatitis B vaccination
C. Chronic Hepatitis B carrier state
D. Early acute Hepatitis B
(Answer: C / Ref: Baily & Scott 's Diagnostic Microbiology, 11 th ed p. 849 / MPL 60)
64. Poliovirus is spread from person to person thru:
A. inhalation of aerolized droplet
C. sexual transmission
B. fecal-oral route
D. skin penetration
(answer: B / Reference: Microbiology page: 981 / MPL 90 )
65. A 7 year old male patient was brought to the OPD reporting symptoms of mild fever and cough preceding the
onset of macular rash which appeared first on the face, then on the neck, trunk and extremities. The rashes
appeared to be discrete and rarely coalesce, Postauricular lymphadenopathy was also noted. The most
likely condition the patient has is:
A. rubella
C. varicella
B. rubeola
D. roseola
(Answer: A / Reference: Microbiology page: 9509,965,1013,1016 / MPL 60 )
66. Which of the following is true regarding dengue virus?
A. It is a DNA virus that causes fever and sometimes rash
B. It belongs to the family of Filoviruses causing hemorrhagic fever like Ebola virus
C. There are 3 serotypes thus there area 3 chances of having dengue in a lifetime
D. Subsequent infection with another dengue serotype is more severe than the previous one
(Answer: D / Reference: Baily & Scott 's Diagnostic Microbiology, 11 th ed page: 806-807 / MPL 60 )
67. The leading cause of diarrhea in the first 2 years of life among cases seen in hospitals and health centers
world wide.
A. rotavirus
C. Entamoeba histolytica
B. Escherichia coli
D. Shigella flexneri
(Answer: A / Ref: Readings on Diarrhea, 2nd printing June 1990 p. 19 / MPL 90)
68. The period of communicability for mumps usually lasts for about:
A. 24 hours
C. 3-6 days
B. 48 hours
D. 7-10 days
(Answer: D / Reference: Microbiology page: 1007 / MPL 33)
69. The incubation period of measles is:
A. 1-2 days
C. 7-9 days
B. 3-6 days
D. 10-14 days
(Answer: D / Reference: Microbiology page: 1012 / MPL 60 )
70. The cornerstone of human rabies prevention in most areas of the world is:
A. killing of stray dogs and cats
B. vaccination of domestic dogs and cats
C. spaying of female dogs and cats
D. vaccination of persons with pet dogs and cats
(Answer: B / Reference: Microbiology page: 1029 / MPL 60)
71. Which of the following statements is correct regarding influenza vaccine?
A. protection is conferred as early as 1 month after vaccination
B. it should also be given to caregivers of elderly individuals in nursing homes.
C. protection will last for 10-15 years
D. at present the vaccines are monovalent
(Answer: B / Reference: Microbiology page: 496-497 / MPL 60)
72. Confirmatory test for Human Immunodeficiency Virus (HIV):
A. Western blot
C. Southern blot
B. Polymerase Chain Reaction (PCR)
D. ELISA test
MICROBIOLOGY
Page 110 of 137
(Answer: A; Ref: Baily & Scott 's Diagnostic Microbiology, 11 th ed p. 832 / MPL 60)
73. The leading cause of posttransfusion heaptitis is:
A. Hepatitis A
C. Hepatitis C
B. Hepatitis B
D. Hepatitis D
(Answer: C / Reference: Microbiology page: 1043 / MPL 90 )
74. Routine fungal cultures are incubated at:
A. 28oC
C. 37oC
B. 35oC
D. 42oC
(Answer: A / Reference: Microbiology page: 1087 / MP :90 )
75. The most common pathogenic species of Aspergillus for humans:
A. Aspergillus niger
C. Aspoergillus terreus
B. Aspergillus fumigatus
D. Aspergillus flavus
(Answer: B / Reference: Microbiology p.1147 / MPL 60)
76. The fungi whose identification is based on microscopic picture of the skin scrapings
and not on culture of the fungus.
A. Microsporum canis
C. Malassezia furfur
B. Exophiala werneckii
D. Epidermophyton floccosum
(Answer: C / Reference: Microbiology p.1131 / MPL 60)
77. The frequent cause of tinea pedis:
A. Microporum canis
C. Epidermophyton floccosum
B. Trichophyton mentagrophytes
D. Candida albicans
(Answer: B / Reference: Microbiology p.1130 / MPL 60)
78. The appearance of pseudohyphae or true hyphae along with budding yeast cells in tissue is pathognomonic
for:
A. Aspergillosis
C. Mucormycosis
B. Cryptococcosis
D. Candidiasis
(Answer: D / Reference: Microbiology p.1142 / MPL 90)
79. The most frequent cause of opportunistic mycosis:
A. Candida albicans
C. Aspergillus spp.
B. Cryptococcus neoformans
D. Penicillium spp
(Answer: A / Reference: Microbiology page: 1043 / MPL 90 )
80. The most important source of Cryptococcus infection in man:
A. pigeon excreta
C. infected person
B. infected dogs
D. contaminated food
(Answer: A / Reference: Microbiology p. 1144 / MPL 90 )
81. Diarrheic stools of patients suspected with Entamoeba histolytica infection must be examined within:
A. 15 minutes
C. 3-5 hours
B. 30 minutes to 60 minutes
D. anytime
(Answer: B / Reference: Parasitology p.104 / MPL 60)
82. For diagnosis of amoebiasis, ideally stool exam should be done:
A. examinations of 3 specimens every other day
C. daily for 3 days
B. examinations of 3 specimens every 2nd or 3rd day
D. weekly for one month
(Answer: B / Reference: Parasitology p.119 / MPL 60)
83. The incorrect statement regarding Trichomonas vaginitis
A. this is usually acquired through sexual intercourse
B. female patients are usually symptomatic
C. in men the infection is usually asymptomatic
D. diagnosis is based on recovery of the cystic stage from genital discharges
(Answer: D / Reference: Parasitology p.50-51; MPL 60 )
84. True relapse in malaria is:
MICROBIOLOGY
Page 111 of 137
A. due to activation of surviving erythrocytic parasites
B. due to activation of hypnozoites
C. seen in Plasmodium falciparum infection
D. seen in carriers of Plasmodium malariae
(Answer: B / Reference: Parasitology p. 197 / MPL 33)
85. The clinical manifestations of malaria is attributed to:
A. destruction of parasitized red blood cells
C. appearance of gametocytes
B. released of merozoites from hepatocytes
D. merozoites enter the RBC
(Answer: A / Reference: Parasitology p.194 / MPL 60 )
86. The nematode whose larva does not undergo lung migration:
A. Ascaris lumbricoides
C. Capillaria philippinensis
B. Necator americanus
D. Strongyloides stercoralis
(Answer: C / Reference: Parasitology p. 246 / MPL 90)
87. The filariform larva is not the infective stage to man of:
A. Necator americanus
C. Ascaris lumbricoides
B. Ancylostoma duodenale
D. Strongyloides stercoralis
(Answer: C / Reference: Parasitology p. 278. / MPL 90)
88. Demonstration of Enterobius vermicularis eggs is best done by:
A. direct fecal examination
C. rectal swab
B. scotch tape method
D. concentration method
(Answer: B / Reference: Parasitology p. 304 / MPL 90)
89. Allergic manifestations of ascariasis include asthma, and urticaria are seen in the following conditions:
A. pulmonary migration of the parasite
C. intestinal phase of ascariasis
B. when pulmonary phase wanes
D. all of the above
(Answer: D / Reference: Parasitology . p.314 / MPL 60 )
90. The microfilaria of Wuchereria bancrofti are best recovered from the:
A. blood
C. CSF
B. lymph
D. urine
(Answer: A / Reference: Parasitology p.362 / MPL 90)
91. The intermediate host of Schistosoma japonicum in the Philippines:
A. Oncomelania quadrasi
C. Bortia asperata
B. Pila luzonica
D. Gyraulus convexiusculus
(Answer: A / Reference: Parasitology p.416-417 / MPL 60)
92. The current treatment for schistosomiasis:
A. Thiabendazole
C. Praziquantel
B. Mebendazole
D. Metronidazole
(Answer: C / Reference: Parasitology p.424 / MPL 90 )
93. The infective stage to man of Echinostoma ilocanum is:
A. encysted metacercaria in snail
C. cercaria
B. encysted metacercaria in fresh water fish
D. embryonated operculated egg
(Answer: A / Reference: Parasitology p.458 / MPL 60)
94. Heterophyes heterophyes infection can lead to cardiac failure as a result of the:
A. adult fluke producing granuloma in the heart
B. metacercaria developing to adult worm in the heart
C. eggs blocking the vessels leading to the heart
D. eggs cause myocarditis
(Answer: D / Reference: Parasitology p.479; MPL 33)
95 . Paragonimus westermani infection can mimick pulmonary tuberculosis and other pulmonary problems.
Laboratory diagnosis of Paragonimus westermani infection is by demonstration of the eggs from the:
A. sputum
C. stool
B. urine
D. A and C
MICROBIOLOGY
Page 112 of 137
(Answer: D / Reference: Parasitology p.466; MPL 60)
96. Cysticercus cellulosae is the larva of:
A. Diphyllobothrium latum
C. Taenia solium
B. Hymenolepis nana
D. Taenia saginata
(Answer: C / Reference: Parasitology p.516 / MPL 60)
97. Diphyllobothrium latum is transmitted through ingestion of:
A. fresh water fish with plerocercoid larva
C. water contaminated with eggs
B. fresh water fish with procercoid larva
D. water with infected Cyclops
Reference: Parasitology . p.497; MPL 60)
(Answer: A /
98. The only human tapeworm which does not require an intermediate host:
A. Hymenolepis diminuta
C. Echinococcus granulosus
B. Hymenolepis nana
D. Multiceps multiceps
(Answer: B / Reference: Parasitology p.511 / MPL 60)
99. A child with a pet dog, passed out gravid proglottids with genital atrium on each lateral margin. This cestode
is:
A. Hymenolepis nana
C. Raillietina garrisoni
B. Bertiella studeri
D. Dipylidium caninum
(Answer: D / Reference: Parasitology . p.508 / MPL 60 )
100. The cestode whose proglottids detach from the worm singly and are capable of independent motion crawling
out of the anus during the daytime while the patient is active:
A. Diphyllobothrium latum
C. Taenia solium
B. Hymenolepis nana
D. Taenia saginata
(Answer: D / Reference: Parasitology p.521 / MPL 60)
Medical Parasitology Section
MULTIPLE CHOICE. Choose the letter of the best answer.
1. Which of these findings on “Enterotest” would a person with Giardia lamblia infection have?
positive for Giardia trophozoites
positive for Giardia cysts
positive for mucosal ulcerations
negative result
- application
2. You have a 36 year old female patient (in a monogamous relationship) infected with Trichomonas vaginalis. You
would advise her to:
A. take Metronidazole and bring her partner for simultaneous treatment.
B. take Metronidazole and abstain from sex while being treated.
C. take Metronidazole.
D. take metronidazole and bring her partner for counseling.
-application
3. The trophozoite of this organism has a “falling-leaf” motility:
A. Trichomonas vaginalis
B. Trichomonas hominis
C. Chilomastix mesnilii
D. Giardia lamblia
-recall
4. A 24-year old male patient from Northern Philippines consulted you because of fever and chills. Peripheral blood
smear revealed microcytic, hypochromic anemia with some blood parasites labeled as asexual forms with unusually
big infected red cells. This patient most likely has ___________ malaria.
A. Falciparum
MICROBIOLOGY
Page 113 of 137
B.
C.
D.
Vivax
Malariae
Ovale
-analysis
5. The best time to get a blood sample for diagnosis in a suspected case of malaria is:
A. before the height of fever.
B. at the peak of fever.
C. after fever.
D. any time of the day.
-comprehension
6. Recurrence of paroxysms in untreated cases of falciparum infection is expected to occur after _____ year(s).
A. one
B. two to three
C. five to ten
D. thirty
-comprehension
7. . In the management of uncomplicated falciparum malaria one should give
A. chloroquine only
B. primaquine only
C. quinine only
D. chloroquine and primaquine
-recall
8. The degree of parasitemia is expected to be highest in infection with Plasmodium:
A. vivax
B. malariae
C. falciparum
D. ovale
-comprehension
9. Toxocara infection causes:
A. cutaneous larva migrans.
B. visceral larva migrans.
C. elephantiasis.
D. migratory edema.
-recall
10. Cellulose tape method in the diagnosis of Oxyuriasis is best done during the night because:
A. the child is sleeping and it will be easier to collect.
B. the gravid female goes to the perianal area to lay eggs.
C. the gravid females will die in the morning.
D. the perianal area has not been washed yet.
-comprehension
11. Loeffler’s syndrome is associated with:
A. Ascariasis
B. Toxocariasis
C. Enterobiasis
D. Anisakiasis
-recall
MICROBIOLOGY
Page 114 of 137
12. Humans can be infected by this nematode through the ingestion of infected raw codfish:
A. Toxocara
B. Ascaris
C. Anisakis
D. Enterobius
-recall
13. A 12 year old male is brought to the ER because of changes in sensorium two days after a swimming outing. A
Few hours ago, the patient complained of severe headache which was followed by fever, headache and vomiting.
The amoeba most probably causing this presentation is:
A. Acanthamoeba
B. Naegleria
C. Balantidium
D. Entamoeba
-analysis
14. Manual extraction of the worm is done gradually when the head of the adult worm comes out of the skin blister.
This is done in infection with:
A. Angiostrongylus cantonensis
B. Angiostrongylus costaricensis
C. Dracunculus medinensis
D. Gnathostoma spinigerum
-comprehension
15. Management includes the removal of a few ml of cerebrospinal fluid to relieve the patient of severe headache
A. Angiostrongylus cantonensis
B. Angiostrongylus costaricensis
C. Dracunculus medinensis
D. Gnathostoma spinigerum
This item tests for “comprehension” skill in relation to objective pertaining to management.
16. The eggs are ovoid and have a single polar thickening or a mucoid plug.
A. Angiostrongylus cantonensis
B. Angiostrongylus costaricensis
C. Dracunculus medinensis
D. Gnathostoma spinigerum
-recall
17. This parasite moves by means of pseudopodia:
A. Entamoeba histolytica
B. Giardia lamblia
C. Balantidium coli
D. Leishmania donovani
-recall.
18. Chilomastix mesnili inhabits its host’s:
A. duodenum, jejunum & upper ileum
B. vaginal and urethral tissues
C. cecum and colon
D. cavities of carious teeth
-recall
19. Complications of infection with this parasite are severe hemorrhage, peritonitis and abscess of the liver:
A. Endolimax nana]
B. Entamoeba histolytica
C. Iodamoeba buetschli
MICROBIOLOGY
Page 115 of 137
D. Chilomastix mesnili
-comprehension
20. A 25 year old female developed vague abdominal discomfort and persistent diarrhea a few hours after eating
roasted pig in a town fiesta. After consultation and stool examination, he was prescribed Praziquantel. The infection is
most likely due to:
A. Taenia solium
B. Taenia saginata
C. Echinococcus granulosus
D. Dipylidium caninum
-analysis
21. Which of the following is true regarding concentration technique in the diagnosis of amoebiasis?
A. The test is more sensitive in the identification of trophozoites.
B. Zinc sulfate is preferred over acid ether conecntration.
C. This is a preferred method especially in the detection of carrier state.
D. This is more sensitive than Indirect Hemagluttination Assay in the diagnosis of intestinal amoebiasis.
-application
22. Carcinoma of the bile passages or of the pancreas with metastasis into epigastric lymph nodes is most commonly
associated with:
A. Fasciola hepatica
B. Clonorchis sinensis
C. Opisthorchis felineus
D. None of the above
-comprehension
23. A patient from a sheep and cattle raising country developed enlarged and tender liver, jaundice, digestive
disturbances and diarrhea. He has history of eating watercress salad a few days prior to onset of symptoms. He is
most likely infected with:
A. Fasciola hepatica
B. Opisthorchis felineus
C. Opisthorchis viverinii
D. Clonorchis sinensis
-analysis
24. Halzoun is associated with which of the following trematode infections?
A. Fascioliasis
B. Fasciolopsiasis
C. Echinostomiasis
D. Heterophyiasis
-recall
25. Developmental stage of Fasciola hepatica infective to the 1st intermediate host
A. egg
B. miracidium
C. cercaria
D. metacercaria
=recall
26. Developmental stage of Fasciola hepatica released by the 1 st intermediate host into the water
A. egg
B. miracidium
C. cercaria
D. metacercaria
-recall
MICROBIOLOGY
Page 116 of 137
27. Among the female Schistosomes, the one which is most prolific in terms of its egg-laying capacity is Schistosoma:
A. japonicum
B. hematobium
C. mansoni
D. indicum
=comprehension
28. The adult form of this Schistosome mainly inhabits the superior mesenteric plexuses:
A. japonicum
B. hematobium
C. mansoni
D. indicum
-comprehension
29. A 57 y/o male underwent cystectomy because of a 2.0x2.0cm. solid mass which on histopathologic examination
revealed a malignant neoplasm. This condition is most associated with Schistosoma:
A. japonicum
B. hematobium
C. mansoni
D. indicum
-analysis.
30. A 20 year old male patient sought consulted due to fever with hepatosplenomegaly and urticarial rashes. He has
history of swimming in a pond three weeks ago followed by development of transient pruritus and skin rashes. He
recalled that three weeks ago there was appearance of transient pruritus and skin rashes after thru ricefields. The
following are true of this case EXCEPT:
A. Infection was acquired through skin penetration of the parasite .
B. Eggs can be the diagnostic tool for identification of the parasite .
C. The probable parasite needs at least two intermediate hosts to survive.
D. Cercariae is the infective stage to humans.
-analysis
31. The males have a gynecophoral canal :
A. Schistosoma mansoni
C. Heterophyes heterophyes
B. Echinostoma ilocanum
D. Opisthorchis viverinii
PRC MICROBIOLOGY & PARASITOLOGY
1.
A patient complains of an “itching rash” on her abdomen. On examination, the lesions were found to be red,
circular; with a vesiculated border and a healing central area. The most appropriate laboratory procedure to
make a diagnosis is a:
A. Potassium hydroxide mount of skin scrapings
B. Giemsa stain for multinucleated cells
C. Fluorescent antibody stain of the vesicle fluid
D. Four-fold rise in antibody titer against the organism
2.
During the first week of the disease Leptospirosis, the most reliable way to detect the presence of the causative
agent is:
A. Culturing the urine
B. Examination of cerebrospinal fluid
C. Culturing of blood
D. Culturing of kidney tissue
3.
A hospital worker is found to have hepatitis B surface antigens. Subsequent test reveal the presence of e
Antigen as well. The worker most likely is:
A. Infective and has active hepatitis
B. Infective but does not have active hepatitis
MICROBIOLOGY
Page 117 of 137
C.
D.
Not infective
Evincing a biologic false-positive test for hepatitis
4.
A patient’s serum yields a positive RPR & a negative FTA. These results most likely indicate:
A. Reactive for syphilis
B. False negative FTA due to low sensitivity of the test
C. Biological false-positive RPR due to cross-reacting antibodies
D. Patient in remission for syphilis
5.
A positive ELISA test for HIV-1 & a Western blot test with bands at p24 & gp41 most likely indicate:
A. False-positive ELISA & inconclusive Western blot
B. Positive ELISA & Western blot; indicating HIV infection
C. False positive ELISA & Western blot
D. Lack of HIV infection
6.
A Staphylococcus produces a fibrin clot in the tube coagulase test, but not in the slide coagulase test. This
organism:
A. Produces only free coagulase & is most likely Staphylococcus aureus
B. Produces only bound coagulase & is most likely Staphylococcus aureus
C. Is most likely Staphylococcus epidermidis because of the negative slide test
D. Is most likely Staphylococcus epidermidis since the slide test in unreliable
Page 1 of 17
MICROBIOLOGY & PARASITOLOGY
7.
If you start out with a population density of 200 CFU/ml of a bacterium that divides every 20 minutes, what will be
the population density at the end of two hours, assuming the cells are in the log phase of growth:
A. 1200 CFU/ml
B. 26 CFU/ml
C. 3200 CFU/ml
D. 2006 CFU/ml
8.
The presence of low levels of IgG but not IgM antibodies to rubella virus in the newborn suggests:
A. Presence of maternal antibodies
B. Susceptibility to rubella virus infection
C. Congenital infection
D. Persistent infection
9.
Despite feeling healthy, a fourth-year medical student has developed a positive PPD skin test after three years of
negative results. The most likely interpretation of this result would be that the student has:
A. Reacted as a result of a BCG vaccine given in childhood
B. An HIV infection which may give a false-positive PPD test
C. Secondary tuberculosis as a result of the activation of a long standing primary infection
D. Been exposed to a person with active tuberculosis and has developed a primary tuberculosis infection
10. Penicillin is added to a culture of a Gram-positive rod. Twelve hours later, the number of bacteria has increased,
and the organisms have assumed a round configuration. The most likely explanation for these observations is
that the:
A. Concentration of penicillin was insufficient
B. Organism carries a mutated transpeptidase
C. Organism is deficient in autolytic activity
D. Organism secretes penicillin
11. A 35-year old migrant worker is admitted to the hospital with a high fever and malaise. When he started feeling
weak 3 days ago, he checked his temperature and found it to be slightly elevated. Over the last 3 days, he has
felt progressively worse and his temperature has reached 39.4ºC. The resident on call in the emergency room
MICROBIOLOGY
Page 118 of 137
observes a scanty maculopapular rash on the patient’s trunk. The patient mentions that another worker in his
group has been sick with similar symptoms. Which one of the following tests would most likely yield a diagnosis:
A. Blood cultures
B. Methylene blue staining of a fecal extract
C. Serologic tests for mumps virus, rubella virus, coxsackie viruses and echovirus
D. Stool cultures
Page 2 of 17
MICROBIOLOGY & PARASITOLOGY
12. Which one of the following procedures is best for decontaminating a heat-sensitive reusable piece of
equipment?:
A. Ethylene oxide gas sterilization
B. Pasteurization
C. Soaking in 3% hydrogen peroxide
D. Soaking in alcohol
13. An alginate-producing, aerobic, oxidase-positive, Gram-negative bacillus is isolated from the sputum of a patient
with cystic fibrosis. This organism is likely to be:
A. Bacteroides fragilis
B. Escherichia coli
C. Klebsiella pneumoniae
D. Pseudomonas aeruginosa
14. A laboratory technician has isolated an organism that grows in blood agar, forms round, smooth, pale yellow
colonies surrounded by a narrow area of hemolysis, ferments mannitol, and is both catalase- and coagulasepositive. This organism is:
A. Streptococcus pyogenes
B. Staphylococcus aureus
C. Staphylococcus epidermidis
D. Staphylococcus saprophyticus
15. Cultures of two neomycin-sensitive strains of Escherichia coli are mixed and cultured in the presence of
neomycin. After 2 hours, the mixture is plated on neomycin-containing agar. A few neomycin-resistant colonies
grow. The emergence of these resistant strains most likely resulted from:
A. Conjugation
B. Spontaneous mutation
C. Transduction
D. Transformation
16. An obstetrician sees a pregnant patient who was exposed to rubella virus in eighteenth week of pregnancy. She
does not remember getting a rubella vaccination. The best immediate course of action is to:
A. Terminate pregnancy
B. Order a rubella antibody titer to determine immune status
C. Administer rubella immune globulin
D. Administer rubella vaccine
17. A nurse develops clinical symptoms consistent with hepatitis. She recalls sticking herself with a needle
approximately 4 months before after drawing blood from a patient. Serologic tests for HBsAg, antibodies to
HBsAg, and hepatitis A virus (HAV) are all negative; however, she is positive for IgM core antibody. The nurse:
A. Does not have hepatitis B
B. Has hepatitis A
C. Is in the “window” phase
D. Has hepatitis C
Page 3 of 17
MICROBIOLOGY & PARASITOLOGY
18. There are millions of cases of leprosy (Hansen’s disease) worldwide, but predominantly in Asia and Africa. The
clinical spectrum of Hansen’s disease is best characterized by:
A. Immunologic anergy
B. Chronic pneumonitis
MICROBIOLOGY
Page 119 of 137
C.
D.
Peripheral neuritis
Erythematous lesions
19. A 6-year old girl presents to the clinic with scaly patches on the scalp. Primary smears and culture of the skin
and hair were negative. A few weeks later, she returned and found to have inflammatory lesions. The hair
fluoresce under Wood’s light and primary smears of skin and hair contained septate hyphae. On speaking with
the parents, it was discovered that there were several pets in the household. Which of the following is the most
likely agent?:
A. Microsporum audouinii
B. Microsporum canis
C. Trichophyton tonsurans
D. Epidermophyton floccosum
20. One of the most remarkable aspects of human immune system is its diversity, that is, the ability to recognize a
wide range of antigens and to mount a specific antibody response. This is called clonal selection. At the cellular
level, which of the following is primarily responsible for such specificity?:
A. Cyctotoxic T cells
B. Hypervariable regions in domains of B cells
C. Specific T cell receptors
D. Memory cells
21. Relative to the primary immunological response, secondary and later booster responses to a given haptenprotein complex can be associated with which of the following?:
A. Lower titers of antibody
B. Decreased antibody avidity for the original hapten-protein complex
C. Increased antibody affinity for the hapten
D. Antibodies that are less efficient in preventing specific disease
22. Which one of the following best describes the mode of action of endotoxin?:
A. Degrades lecithin in cell membranes
B. Inactivation of elongation factor-2
C. Blocks release of acetyl choline
D. Causes the release tumor necrosis factor
23. The major role of T cells in the immune response includes:
A. Recognition of epitopes presented with MHC molecules
B. Complement fixation
C. Phagocytosis
D. Production of antibodies
4 of 17
MICROBIOLOGY & PARASITOLOGY
24. A culture of skin lesions from a patient with pyoderma (impetigo) shows numerous colonies surrounded by a
zone of beta hemolysis on a blood agar plate. A Gram-stained smear shows gram-positive cocci. If you found the
catalase test to be negative, which one of the following organisms would you most probably have isolated?:
A. Streptococcus pneumoniae
B. Staphylococcus aureus
C. Staphylococcus epidermidis
D. Streptococcus pyogenes
25. Five hours after eating fried rice at a restaurant a 24-year old woman and her husband both developed nausea,
vomiting and diarrhea. This is most likely due to:
A. Clostridium perfringens
B. Escherichia coli
C. Bacillus cereus
D. Salmonella enteritidis
26. Escherichia coli has two major porins located in the outer membrane. The function of porins is:
A. Stabilization of the mesosome
B. Metabolism of phosphorylated intermediates
C. Transfer of small molecules through the outer membrane
MICROBIOLOGY
Page 120 of 137
D.
Serologic stabilization of the O antigen
27. A recently hired laboratory technologist forgets the iodine-fixation step while performing a Gram’s stain on a
strain of Staphylococcus. The most likely result is that the organisms would:
A. Appear pink
B. Appear blue
C. Be colorless
D. Wash off the slide
28. Your patient is a woman with vaginal discharge. You suspect on clinical grounds, that it may be due to Candida
albicans. Which of the following statements is true?:
A. A Gram stain of the discharge should reveal an encapsulated yeast cells
B. Culture of the discharge on Sabouraud’s agar should produce a white mycelium with aerial conidia
C. To identify the organism, you should determine whether germ tubes are produced
D. Dematiaceous, septated hyphae should be seen together with the yeast cells
29. Fungal cells that reproduce by budding are seen in the infected tissues of patients with:
A. Candidiasis, Cryptococcosis, and Sporotrichosis
B. Mycetoma, Candidiasis, and Mucormycosis
C. Tinea corporis, Tinea unguium, and Tinea versicolor
D. Sporotrichosis, Mycetoma, and Aspergillosis
Page 5 of 17
MICROBIOLOGY & PARASITOLOGY
30. Bacteriologic examination of the cerebrospinal fluid (CSF) of a 6-year old girl with suspected meningitis results in
the isolation of a Gram-negative diplococcus that grows in chocolate agar and is oxidase positive. Which type of
antibacterial should be selected for the treatment of this patient?:
A. cause misreading of bacterial messenger RNA
B. inactivate DNA-dependent RNA polymerase
C. inhibit peptidoglycan cross-linking
D. inhibit DNA gyrase
31. A 22-year old woman developed urinary tract infection during her honeymoon. At the time she sought medical
advice, she was febrile and complained of painful urination and flank pain. Her urine appeared “cloudy.” Urine
culture yields a lactose-fermenting, indole-positive, Gram-negative bacillus. The infectiveness of the organism
responsible for this urinary tract infection is associated with specific:
A. exotoxins
B. P fimbriae
C. K antigens
D. plasmids
32. The survival of Mycobacteria after ingestion by macrophages is attributed to:
A. bacterial inhibition of complement activation via the alternative pathway
B. bacterial inhibition of phagolysosome formation and interference with endosomal acidification
C. the poor immunogenecity of the cell wall glycolipids
D. the bacterium’s rapid escape from the endosome into the cytoplasm of infected cells
33. A Gram-negative bacterium is isolated from a patient’s cerebrospinal fluid (CSF). It grows on enriched chocolate
agar but does not grow on blood agar except adjacent to a streak of staphylococci. The organism is most
probably:
A. Neisseria meningitidis
B. Neisseria gonorrheae
C. Haemophilus influenzae
D. Listeria monocytogenes
34. A gardener pricked his toe while cutting rose bushes. Four days later a pustule that changed to an ulcer
developed on his toe. Then 3 nodules formed along the local lymphatic drainage. The most likely agent is:
A. Sporothricum schenckii
B. Trichophyton rubrum
C. Aspergillus fumigatus
MICROBIOLOGY
Page 121 of 137
D.
Candida albicans
Page 6 of 17
MICROBIOLOGY &PARASITOLOGY
35. A 24-year old construction worker, who has had four injections of the DPT vaccine in his first year of life and
boosters at ages 5 & 19, received a deep laceration while excavating a building’s foundation. The preferred
method of treatment would be:
A. An aminoglycoside antibiotic
B. Human tetanus immunoglobulin, because it will stimulate his anamnestic response
C. Equine tetanus immune globulin, because it will passively immunize him.
D. Tetanus toxoid, because it will stimulate his anamnestic response
36. A Gram stain from the purulent drainage of a synovial fluid reveals numerous gram-negative diplococci. The
organism grew on chocolate agar and Thayer Martin medium but failed to grow on blood agar or MacConkey
agar. Further testing revealed the organism to be oxidase positive with positive utilization of glucose. All other
carbohydrates were negative. The beta lactamase result was positive. The organism is most likely:
A. Streptococcus pneumoniae
B. Neisseria gonorrheae
C. Neisseria meningitidis
D. Moraxellal catarrhalis
37. Beta-hemolytic streptococci were isolated from the throat culture of a 15-year old male. Select the best group of
tests to identify this organism:
A. Bacitracin and SXT
B. Bacitracin and CAMP
C. Bile esculin and PYRase
D. Optochin and CAMP
38. Salmonella has been isolated as a source of contamination in an outbreak of gastroenteritis associated with
eggs. Which of the following reactions is correct for this genus?:
A. H2S positive, urease negative, non-motile
B. H2S positive, urease negative, motile
C. H2S negative, urease positive, motile
D. H2S positive, urease positive, non-motile
39. An organism isolated from an anaerobic blood agar was subcultured onto both aerobic blood agar, which was
incubated aerobically and anaerobic blood agar, which was incubated anaerobically. After 48 hours, growth was
evident only on the anaerobic blood agar plate. This organism is most likely:
A. An obligate aerobe
B. An obligate anaerobe
C. A facultative anaerobe
D. Capnophilic
Page 7 of 17
MICROBIOLOGY & PARASITOLOGY
40. As compared to a primary response, a typical anamnestic response yields:
A. About the same antibody level but the level persists longer
B. More antibody but the level is lower to develop
C. A much higher antibody level that also persists longer
D. A higher antibody level that declines much more rapidly
41. Most polar compounds cross the outer membrane of Gram negative bacteria by:
A. Simple diffusion through the lipid bilayer
B. Diffusion through porins
C. Facilitated diffusion via specific carriers
D. Active transport using ATP in the periplasm
42. Individual prokaryotic cells grow:
A. Slowly at first, then more rapidly
B. At a constant rate throughout the cell cycle
MICROBIOLOGY
Page 122 of 137
C.
D.
Rapidly at first, then more slowly
In brief spurts, interspersed with periods of non-growth
43. In terms of temperature optimums for growth, which group encompasses the human pathogens?:
A. Psychrophile
B. Mesophile
C. Thermophile
D. Hyperthermophile
44. Pili contribute to bacterial invasiveness by:
A. Helping pathogens attach to host cells
B. Inhibiting phagocytosis
C. Dissolving the extracellular matrix
D. Inhibiting phagosome-lysosome fusion
45. The symptom of diffuse, watery diarrhea that produces a relatively clear stool containing mucus flecks (rice water
stool) is most closely associated with an infection caused by:
A. Escherichia coli
B. Shigella dysenteriae
C. Vibrio cholerae
D. Campylobacter jejuni
46. A clinical problem has emerged concerning infections after prosthetic heart valve insertion or other cardiac
procedures with methicillin-resistant strains of:
A. Staphylococcus epidermidis
B. Serratia marcescens
C. Streptococcus salivarius
D. Enterococcus faecalis
47. Shigella sonnei is differentiated from the other species by:
A. Its ability to ferment lactose
B. Its positive phenylalanine deaminase reaction
C. Its negative oxidase reaction
D. Its ability to demonstrate motility at 42ºC
Page 8 of 17
MICROBIOLOGY & PARASITOLOGY
48. The optimal specimen for the recovery of Bordetella pertussis is:
A. Coughed sputum
B. Blood
C. Anterior nares swab
D. Nasopharyngeal swab
49. A gram-negative bacillus was recovered from the urine of a child with a history of recurrent urinary tract
infections. The organism was oxidase negative, lactose negative, urease positive, and motile. The most likely
identification of this agent would be:
A. Proteus mirabilis
B. Escherichia coli
C. Pseudomonas aeruginosa
D. Enterobacter aerogenes
50. The enzyme produced by Helicobacter pylori that appears to be important for survival of the organism in the
stomach is:
A. Protease
B. Urease
C. Adenylate cyclase
D. Alkaline phosphatase
51. A microaerophilic curved rod, which frequently causes enteritis in humans, resides in the intestinal and
reproductive tracts of animals:
A. Vibrio cholerae
MICROBIOLOGY
Page 123 of 137
B.
C.
D.
Salmonella choleraesuis
Shigella flexneri
Campylobacter jejuni
52. The most appropriate method for sterilizing a bacteriologic medium is:
A. Autoclave
B. Ethylene oxide application
C. Filtration
D. Use of dry heat
53. Urease production by enteric bacteria involved with urinary tract infections is clinically significant because:
A. The enzyme destroys the epithelial lining of the bladder
B. Products of urease activity cause an alkalinization of the urine and precipitation of calcium and magnesium
phosphates
C. The enzyme facilitates migration of bacteria into the kidneys
D. The enzyme is required for adherence of bacteria to uroepithelial cells
54. The highest incidence of meningococcal infection occurs in which age group?:
A. 0 to 6 months of age
B. 6 to 24 months of age
C. 2 years to 5 years of age
D. 5 years to 21 years of age
Page 9 of 17
MICROBIOLOGY & PARASITOLOGY
55. The agent for artificial immunization against Clostridium tetani consists of:
A. Purified capsular antigen
B. Purified cell-wall protein
C. An attenuated live organism
D. A toxic protein treated with formaldehyde
56. An individual experiences severe diarrhea after eating sushi in a restaurant. The most probable cause of the
problem is:
A. Salmonella enteritidis
B. Campylobacter jejuni
C. Vibrio parahaemolyticus
D. Shigella sonnei
57. The lowest amount of antibiotic that results in 99.9% in vitro killing of the organism is the:
A. Minimal bacteriostatic concentration
B. Serum bactericidal level
C. Serum peak level
D. Minimal bactericidal concentration
58. Propionibacterium acnes were isolated in one tube of a set of three blood cultures. The other two tubes revealed
no growth and no organisms were seen on the Gram stain. This most likely indicates:
A. Septicemia from the organism
B. Failure to inoculate the negative bottles properly with blood
C. Skin contamination from improper disinfection before venipuncture
D. None of the above
59. The iodine stain for the identification of Chlamydia trachomatis stains:
A. The glycogen in the inclusion brown
B. The periplasmic space purple
C. The cell membrane brown
D. The DNA in the inclusion purple
60. An infant was seen in the emergency room with symptoms of neuromuscular weakness and constipation. The
diagnosis of infant botulism was confirmed by the demonstration of toxin in the child’s stool. The child most likely
contracted the disease by:
A. Ingestion of spores that germinated in the intestine
MICROBIOLOGY
Page 124 of 137
B.
C.
D.
A puncture wound with contaminated household item
Ingestion of preformed toxin found in a contaminated jar of pureed vegetables
Inhalation of contaminated fomites
Page 10 of 17
MICROBIOLOGY & PARASITOLOGY
61. Each of the following statements correctly describes the mycoplasmas, EXCEPT:
A. They contain both DNA and RNA
B. They do not have a cell wall
C. They cannot replicate on their own
D. They are the smallest free living organisms known
62. The synergistic action of sulfonamides and trimethoprim is related to which of the following?
A. Inhibition of beta-lactamase
B. Inhibition of sequential steps in tetrahydrofolate synthesis
C. Altered penicillin-binding proteins
D. Inhibition of DNA gyrase
63. The characterization of spirochetes relies primarily on:
A. Metabolic end products
B. Cell wall constituents
C. Carbohydrate degradation
D. Morphology
64. Chronic carriers, persons who remain infected with an organism for long periods, are typically associated with
the dissemination of:
A. Salmonella typhi
B. Corynebacterium diphtheriae
C. Streptococcus pneumoniae
D. Bordetella pertussis
65. A college student got a summer job working at a marina. While working on one of the outboard motors on a
rental boat, he received several lacerations on his right forearm. No medical treatment was sought at the time of
the injury but after several weeks he noted that the lesions were not healing and sought the opinion of his
physician. A biopsy of one of the lesions showed it to be a cutaneous granulomatous condition. Given the
history, which of the following microorganisms would most likely be the etiologic agent in this case?:
A. Vibrio vulnificus
B. Pseudomonas aeruginosa
C. Mycobacterium marinum
D. Nocardia asteroids
66. A microbiology student noticed that a Fluid Thioglycollate culture broth was very turbid at the surface and turbid
throughout the rest of the tube. She can conclude that the:
A. Organisms cannot tolerate oxygen
B. Organisms are aerobes
C. Organisms should be put in a candle jar
D. Organisms are facultative anaerobes
Page 11 of 17
MICROBIOLOGY & PARASITOLOGY
67. All of the DNA elements carried within a bacterium – whether plasmid, a cloned fragment of DNA carried by a
vector, or the chromosome – must share which of the following?:
A. Common promoter sequences
B. Common ribosome binding sites
C. Bacterial origins for DNA replication
D. Common repressor binding sequences
68. Serologic diagnosis is an important tool in infectious disease treatment because:
A. It detects the presence of the specific antigen
B. It is useful even if the etiologic agent is difficult or impossible to isolate
MICROBIOLOGY
Page 125 of 137
C.
D.
It is a reflection of current infection
It is effective earlier in infection than any type of direct culture
69. Fungal cultures should be held and observed for growth for at least:
A. 72 hours
B. 1 week
C. 1 month
D. 2 months
70. A laboratory contaminant that produced blue-green growth and microscopically showed branching septate
hyphae, large, flask-shaped vesicles, phialides, uniserated heads and parallel chains of conidia covering the
upper half of the vesicle is most likely:
A. Rhizopus
B. Aspergillus
C. Absidia
D. Penicillium
71. Which of the following is NOT appropriate when discussing cultures of blood for the recovery of bacteria?
A. No more than 3 cultures should be drawn in one day
B. Should be drawn before the expected fever spike
C. Cultures are incubated aerobically and anaerobically
D. Collect 5 ml of blood for optimal recovery of pathogen
72. When clinical specimens are being processed for the recovery of Mycobacterium tuberculosis, the generally
recommended method for digestion and decontamination of the sample is:
A. Sodium hypochlorite
B. Trisodium phosphate
C. N-acetyl-L-cysteine
D. Potassium hydroxide
Page 12 of 17
MICROBIOLOGY & PARASITOLOGY
73. Effective vaccines presently in use against meningococcal disease contain which of the following as the primary
immunizing agent?:
A. heat-killed bacilli
B. lipopolysaccharide
C. major outer-membrane protein
D. capsular polysaccharide
74. The mycobacteria are described as “acid-fast” because:
A. The organisms cannot be stained with acidic dyes
B. The organisms are rapidly decolorized with acid-alcohol
C. Once stained, the organisms cannot be decolorized with acid-alcohol
D. The organisms are easily stained with acidic dyes
75. You are growing bacterial cells in culture and notice that the cells do not look very healthy. After some checking
you discover that there is a lot of lactic acid in the culture fluid. What is probably wrong with this culture?:
A. Too much sugar is in the medium
B. Ethyl alcohol is being produced in excess
C. The cells do not have enough oxygen
D. Glycolysis is being inhibited
76. What is the cause of anemia associated with ancylostomiasis?:
A. Bone marrow depression caused by parasitic invasion
B. Vitamin B12 deficiency caused by competitive absorption by the parasite
C. Malabsorption of folic acid as a result of chronic diarrhea
D. Persistent iron loss
77. A child who plays in dirt contaminated with human and pet feces is susceptible to which of the following set of
parasites?:
A. Ascaris lumbricoides, Trichuris trichiura, Trichinella spiralis, Wuchereria bancrofti
MICROBIOLOGY
Page 126 of 137
B.
C.
D.
Loa loa, Capillaria philippinensis, Enterobius vermicularis, Trichinella spiralis
Strongyloides stercoralis, Toxocara canis, Ascaris lumbricoides, Necator americanus
Ancylostoma braziliense, Trichuris trichiura, Trichinella spiralis, Necator americanus
78. A scolex is recovered in human feces after the patient is treated with medication. The scolex bears four cupshaped suckers. This parasite is.
A. Diphyllobothrium latum
B. Echinococcus granulosus
C. Taenia saginata
D. Hymenolepis nana
Page 13 of 17
MICROBIOLOGY & PARASITOLOGY
79. While examining a fecal specimen, Isospora belli is suspected. The technologist would expect to see:
A. Cysts containing sporozoites
B. Oocysts that are acid fast
C. Cysts with red blood cell inclusions
D. Spores that are acid fast
80. You have decided to move to the Great Lakes area of the United States to become a sheepherder. You will be a
hermit, enjoying a completely self-sustained life by the edge of a lake with your sheepdog and sheep. Which of
the following sets of Platyhelminthes are you most likely to contract?:
A. Fasciola hepatica, Paragonimus westermani, Taenia solium
B. Schistosoma mansoni, Echinococcus granulosus, Clonorchis sinensis
C. Fasciolopsus buski, Diphyllobothium latum, Schistosoma japonicum
D. Fasciola hepatica, Echinococcus granulosus, Diphyllobothrium latum
81. Fly larvae were recovered from an open wound on the arm of a nursing home patient. This finding is called:
A. Facultative myiasis
B. Intestinal myiasis
C. Mechanical myiasis
D. Obligate myiasis
82. An 8-year old girl with respiratory symptoms comes to the emergency room. Her mother tells the physician that
her daughter has been more clumsy than usual over the past 3 days. On physical examination, an engorged tick
is found on the child’s neck. Her temperature is 37ºC, and her complete blood count levels are normal. These
symptoms suggest that the child has:
A. Babesiosis
B. Colorado tick fever
C. Lyme disease
D. Tick paralysis
83. Recovery in human feces of a 7 mm-long gravid proglottid containing eight lateral branches indicates infection
with:
A. Diphyllobothrium latum
B. Taenia solium
C. Echinococcus granulosus
D. Taenia saginata
84. A patient with AIDS returned from Thailand with acute diarrhea. The stool revealed an oval organism that was
acid-fast and fluoresced blue under ultraviolet light. The most likely identification of this organism is:
A. Giardia
B. Toxoplasma
C. Cyclospora
D. Cryptosporidium
Page 14 of 17
MICROBIOLOGY & PARASITOLOGY
85. A newspaper correspondent has diarrhea for 2 weeks after an assignment in Quezon province. You might expect
to have:
MICROBIOLOGY
Page 127 of 137
A.
B.
C.
D.
Schistosomiasis
Toxoplasmosis
Visceral larva migrans
Giardiasis
86. A retired Air Force colonel has had abdominal pain for 2 years. He makes yearly visits to his hometown in Leyte
where he enjoys wading with bare feet into streams. Which of the following should be in your differential
diagnosis?:
A. Trichinosis
B. Schistosomiasis
C. Visceral larva migrans
D. Giardiasis
87. A woman from Mindanao complains of having paroxysmal attacks of chills, fever, and sweating; these attacks
last a day or two at a time and recur every 36 to 48 hours. Examination of a stained blood specimen reveals ringlike and crescent-like forms within red blood cells. The infecting organism is most likely:
A. Plasmodium falciparum
B. Plasmodium vivax
C. Babesia microti
D. Wuchereria bancrofti
88. A 56-year old missionary in Mindanao is in acute renal failure. Two months earlier, the patient had been
diagnosed with malaria and treated with Chloroquine. Recrudescence occurred after the first month of therapy.
The recent deterioration in his kidney function followed a febrile crisis during which the patient noted that his
urine was very dark. The most likely cause for this patient’s acute renal failure is:
A. acute dehydration
B. bilirubin toxicity
C. deposition of immune complexes in the glomeruli
D. massive intravascular hemolysis
89. A pear-shaped flagellate with an undulating membrane extending one-half the length of its body with four
recurrent flagella but no anterior flagellum was recovered from a urine specimen. This is most likely:
A. Giardia lamblia
B. Trichomonas hominis
C. Trichomonas vaginalis
D. Trichomonas tenax
Page 15 of 17
MICROBIOLOGY & PARASITOLOGY
90. A 3-year old girl was admitted to the hospital with severe emaciation and anemia secondary to a long term
bloody diarrhea. While hospitalized, she experienced several episodes of rectal prolapse. Numerous white
worms were seen attached to the surface of the prolapsed rectal mucosa. This patient most probably has:
A. Diphyllobothrium latum
B. Necator americanus
C. Enterobius vermicularis
D. Trichuris trichiura
91. An ameba isolated from a patient with keratitis has spiny pseudopods, well-defined ectoplasm and endoplasm,
and no flagellar phase noted. It is most likely:
A. Iodamoeba
B. Cryptosporidium
C. Naegleria
D. Acanthamoeba
92. The host where the asexual or larval phase of the parasite occurs is known as the:
A. Incidental host
B. Intermediate host
C. Definitive host
D. Carrier
MICROBIOLOGY
Page 128 of 137
93. Finding intraerythrocytic trophozoite, schizont and gametocyte stages in stained thin blood smears during febrile
periods is the usual means of diagnosis of which of the following?:
A. Malaria
B. Toxoplasmosis
C. Visceral leishmaniasis
D. Babesiosis
94. Initial gastritis followed by hypereosinophilia and vasculitis, and ultimately myositis, are symptoms of clinical
cases of which of the following:
A. Cysticercosis
B. Sparganosis
C. Trichinosis
D. Visceral larva migrans
95. Disseminated intravascular coagulation resulting in decreased microcirculation and tissue anoxia is responsible
for the severe renal and cerebral manifestations of:
A. Amebic meningoencephalitis
B. Malignant malaria
C. Neonatal toxoplasmosis
D. Urinary schistosomiasis
Page 16 of 17
MICROBIOLOGY & PARASITOLOGY
96. Ulcerative colitis describes the intestinal lesions seen in clinical cases of which of the following?:
A. Ascariasis
B. Amebiasis
C. Giardiasis
D. Enterobiasis
97. The modified acid-fast stain is most often used in parasitology to identify:
A. Protozoan cysts and trophozoites
B. Helmonth eggs
C. Plasmodium
D. Cryptosporidium and other Coccidia
98. The observation of embryonic flame cell activity is recommended to demonstrate the viability of the eggs of:
A. Clonorchis sinensis
B. Fasciola hepatica
C. Schistosoma japonicum
D. Taenia solium
99. Enterobius vermicularis infection is usually diagnosed by:
A. Finding adult worm in feces
B. Finding larvae in feces
C. Finding eggs in the feces
D. Finding eggs in the perianal specimens
100. Which of the following findings in a peripheral blood smear is especially associated with tissue-invading
helminths but may also be found in a variety of allergic conditions and other diseases?:
A. Anemia
B. Eosinophilia
C. Leucopenia
D. Neutropenia
Angeles University
COLLEGE OF MEDICINE
REFRESHER COURSE EXAMINATION
MICROBIOLOGY AND PARASITOLOGY
January 09, 2004
MICROBIOLOGY
Page 129 of 137
Name: _________________________________
SCORE: ________________
I. CHOOSE THE CORRECT ANSWER.
_____ 1.
Mycobacterial lipid:
A. Responsible for the acid fastness of the organism
B. Makes the organism resistant to the effects of the complement and antibodies
C. Responsible for their serpentine growth
D. All of the above
_____ 2.
In primary tuberculosis…
A. The person becomes sensitie to the tuberculo protein of the organism
B. When the infection resolves, the Gohn complex is seen in the hilar region of the lungs
C. Dormant bacilli are establish
D. All of the above
_____ 3.
Culture medium used for M. tuberculosis:
A. Lowenstein-Jennsen medium
B. Blood agar
C. Subouraud dextrose agar
D. TCBS
_____ 4.
The most definitive tool in the diagnosis of tuberculosis due to M. tuberculosis is:
A. Acid-fast staining
C. Culture
B. Chest X-ray
D. Serology
_____ 5.
One of the following is NOT a part of the tubercle bacilli complex:
A. M. africanum
C. M. bovis
B. M. canis
D. M. tuberculosis
_____ 6.
Controlling tuberculosis means:
A. Finding cases and treating them
B. Seeing to it that the cases will finish the full course of treatment
C. Both
D. Neithr
_____ 7.
The advantage of the Bactec method over the conventional culture for TB is:
A. It is highly specific for M. tuberculosis
C. It is a rapid test
B. It is cheap
D. All of the above
_____ 8.
In pulmonary tuberculosis is the host response is primarily:
A. Cell mediated immune response
C. Both
B. Humoral immune response
D. Neither
_____ 9.
The most common mode of transmission of HIV:
A. Homosexual and heterosexual intercourse
B. Transfusion of blood and blood products
C. Mother to child
D. All of the above
Possible outcome of HIV infection of T helper cells:
A. Stay latent
B. Go into productive cycle
C. Both
D. Neither
_____10.
_____11.
_____12.
_____13.
The decline in the antibodies against this HIV marker heralds the beginning of AIDS
symptoms:
A. Anti-p24
C. Anti-gp41
B. Anti-gp120
D. None of the above
Which of the following is an AIDS indicator disease?
A. Toxoplasmosis of brain
C. Kaposis sarcoma
B. PC pneumonia
d. All of the above
The test used to confirm HIV infection is:
A. ELISA
C. Northern blot
MICROBIOLOGY
Page 130 of 137
B. Southern blot
D. Western blot
_____14.
Which is most commonly used for the treatment of HIV?
A. Zidovudine
C. Amantadine
B. Saquinavir
D. Rifampicin
_____15.
A specific virologic diagnosis of HIV can be achieved by:
A. Detection of viral components in the plasma, such as p24
B. Presence of anti-HIV in the serum
C. Isolation of virus
D. All of the above
_____16.
Most common viral cause of diarrhea in infancy:
A. Adenovirus
B. Norwalk virus
C. Rotavirus
D. Polio virus
_____17.
Which of the following protozoans has a “smiling face”?
A. Giardia lamblia
C. Trichomonas vaginalis
B. Chilomastix mesnili
D. Balantidium coli
_____18.
Characteristics of nematodes:
A. They are roundworms
B. They are flatworms
C. They are unicellular
D. They are hermaphroditic
Trichuriasis may cause
A. Intestinal obstruction
B. Pruritus ani
C. Pneumonitis
D. Rectal prolapse
Enterobiasis:
A. Common cause of pruritus ani
B. Diagnosed by direct fecal smear
C. Has an indirect life cycle
D. Also known as whipworm
This parasite does not require development in soil:
A. Trichuris trichiura
B. Ancylostoma duodenale
C. Ascaris lumbricoides
D. Enterobius vermicularis
_____19.
_____20.
infection
_____21.
_____22.
The larvae of this parasite migrate via the pulmonary route:
A. Trichuris trichiura
C. enterobius vermicularis
B. Capillaria philippinensis
D. Ascaris lumbricoides
_____23.
This will differentiate E. histolytica from E. coli
A. Number of nuclei in trophozoite
B. Size of trophozoite
C. Shpae of nucleus in trophozoite
D. Presence of RBC inside
trophozoite
_____24.
A 2-year-old passed out a “big worm” that looks like an earthworm. Most likely diagnosis:
A. Ascaris lumbricoides
C. Trichuris trichiura
B. Enterobius vermicularis
D. Strongyloides stercoralis
_____25.
Which of the following will most likely cause intestinal obstruction?
A. Trichuris trichiura
C. Capillaria philippinensis
B. Ancyiostoma duodenaie
D. Ascaris lumbricoides
_____26.
Etiologic agent of traveler’s diarrhea:
A. ETEC
C. EIEC
B. EPEC
D. Vibrio cholerae
Typhoid fever is characterized by the following, EXCEPT:
A. Splenomegaly
C. Rose colored spots
B. High grade fever
D. Rice water stools
_____27.
_____28.
Cholera is characterized by the following, EXCEPT:
MICROBIOLOGY
Page 131 of 137
_____29.
A. Washerwoman’s hands
B. Rice water stool
C. Sunken eyes
D. Mucoid and bloody diarrhea
The following are gram-negative curved rods:
A. Vibrio cholerae
B. Campylobacter jejuni
C. Yersinia enterocolitica
D. Only A and B are correct
_____30.
Shigellosis is characterized by the following, EXCEPT:
A. Tenesmus
C. Vomiting
B. Copious water diarrhea
D. Stool with blood and mucus
_____31.
The best specimen to collect during the 1st week of typhoid fever is:
A. Blood
C. Both A and B
B. Stool
D. Either A or B
_____32.
Vibrio parahemolyticus is associated with:
A. Poultry
B. Meal and meat products
C. Seafoods
D. Dairy products
Campylobacteriosis is usually seen in:
A. Adults
B. Teen-agers
C. <10 years old
D. Neonates
_____33.
_____34.
Which of the following produce disease similar to Shigellosis?
A. ETEC
C. EPEC
B. EIEC
D. EHEC
_____35.
Which of the following produce Shigalike toxin?
A. ETEC
B. EIEC
C. EPEC
D. EHEC
Larvae are passed out in feces:
A. Capillaria philippinensis
B. Ascaris lumbricoides
C. Strongyloides stercoralis
D. Ancylostoma duodenale
_____36.
_____37.
This organism produce a toxin whose action is similar to choleragen:
A. ETEC
C. EPEC
B. EIEC
D. EHEC
_____38.
The following has carrier state, EXCEPT:
A. Salmonella typhi
B. Shigella sonnei
C. Vibrio cholerae
D. ETEC
In cholera cases the first line of treatment is:
A. Tetracycline
B. Fluid & electrolyte replacement
C. Both A and B
D. None of the above
Shigella sp. Move from one cell to the next by:
A. GM1 receptor
B. CFA1
C. Actin polymerization
D. None of the above
_____39.
_____40.
_____41.
Which symptom is absent in food poisoning caused by common bacterial agents?
A. Vomiting
C. Fever
B. Diarrhea
D. Dysphagia
_____42.
Food poisoning due to which organism is most likely to cause a flaccid muscle paralysis?
A. Clostiridum botulinum
B. Staphylococcus aureus
C. Bacillus cereus growing in cooked meat
D. Clostridium perfringens
MICROBIOLOGY
Page 132 of 137
_____43.
A 4-year-old female presents with a history of diarrhea and violent vomiting that occurred
last night as well as one night a week ago. No gastrointestinal disease symptoms were
noted in between the two events. The child’s meal last night consisted of ham and
cheesy-potato leftovers. The child also shows honey-crusted dried lesions below her
nose and in the near facial vicinity. You are not pleased with the child’s level of hygiene
and the mother’s obvious personal problem with folliculitis on her face and lower arms.
Culture of a fecal swab from the child produces a few pinkish-red colonies on MacConkey
agar that were later shown to be negative for both citrate utilization and urease production.
Based on the data available, which etiologic agent seems most likely to have caused the
child’s emesis?
A. Bacillus cereus
C. Streptococcus pyogenes
B. Escherichia coli
D. Staphylococcus aureus
_____44.
Injectable therapeutics used for C. botulinum food poisoning include, the following,
EXCEPT:
A. Guanidine HCI
C. Edrophonium
B. Trivalent (ABE) botulinum antitoxin
D. 4-aminopyridine
_____45.
Presumptive diagnosis of botulism is made by:
A. Demonstrating botulinal toxin in serum
B. Presence of rapidly ascending paralysis
C. Presence of botulinal toxin in feces
D. Presence of rapidly descending paralysis
_____46.
Bacterial food poisoning commonly associated with fried rice:
A. Clostridium botulinum
C. S. aureus
B. Clostridium perfringens
D. B. cereus
_____47.
The following organisms can produce a non-inflammatory type of gastroenteritis,
EXCEPT:
A. C. difficile
C. C. botulinum
B. C. perfringens
D. B. cereus
_____48.
Most probable cause of food poisoning in newborns after ingestion of honey:
A. C. perfringens
C. C. botulinum
B. S. aureus
D. B. cereus
_____49.
Drug of choice for Staphylococcal food poisoning:
A. Erythromycin
B. Penicillin
C. Clindamycin
D. Chloramphenicol
_____50.
Most important management for non-inflammatory type of gastroenteritis:
A. Antibiotics
C. Antitoxin
B. Replenishment of fluids
D. None of the above
_____51.
A carrier of HBV will have the following markers:
A. Anti-Hbe (+); Anti-HBs (+), HbeAg (+)
B. Anti-Hbe (-); Anti-HBs (-), HbeAg (+)
C. Anti-HBc IgM (+); Anti-HBs (+), HbeAg (+)
D. Anti-HBc IgG (+); Anti-HBs (+), HbeAg (+)
_____52.
Infectiousness in HBV is a measure of:
A. HBc Ag
C. HBs Ag
B. Hbe Ag
D. All of the above
When baby is born to a mother who is HBV carrier, which of the following should be
immediately done?
A. Immunized with Hep B vaccine within few hours after deliver
B. If mother is Hbe Ag positive, simultaneously give passive immunization at different
site
C. Isolate the baby
D. Only A and B are correct
_____53.
MICROBIOLOGY
Page 133 of 137
_____54.
Which of the following statement/s on Hep D is/are correct?
A. Needs helper virus to cause infection
B. Common among drug-users infected with HBV
C. Has an RNA genome
D. All of the above are correct
_____55.
Which of the following is/are TRUE of Hepatitis C?
A. Has an RNA genome, belongs to the flavivirus group.
B. PCR on blood samples detect viral RNA and indicates infectivity
C. The parenteral non-A, non-B hepatitis
D. All of the above are correct
_____56.
Which of the following hepatitis is also a flavivirus, but antigencially distinct from HCV, is
transmitted by blood transfusion and its role in disease is still uncertain.
A. HDV
C. HFV
B. HEV
D. HGV
_____57.
A calicivirus, has high morality rate among pregnant women:
A. HDV
C. HFV
B. HEV
D. HGV
_____58.
Though there is not enough evidence yet, this virus is regarded as mutant of HBV:
A. HDV
C. HFV
B. HEV
D. HGV
_____59.
Pathology seen in women with this virus is similar to drug induced hepatotoxicity:
A. Hep A
C. Hep F
B. Hep C
D. Hep E
_____60.
A person immunized with Hep B vaccine will have this/these antibody/ies:
A. Anti-HBs
D. All of the above
B. Anti-HBc
E. Only A and B are correct
C. Anti-Hbe
_____61.
The causative agent of Kala-azar or Dum-dum fever:
A. Echinococcus granulosus
C. Entamoeba histolytica
B. Leishmania donovani
D. Schistosoma japonicum
_____62.
The first choice of drug for the treatment of amebic liver abscess:
A. Mebendazole
C. Diethylcarbamazine
B. Emetine HCl
D. Metronidazole
_____63.
The molluscan intermediate host of Schistosoma japonicum in the Philippines is:
A. Phlebotomus sp.
C. Lutzomyla sp.
B. Oncomelania quadrasi
D. Biomphalaria glabrata
_____64.
The intermediate host of visceral leishmaniasis is:
A. Snail
B. Dog
C. Cat
D. Sandfly
The infective stage of Echinococcus granulosus:
A. Scolex
B. Embryonated egg in dog feces
C. Sporozoite
D. Amastigote
Diagnostic stage of L. donovani:
A. Amastigote
B. Trophozoite
C. Promastigote
D. Cyst
Infective stage of L. donovani:
A. Amastigote
B. Trophozoite
C. Promastigote
D. Cyst
_____55.
_____56.
_____57.
MICROBIOLOGY
Page 134 of 137
_____58.
Infective stage of Schistosoma sp. In man:
A. Amastigote
B. Cercaria
C. Miracidium
D. Trophozoite
_____59.
Infective stage of Fasciola hepatica in man:
A. Metacercaria encysted in watercress
B. Miracidium in snail
C. Sporocyst and redia in snail
D. Cercariaa in water
_____60.
The “small fox tapeworm” which causes alveolar echinococcus:
A. E. granulosus
C. E. vogeli
B. E. multilocularis
D. E. oligarthrus
_____61.
It is also known as the “Chinese liver fluke”.
A. Fasciola hepatica
B. Schistosoma japonicum
C. Entamoeba histolytica
D. Clonorchis sinensis
It is also known as the “cat liver fluke”.
A. Fasciola hepatica
B. Opisthorchis viverinii
C. Clonorchis sinensis
D. Opisthorchis felineus
It is also known as the “sheep liver fluke”.
A. Fasciola hepatica
B. Schistosoma japonicum
C. Entamoeba histolytica
D. Chlonorchis sinensis
The intermediate host of Schistosoma sp.:
A. Snail
B. Dog
C. Cat
D. Sandfly
_____62.
_____63.
_____64.
_____65.
The intermediate host of L. donovani belong to the following:
A. Phlebotomus
C. Lutzomyia
B. Anopheles
D. A and C only
_____66.
The alpha viruses and flavi viruses are classified based on:
A. Neutralization
C. Complement fixation assay
B. Hemagglutination – inhibition
D. All of the above
_____67.
Which of the following arthropods is not a vector of the arboviruses?
A. Culex mosquitoe species
C. Anophelined mosquitoe species
B. Aedes mosquitoe species
D. ixodes (ticks)
_____68.
Arboviral infections are characterized by:
A. Short prodrome
B. Mild systemic disease in the majority of cases
C. Antibody rises when fever subsides
D. All of the above
_____69.
Members of the flaviviruses exhibit cross-reaction in serologic tests-against:
A. Core antigens
C. Core enzymes
B. Envelope antigens
D. All of the above
_____70.
Which is TRUE of Dengue Infections?
A. The dengue virus multiples in the gut of mosquitoes.
B. Human to human transmission is a problem.
C. Immunity to one serotype leads to life long immunity to the others.
D. The best mode of control is to kill adult mosquitoes.
_____71.
The pathogenesis of dengue hemorrhagic fever/dengue shock syndrome (DSS) is based
on:
A. Previous infection with a different type
B. Formation of infectious immune complex i.e. virus + non-neutralizing IgG
MICROBIOLOGY
Page 135 of 137
C.
D.
Immune clearance
All of the above
_____72.
In the WHO grading of Dengue which of the following is/are true?
A. All stages have thrombocytopenia and hemoconcentration
B. Grade III and IV are already considered DSS
C. All stages will exhibit positive tourniquet test
D. All of the above
_____73.
Laboratory diagnosis of dengue infection includes:
A. Clinical monitoring; serial hematocrit determination
B. Virus isolation and serology
C. Tourniquet test
D. All of the above
_____74.
The method/s of prevention and control of dengue infections includes:
A. Elimination of breeding places of mosquito vector
B. Killing of adult mosquitoes
C. Personal protection such as use of insect repellant
D. All of the above
_____75.
Which is true of cytomegalovirus infection?
A. Caused only by one serotype
B. Primary disease is rare but infection is widespread
C. Infections in the baby can be congenital or acquired postnatal
D. All of the above
_____76.
The cytomegalovirus (CMV):
A. Is a DNA virus
B. Produces the biggest intrnuclear inclusion body
C. Is carried by macrophages and lymphocytes to tissue
D. All of the above are characteristics of CMV
E. None of the above is a characteristic of CMV
_____77.
Which of the following is not used in CMV infection?
A. Ganciclovir
B. Foscarnet
_____78.
Which is true of Epstein-Barr virus?
A. Has oncogenic properties
B. Persists in the lymphocytes
D. Can grow in suspension of human lymphoblast
C. Amantadine
D. None of the above
D. All of the above
E. Only A and B are correct
_____79.
Which is not true of Purtillo’s syndrome?
A. Seen among boys
B. Fatal infectious mononucleosis in the patient
C. Patient has x-linked lymphoproliferative syndrome with immunodeficiency
D. All of the above
E. Only B and C are correct
_____80.
Which is true of EBV infections?
A. Most infections are mild and symptomless
B. Can transform lymphocytes
C. Associated with nasopharyngeal Ca
D. All of the above
_____81.
The following can cause Endocarditis, EXCEPT:
A. C. diphtheria
B. Viridans strep
_____82.
C. S. aureus
D. S. epidermis
All of the following can cause Myocarditis, EXCEPT:
A. B. burgdorferi
C. R. tsutsugamushi
MICROBIOLOGY
Page 136 of 137
B. C. perfringens
D. P. aeruginosa
_____83.
The following bacterial agents can cause pericarditis, EXCEPT:
A. H. influenza
C. N. meningitides
B. S. pneumonia
D. S. epidermis
_____84.
Which of the following is associated with dental caries and subacute bacterial
endocarditis?
A. S. aureus
C. S. epidermis
B. Viridans strep
D. S. fecalis
_____85.
Which of the following is associated with late prosthetic valve endocarditis?
A. S. aureus
C. S. epidermis
B. Viridans strep
D. S. fecalis
_____86.
Most frequent cause of early prosthetic valve endocarditis:
A. S. aureus
C. S. epidermis
B. Viridans strep
D. S. fecalis
_____87.
Which of the following can cause bacterial endocarditis in debilitated or
immunocompromised patients?
A. Group A streptococci
C. S. aureus
B. Group D streptococci
D. S. viridans
_____88.
Drug of choice for methicillin-resistant staphylococci:
A. Cephalosporins
C. Clindamycin
B. Erythromycin
D. Vancomycin
_____89.
Which of the following is associated with rheumatic fever?
A. S. pyogenes
C. S. aureus
B. Viridans strep
D. S. epidermis
_____90.
Which of the following can cause myocarditis after absorption of a preformed exotoxin?
A. S. aureus
C. Viridans strep
B. S. epidermis
D. C. diptheria
_____91.
Drug of choice in patients who are allergic to penicillins:
A. Vancomycin
C. Cephalosporin
B. Erythromycin
D. Clindamycin
_____92.
Waterhouse Friderichsen syndrome characterized by pericarditis and bilateral
hemorrhagic destruction of the adrenals is associated with one of the following:
A. N. meningitides
C. S. epidermis
B. S. sanguis
D. S. mutans
_____93.
All of the following belong to viridans streptococcus, EXCEPT:
A. S. mitis
C. S. agalactiae
B. S. sanguis
D. S. mutans
_____94.
Which of the following is TRUE of S. pyogenes?
A. Group A α hemolytic
B. Group A β hemolytic
C. Group D α hemolytic
D. Group B β hemolytic
Which of the following organisms is a strict aerobe?
A. Borrelia burgdorferli
B. Treponema pallidum
C. Leptospira interrogans
D. All of the above
_____95.
_____96.
The cell wall peptidoglycan of Leptospira spp. Contains:
A. Diaminopimelic acid
C. Ornithin
B. 4-0 methyl mannose
D. All of the above
_____97.
The source of energy/carbon of Leptospira spp. Is from:
MICROBIOLOGY
Page 137 of 137
_____98.
_____99.
_____100.
A. Amino acids
B. Long chain fatty acids
C. Protein and lipids
D. All of the above
Leptospirosis is also known as:
A. Weil’s disease
B. Woolsorter’s disease
C. Burkitt’s syndrome
D. None of the above
Leptospira manilae was isolated in:
A. 1963
B. 1975
C. 1960
D. 1957
All of the following are pathogenic leptospira, EXCEPT:
A. Leptospira interrogans
C. Leptospira biflexa
B. Leptospira santarosai
D. Leptospira borgpetersenii
-end-
MICROBIOLOGY